Vous êtes sur la page 1sur 207

MCS University Paper Solutions May 2001 to May 2011

MAY 2011 Q # 1.) Define MCS. Which levels of Managers are involved in it? How does MCS differ from Simpler Control processes? A # 1.) MCS is a process by which management influences other members of the organisation to implement the organisations strategies effectively and efficiently. MCS is used by management to exercise control over the implementation of strategies

MCS differs from planning and control systems namely strategy formulation, operational control and financial control in the following ways: 1. Systematic Strategy formulation is least systematic, operational control is the most systematic, whereas management control lies in between. 2. Focus Strategy formulation focuses on long run, operational controls on short term operating activities and management control system lies in between.
3. Estimates Strategy formulation is based on rough approximations of the future,

Operational control makes use of accurate data and MCS lies in between. 4. Degree of variation Planning is more important than control in strategy formulation. While control process is more important in operational control. However, in MCS planning and control both are of equal importance. Q # 2.) Briefly describe Responsibility centre, engineered expense centre, discretionary expense centre, revenue centre, profit centre. How is the performance of the head of these centres evaluated? A # 2.)
1. Responsibility centre A responsibility centre can be defined as an organisational unit

which is headed by a responsible person namely a manager. He is responsible for the activities of the unit. Responsible centre is responsible for performing certain functions which is its output.

2. Engineered expense centre - engineered costs are the costs for which the right or

proper amount can be estimated with reasonable reliability. For example, cost of material, labour, supplies, components and utilities, etc.
3. Discretionary expense centre - discretionary costs are those costs for which no such

engineered estimates are possible. The costs incurred in the discretionary expense centre depend upon judgement of the management as to the appropriate amount under the circumstances.
4. Revenue centre In revenue centre, output is measured in monetary terms. However,

there is no formal relationship between input and output. Marketing organisations are the examples of revenue centres where no responsibility for profit exists.
5. Profit centre A profit centre is an organisational unit in which both revenues and

expenses are measured in monetary terms. Profit is a useful performance measure of a responsibility centre. Thus the performance in a responsibility centre is measured in terms of relevant revenue it earns and the cost it incurs. Measures of performance management can be broadly defined in 2 categories 1.) Financial performance measures Return on Investment Residual Income Earnings per share Net present value Economic Value added

2.) Non-financial performance measures Balanced score card Benchmarking

Q # 3.) Every SBU is a profit centre but not every profit centre may be SBU. Explain. Under what conditions production, marketing and service dept. are converted into profit centre.

A #3.) Conditions for an organization to be converted into a profit centre: The manager should have access to the relevant information needed for making such a decision. There should be some way to measure the effectiveness of the trade-offs the manager has made.

A major step in creating profit centres is to determine the lowest point in an organization where these two conditions prevail.

Production dept.: Production department can be converted into a profit centre by the manufacturing department to be credited with the sales proceeds of the products. The estimated marketing expenses are debited. Other factors which influence on the mix and volume of sales can produce better results if they are designed properly.

Marketing dept.: When the marketing department head is the best judge, regarding principal cost and revenue trade-offs, the marketing dept is treated as a profit centre. These expenses should be charged with the cost of product sold for the purpose of converting a marketing dept into a profit centre.

Service dept.: The dept is credited with the charges for services rendered to other departments in the organization. The service charges are calculated on a reasonable basis and the departments using the services are charged on a reasonable basis. The departments using the services can be given option of availing of such services from other firms provided they offer the same quality and lower price.

Q # 4.) When are market based transfer prices most appropriate? How do we deal with the condition of Limited Market, Situation of excess/shortage of capacity?

A #4.) Market based transfer prices are most appropriate when they are determined by the forces of demand and supply in the long run. The ideal situation when a market based transfer price will induce goal congruence if following conditions exist: Competent people Good atmosphere A market price Freedom to serve Full information Negotiation

In case of Limited markets, the transfer price that best satisfies the requirements of a profit centre system is the competitive price. Competitive prices measure the contribution of each profit centre to the total company profits. Moreover, a competitive price measures how well a profit centre may be performing against competitors. In situation of excess capacity, the selling department does not sell in the outside market, whereas the buying department may buy from outside vendors, though the inside capacity is available in the company. However the company as a whole may not optimize its profits. A cost-based transfer price using the variable cost of production will align incentives. Q # 5.) What do you understand by Investment Centre? Explain two different methods by which the performances of these centres are measured? Also discuss their relative merits and demerits. A #5.) An Investment Centre is a responsibility centre in which the manager is held responsible for the use of assets as well as for revenues and expenses of the centre. The manager is expected to earn a satisfactory return on capital employed in the business units. The two different methods by which the performance of Investment centre is measured are: 1.) Return on Investment (ROI)= Operating profit / Capital employed * 100

Merits 1. To measure the operating performance of the organisation 2. To evaluate and control the capital expenditure projects 3. To make profit planning 4. To analyse the profit by operating divisions 5. To analyse the profit by product line 6. To price new products 7. To analyse major cost areas in a cost reduction programme 8. To determine the relative profitability of different projects

De-merits 1. Manipulative 2. Different bases for computation 3. Emphasis on short term profits 4. Poor measure 5. Allocation of resources 2.) Economic Value Added (EVA)= NOPAT Cost of Capital Merits 1. Helps measure corporate performance and performance of business segment 2. Tells how managers are creating wealth 3. Most appropriate determination of cost of capital 4. Strong tool for business planning De-merits

1. failure to consider the future prospects of the company 2. Requires a lot of adjustments to financial information company
3. Requires a tradeoffs between accuracy and simplicity of calculation, since very

complicated adjustments result in a lack of credibility. 4. Doubt about universal suitability of EVA. Q6) 6) Types of Organization: a. Functional Organization i. Oldest ii. According to Functions iii. Headed by an expert iv. Special qualification needed v. Better supervision and Control vi. Higher efficiency
vii. Division of Labour

viii. Specialization ix. Old, single product organizations x. No diversification allowed b. Divisional Organization i. Specific product line ii. As a separate company iii. Responsible for planning and coordination iv. Profitability of a division is performance measure

v. Can buy/ sell / modify plant/ product vi. A company within company vii. Specialized knowledge related to product viii. Complex Business ix. Entrepreneurial spirit x. Duplication of staff c. Matrix Organization i. Combination of Functional and Divisional ii. Each division has target iii. Project draws personnel from function iv. 2 bosses v. Better planning and control vi. More flexible vii. Improved communication viii. Conflict between 2 or more functions ix. No unity of command Most appropriate form for control is Functional Q7) 7) Non Profit Organizations a. Cannot distribute asset or income b. Employee compensation is allowed c. Special purpose d. No Profit

e. Exempted from Income tax f. Characteristics i. Contributed capital ii. Absence of profit iii. Fund accounting iv. Financial accounting v. Governance Product Pricing a. Full cost b. Peripheral activities at market price c. Cost of admin work to be added Performance Evaluation a. Difficult as different activities b. Budget allocation should match spending

Q8 8) a. Implication of Organizational structure Sr No 1 Organizational Structure 2 Industry Familiarity of Corporate mgmt High Particulars Single Industry Functional Related Diversification Business Units Unrelated Diversification Holding company Low

Functional Background

Relevant operating exp More centralization Large

Mainly Finance

Decision-making authority

More decentralization

Size of corporate staff

Small

6 7 8

Internal promotion Lateral transfers Corporate culture

High High strong

Low Low weak

Q 09 9) SN a. Marketing by Service Organizations i. No clear distinction between Marketing and production ii. Difficult to assign responsibility for a particular sale iii. Subjective rewards iv. Sometimes as a percentage of project revenue b. Balance Scorecard i. Linking of Financial and Non-financial measures of performance ii. A set of measures that give fast comprehensive view of the business iii. Result of failure of traditional cost system iv. Emphasis on the external reporting more than internal decision making v. Better measure for Service industry

vi. Aim of BS is to translate organizations strategic objectives to coherent set of performance measures vii. Cause and effect relationship viii. Long term benefits for the business ix. Four basic measures: 1. Financial 2. Customer 3. Internal business and production process 4. Learning and growth c. Interactive Controls i. Objective of IC is to facilitate the creation of a leaning organization ii. Help to cope with changing environment iii. Strategic uncertainties guide the use of a subset of management control information interactively in developing new strategies iv. Critical success factors are imp v. Critical success factors are derived from chosen strategies vi. They support implementation of strategies vii. Strategic uncertainties are base for developing new strategies viii. Alert system of troubles or opportunities ix. Managers and subordinates meet face to face for discussion x. They are not separate systems xi. They are integral part of the MCS

10)Problem /unit Units Utilization Mat cost Manu cost Var selling cost Var Admin Cost Var Total Cost Profit S.P. 5 90 10 100 100000 1800000 200000 2000000 160000 2728000 312000 3040000 200000 3368000 312000 3680000 Fixed 6 5 120000 100000 192000 100000 240000 100000 Fixed 21 4 420000 80000 672000 80000 840000 80000 Fixed 9 180000 180000 180000 40/42/43.2 1 50% 20000 50% 800000 80% 32000 80% 1344000 100% 40000 100% 1728000

Company should work at 80%

Q11) The actual sales are lesser than the budgeted sales. To find out the reason for this I would investigate mainly 2 areas-

1. Sales promotion-

This is a major driving factor for sales at all levels. The actual spend on sales promotion is lesser than budgeted which means that not enough resources were directed towards sales promotion which has resulted in lesser sales.

2. Fixed AssetsThe investment in fixed assets was also less than the budgeted amount. This may imply that the production was hampered due to less investment in machinery or plant and equipment.

Q12)

A Capacity Cost of production: Material Cost Processing cost Fixed Cost per unit Transfer cost Required Return 10 20 20 20 70 10000

B 10000

C 10000

10 10 20 70 30 140

10 10 10 140 40 210

Required Return for A= (10%*2000000)/10000= 20 Required Return for B= (15%*2000000)/10000= 30 Required Return for C= (20%*2000000)/10000= 40

Product transferred from to A to B- Rs. 70

Product transferred from B to C- Rs. 140 Minimum price C should charge an external customer- Rs. 210

If the price of the product falls to Rs. 200, the company will not earn the required return. Therefore unless the company is willing to reduce its profit expectations, it should not manufacture the product.

Q13) A 2010 ROTA EVA 12% 42 2011 13% 40 B 2010 13% 48 2011 12% 56

ROTA= PAT/TA EVA for A, 2010= 72- 30 (10%* equity Capital) EVA for A, 2011= 65- 25 EVA for B, 2010= 78- 30 EVA for B, 2011= 96- 40 (Assume Debt Equity ratio of 1:1 as no details are mentioned in the sum)

Based on ROTA division A has improved performance but based on EVA the division B has performed better. I feel Manager of division B is right as his division is adding value to the shareholders which is the basic aim of running any business.

MAY 2010 Q1) Management control lies between strategy formulation and operational control. MCS has certain limitations which are called as boundaries; that distinguish it from other planning and control systems namely strategy formulation, operational control and financial control. Strategy formulation has found to be the least systematic, operational control, the most systematic and MCS falling in between. Strategy formulation focuses on the long run, operational control on short term operating activities and MCS lies in between. Strategy formulation is based on rough approximations of the future, operational control makes use of current accurate data and MCS lies in between. The three concepts are explained as follows1. Strategy formulationIts a planning process used by firms to decide on the goals of the organization and strategies to be used for achieving these goals. Goals are the overall aims of the organisation. Strategies are the plans for achieving the goals. Once a firm has formulated its strategies, it operates in accordance with the strategies. They may be re examined during the annual strategic planning exercise and some of them may be changed or modified. Steps in strategy formulation are as followsa. Framing mission and objectives b. Analysis of internal environment c. Analysis of external environment d. Gap analysis e. Framing alternative strategies f. Choice of strategy

2. Operational control-

Its a process used for ensuring the tasks which are specified are carried out efficiently and effectively. It involves the control of individual tasks. The rules to be followed for accomplishing the tasks are prescribed as part of the management control process. Many operational control activities are scientific in nature.

3. Management ControlIt involves the process of implementing strategies. There is generally a fixed time table and a series of steps in accordance with which management control takes place. It is a type of planning and control activity that is done by the organization. It is a process by which management influences other members of the organisation to implement the strategies effectively. Thus management control involves the behaviour of managers as managers interact with other managers and this cannot be shown in the form of equations.

Q2) Expense centres are Responsibility centres whose inputs are measured in monetary terms but outputs are not measured in monetary terms. In many cases it is not feasible to measure the output in monetary terms. It is very difficult to measure the monetary value that the human research department contributes to the organisation. If the control system measures the expense incurred by the organisational unit but does not measure the monetary value of its output, the unit is a expense centre. There are two types of expense centres, engineered and discretionary. 1. Engineered Expense centresEngineered costs are the costs for which the right or proper amount can be estimated with reasonable reliability e.g. costs of material, labour, supplies, components and utilities etc. In an engineered expense centre, an optimal relationship can be established between the inputs and the outputs. They are generally found in manufacturing operations. They have the following characteristics-

a. Inputs can be measured in terms of money b. Outputs can be measured in physical terms c. Optimum amount of inputs which are required to produce one unit of output can be established. The output of an engineered expense centre gives the standard cost of finished product if it is multiplied by standard cost of each unit manufactured. The difference between the actual and the engineered costs represents the efficiency of the organisational unit. 2. Discretionary expense centresDiscretionary costs are those costs for which no such engineered estimates are possible. These costs depend on the judgement of the management as to the appropriate amount under the circumstances. Discretionary expense centres include administrative and support units, R&D and marketing activities e.g. accounting legal, industrial relations, public relations and human resources. The output of these centres cannot be measured in monetary terms. An optimum relationship cannot be established between input and output. The difference between budgeted input and actual input measures the efficiency of discretionary expense centres. This does not measure the value of the output. Q3: SHORT NOTE ON ZERO BASED BUDGETING ANS: Zero based budgeting (ZBB) is an alternative approach from the traditional cost approach to budgeting. Under this method, the base line for the budget is zero rather than the last years budget. So the manger must be able to justify each budget requirement instead of simply making changes to the previous periods budget. In this way each function within an organization is analysed for its needs and costs each year and around which the budget is built. This method is usually used by governments and non-profit organizations. ZBB requires considerable documentation. In addition to all of the schedules in the usual master budget, the manager must prepare a series of decision packages in which all of the activities of the department are ranked according to their relative importance and the cost of each activity is identified. Higher level managers can then review the decision packages and cut back in those areas that appear to be less critical or whose costs do not appear to be justified.

ZBB is a good approach. The only issue is the frequency with which a ZBB review is carried out which is on a yearly basis. Critics of such type of budgeting charge that properly executed ZBB is too time consuming and too costly to justify on an annual basis. In addition, it is argued that annual reviews soon become mathematical and that the whole purpose of zero based budgeting is then lost. Whether or not a company should use annual reviews is a matter of judgment. In some situations, annual zero based reviews may be justified; in other situations they may not because of the time and cost involved. However, most managers would at least agree that on occasion zero based reviews can be very helpful. Advantages of ZBB: 1. Efficient allocation of resources, as it is based on needs and benefits. 2. Drives managers to find cost effective ways to improve operations. 3. Detects inflated budgets. 4. Municipal planning departments are exempt from this budgeting practice. 5. Useful for service departments where the output is difficult to identify. 6. Increases staff motivation by providing greater initiative and responsibility in decisionmaking. 7. Increases communication and coordination within the organization. 8. Identifies and eliminates wasteful and obsolete operations. 9. Identifies opportunities for outsourcing. 10. Forces cost centres to identify their mission and their relationship to overall goals. Disadvantages of ZBB: 1. Difficult to define decision units and decision packages, as it is time-consuming and exhaustive. 2. Forced to justify every detail related to expenditure. The research and development (R&D) department is threatened whereas the production department benefits. 3. Necessary to train managers. Zero based budgeting (ZBB) must be clearly understood by managers at various levels to be successfully implemented. Difficult to administer and communicate the budgeting because more managers are involved in the process. 4. In a large organization, the volume of forms may be so large that no one person could read it all. Compressing the information down to a usable size might remove critically important details.

5. Honesty of the managers must be reliable and uniform. Any manager that exaggerates skews the results. Q3: SHORT NOTE ON FREE CASH FLOW ANS: 1. Free cash flow (FCF) is a measure of financial performance calculated as operating cash flow minus capital expenditures. 2. FCF represents the cash that a company is able to generate after laying out the money required to maintain or expand its asset base. 3. FCF is important because it allows a company to pursue opportunities that enhance shareholder value. 4. Without cash, it is tough to develop new products, make acquisitions, pay dividends and reduce debts. 5. In corporate finance, FCF is the cash flow that is available for distribution among all the securities holders of an organization. They include equity holders, debt holder, preferred stock holders, and convertible security holders and so on. 6. We calculate FCF as: FCF = Net income + Amortization/Depreciation Changes in working Capital Capital Expenditures. 7. However, the FCF definition should also allow for cash available to pay off the companys short term debt. It should also take into account any dividends that the company means to pay. 8. Therefore,: Net FCF = Operating cash flow Capital expenses to keep current level of operation Dividends Current portion of long term debt Depreciation 9. Net of all the above fives the free cash that is available to be reinvested in operations without having to take on additional debt. 10. Here, capital expenditure definition should not include additional investment on new equipment. However, maintenance cost can be added. The dividend should be the bases dividend that the company intends to distribute to its shareholders. The current portion of long term debt is the minimum debt that the company needs to pay in order to create no defaults. Depreciation is taken out since it accounts for future investment for replacing the current property, plant and equipment. If the net income category includes the income from discontinued operation and extraordinary income, it should not be a part of free cash flow.

Q4: WHAT IS A STRATEGIC BUSINESS UNIT? WHAT ARE THE CONDITIONS REQUIRED FOR CREATING AN SBU? HOW IS THE PERFORMANCE OF SBU MEASURED? WHAT ARE THE ADVANTAGES AND DISADVANTAGES OF CREATING SBUS? ANS: Strategic business units are autonomous divisions or organizational units within an organization which are small enough to be flexible and large enough to exercise control over most of the factors affecting its long-term performance. The concept of SBU was developed by General Electric Company of USA to manage its multi-product business. In order to create a SBU, the organization needs to be large and diversified and should have multi businesses across various geographies. This is because a single corporate strategy for such type of an organization is inappropriate. Because strategic business units are more agile (and usually have independent missions and objectives), they allow the owning conglomerate to respond quickly to changing economic or market situations. An SBU does not group a distinct set of products or services, which are sold to a uniform set of customers, facing a well-defined set of competitors. The external (market) dimension of a business is the relevant perspective for the proper identification of an SBU. Therefore, an SBU should have a set of external customers and not just an internal supplier. Companies today often use the word Segmentation or Division when referring to SBUs, or an aggregation of SBUs that share such commonalities. Thus an SBU helps an organization to gain a competitive advantage over its competitors in each market while also ensuring proper allocation of resources. SBUs are also known as strategy centres, independent business unit or even strategic planning centres. Each business unit must meet the following criteria: 1. Have a unique business mission, independent from other SBUs. 2. Have clearly definable set of competitors. 3. Is able to carry out integrative planning relatively independently of other SBUs. 4. Should have a Manager authorized and responsible for its operation. Advantages of SBU: 1. Effective management 2. Proper recognition 3. Employee motivation

4. Higher efficiency 5. Better customer services 6. Facilitates innovation 7. Improvement of corporate image Disadvantages of SBU: 1. Increased expenditure 2. Difficulty in maintaining a single corporate image 3. Focus on performance of SBU may divert focus from corporates overall goals 4. Focus on short term performance 5. Distortion of information When an entity is structured into strategic business units and the performance of these units is measured in terms of accounting results, we use responsibility accounting. Managers are then held accountable and rewarded on the basis of the results of their department. This is to lessen the burden of top management by decentralising the responsibility along with authority and decision-making. This also ensures more detailed and timely information that higher level managers can use for making overall company policy-making decisions. Under this method, revenues and expenses are accumulated and reported by levels of responsibility so that actual cost is controlled by the appropriate manager responsible for its incurrence by comparison with budgeted cost. Q5: WHAT ARE THE OBJECTIVES OF A TRANSFER PRICING? WHAT ARE THE DIFFERENT METHODS TO ARRIVE AT TRANSFER PRICE? DISCUSS THE APPROPRIATENESS OF EACH METHOD. EXPLAIN WITH EXAMPLE. ANS: the objectives of transfer pricing are as follows: 1. To foster a commercial attitude in those who are responsible for the performance of profit centres. The main emphasis should be on profitability. It will force the units to improve their profit position. 2. To optimise the profit of the company over a given period of time. For this purpose the resources should be utilised to the maximum extent. 3. To make optimal use of companys financial resources. It should be based on relative performance of various profit centres, which are influenced by transfer pricing policies.

4. To enable the performance of a division to be evaluated by compensating it for benefits provided for other divisions and changing it for benefits received by the division. 5. To motivate divisional manager for maximising the profitability of their divisions acting in the best interests of the company as a whole. 6. To manage transfer prices between countries in order to minimise overall tax burden by the international companies/groups. The different methods used for arriving at a transfer price are as follows:
1. Cost-based transfer pricing: Transfer prices may be set on the basis of cost plus profit, even

though such prices may be complex to calculate and the result may be less than satisfactory than a market-based price. One must first decide how to define cost and then how to calculate the profit mark-up. Under cost-based transfer pricing, we have the following methods:
1.1. Cost of production: here, the transfer price equals the cost price or the goods or services

will be transferred at the selling divisions unit cost of production. It is the simplest and most convenient method of transfer pricing because all cost accounting information is readily available. The purpose of this method is to meet the demand for user unit and also where the responsibility for profit performance is centralised. However, it is difficult to measure the performance of each profit centre. Also the profit of the selling division is underestimated since it does on earn any profit on the goods transferred at this price. Whereas, the profit of the purchasing department would be inflated since it would be procuring at a cost that is much lower than the market given the absence of profit mark-up. (example on page 134, illustrn6.1 a)
1.2. Marginal cost: this method is used when the objective is overall profitability of the firm.

This method can be used particularly when the selling unit has idle capacity thereby leading to full utilisation of capacity. However, this method has certain problems such as additional capacity utilisation incurs additional costs which may not be accounted for, prices are difficult to predict when volumes change thereby making it difficult to predict the point at which additional production does not result in additional profits, step costs and new asset acquisitions increase cost which are not accurately estimated and accounted for and finally marginal costs will equal to marginal revenue at a certain point beyond which the selling division has no incentive to produce more units as it will result in escalating costs at no profit. (example on page 134, illustrn6.1 b)

1.3. Standard cost: the transfer price is based on the standard cost which is a pre-determined

cost. The variances from standard cost are normally absorbed by the supplying unit or sometimes transferred to the user unit. Therefore, inventories carried by both units are at standard cost. Responsibility of profit performance is centralised. Profit performance of each unit cannot be measured.
1.4. Cost of sales: also known as full cost, it allows for expenses on selling and distribution

in addition to the cost of production. However, there is no profit for the supplying unit. Measurement of divisional profit is not possible under this method.
1.5. Cost plus a normal mark-up: this transfer price includes a profit margin or normal mark-

up in addition to the cost of production. The assumption is that the supplying unit is selling to outside parties as well internal divisions, although the margins may not be the same for both. This profit is expressed as a percentage of capital employed or cost of sales. Profit performance of each unit is measurable and efficiency can be reasonable determined. As the transfer price is arrived at by adding certain percentage of total cost, the receiving division is not expected to pay for the inefficiency of the transferring division. Hence this method is not very popular.
1.6. Opportunity based transfer pricing: when the goods being transferred have an external

market, the transfer price can be based on the opportunity cost of transferring the goods internally. Opportunity cost represents the maximum contribution forgone by the supplying unit in transferring the goods internally rather than selling them in the external market. For this purpose a transfer price equal to market value is often treated as an opportunity cost. The transfer price can be negotiated by recognising the levels of output external sales and internal transfers that are best for the company as a whole. For arriving at a transfer price that ensures all divisions is to maximise their profits at the same level of output. The transfer price should be such that there is no more profitable opportunity for individual divisions. 2. Contribution Margin Transfer Pricing: Under this method, a company determines the total contribution margin earned after a product is sold externally. The company allocates this margin back to each division based on their respective proportions of the total product cost. This is used when the market prices is not available. The company can use internal information for determination of transfer price. This method can also used in special circumstances when a group benefits as a whole from doing work internally rather than accepting an external quotation. It thus eliminates

unfair distribution of profit and each division is given a share of the total contribution, possibly pro-rated to its standard variable costs. It is not the perfect approach but is a reasonably workable method for determining a suitable transfer price. This method is applicable when: (A) Several divisions contribute work to a fixed price contract with an outside customer and the final profitability remains uncertain till the contract completion (B) Where the selling price is subject to negotiation especially in the case of government contracts (C) Where the final selling price results in loss to the company as a whole.
3. Transfer pricing based on market prices: market prices are determined by the forces of

demand and supply in the long run. Therefore, the profits will provide a good indicator of the overall efficiency of the operating units. Thus transfer prices to other internal divisions are based on market prices. If the division does not trade externally, market reports are used to arrive at a market price. This method is most commonly used method. It results in the highest profits for the company. It is unbiased and effective and efficient.
4. Transfer pricing based on negotiated prices: Under this method, managers of buying and

selling divisions negotiate a transfer price between them using a products variable cost. The lower boundary is an acceptable price and the upper boundary is the market price. A negotiated transfer pricing system allows for goal congruence. This method is advantageous in that it allows divisional managers to operate their divisions in a more independent manner and not have to rely on pre-set pricing.
5. Dual Pricing: Making use of two transfer prices is known as dual transfer pricing. Due to

conflict of interest between the buying n selling divisions, arriving at a single transfer price may be difficult. The transfer price is used for decision making for one unit and performance evaluation for another. Dual transfer pricing may not be used commonly because of practical difficulties. However, it is capable of promoting goal congruence, motivation and autonomy and performance evaluation under all conditions.

Q6: HOW IS AN INVESTMENT CENTRE DIFFERENT FROM A PROFIT CENTRE? WHAT ARE THE DIFFERENT METHODS OF JUDGING THEIR PERFORMANCE? WHICH IS A BETTER METHOD? ANS: 1. An investment centre is a responsibility centre in which the manager is held responsible for the use of assets as well as for revenues and expenses of the centre. 2. A profit centre is the organisational unit in which both revenues and expenses are measured in monetary terms. 3. For an investment centre, performance is based on return on capital employed 4. However, since it is difficult to measure the investment base or the capital employed this is not the best measure of performance of an investment centre. 5. Similarly, using assets employed, valuation of fixed and current assets and valuation of liabilities becomes difficult. 6. Accounting rate of return is another fairly good measure of performance of the business manager. 7. Therefore, the best method to measure performance in an investment centre is economic value added. 8. In a profit centre, performance is measured by one comprehensive indicator rather than many indicators and that is profit which is the excess of revenues over expenses. 9. In terms of profitability, the manager can be evaluated based on his effectiveness and efficiency or actual economic profitability. 10. In case of non-profit making organisations, the term financial performance centres is used as an alternative concept. Q7: WHAT DO YOU UNDERSTAND BY BALANCE SCORECARD? EXPLAIN WITH AN EXAMPLE. ANS: The linking of financial and non-financial measures of performance and identification of key performance measures helps to devise the balance score card, a set of measures that give top managers a fast but comprehensive view of the business. It assists management in strategic policy formulation and achievement. It emphasises the need to provide the user with a set of information which addresses all the relevant areas of performance in an objective an unbiased fashion.

The aim of balanced scorecard is to provide a comprehensive framework for translating a companys strategic objectives into a coherent set of performance measures. The term became popular after an article in 1992 in the Harvard Business Review by Robert Kaplan and David Norton. It has four concepts which are- focus on strategies for each business unit, assessing lead and lag indicators which suggest financial and non-financial measures of performance, inclusion of financial and non-financial measures of performance, communication with employees.

The four basic perspectives of a balanced score card are- financial perspective (cash flow, return on equity, etc), customer perspective (%of sales from a new product, on time delivery), business and production process perspective (cycle time, unit cost) and learning and growth perspective (time to develop next generation, new product introduction)

Q8:

HOW

DOES

SERVICE

ORGANISATION

DIFFER

FROM

MANUFACTURING ORGANIZATION? HOW IS A PROFESSIONAL SERVICE ORGANISATION DIFFERENT FROM A NORMAL SERVICE ORGANISATION? HOW IS PRICING AND MARKETING DONE BY PROFESSIONAL SERVICE organisation ?

ANS: Manufacturing Organisation engaged in the production of goods (finished products) that have value in the marketplace. These Organisations are further classified into two as Process Organisation (Flow production or continuous process production industries) and Discrete Manufacturing Organisation. Service Organisation includes those Organisations that do not produce goods, but provide certain services. The peculiarity of these organisations is that often the consumption of the service takes place while it is in the generation. Typically, this sector includes hospitality, advertising, banking, insurance, consultancy, logistics, etc. The significant difference between the various types of organisations is observed when we analyze the manufacturing or service environment in which they operate. Elements of the manufacturing environment include external environmental forces, corporate strategy, business unit strategy, other functional strategies (marketing, engineering, finance, etc.), product selection, product/process design, product/process technology and management of competencies. Ultimately, what matters is the framework in which the overall manufacturing or service strategy is developed and implemented. Professional Organisation

Professional Organisation is labour intensive and the labour is of a special type. Many professionals prefer to work independently, rather than as part of a team. Professional who are also managers tend to work only part time on management activities; senior partners in the accounting firm participate actively in audit engagement; senior partners in the law firm have clients. In most professional, education does not include education in management; quite naturally it stresses the importance of the professional, rather than that of management; for this and other reason, professional tend to look down on manager. Professionals tend to give inadequate weight to the financial implication of their decisions; they want to do the best job they can, regardless of its cost. Because professional are the organisations most important resources, some authors have advocated that the value of these professional should be counted as assets. The system that does this is called Human Resource Accounting, but the problem of measuring the value of human assets is intractable. Marketing in Professional Organisation In a manufacturing company there is a clear dividing line between marketing activities and production activities; only senior management is concerned with both. Such a clean separation does not exist in most professional organisation, however. In some, such as law, medicine and accounting, the professionals ethical code limits the amount and the character of overt marketing efforts by professional. Marketing is an essential activity in almost all organisations, however. If it cant be conducted openly, it takes the form of personal contact, speeches, articles, golf and similar activities. These marketing activities are conducted by professionals, usually by professional who spend much of their time in production work that is working for clients. Pricing in Professional Organisation The selling price of work is set in a traditional way in many professional firms. If the profession is one in which members are accustomed to keeping track of their time, fee generally are related to professional time spent on the engagement. The hourly billing rate typically is based on the compensation of the grade of the professional plus a loading for overhead costs and profit. In other professions such, as investment banking, the fee typically is based on the monetary size of the security issue. In still others, there is a fixed price for the project. Prices vary widely among professions; they are relatively low for research scientists and relatively high for accountants and physicians. Q.9

Corporate Level Strategy Corporate level strategy occupies the highest level of strategic decision-making and covers actions dealing with the objective of the firm, acquisition and allocation of resources and coordination of strategies of various SBUs for optimal performance. Top management of the organization makes such decisions. The nature of strategic decisions tends to be value-oriented, conceptual and less concrete than decisions at the business or functional level. Business Unit level Strategy Business-level strategy is applicable in those organizations, which have different businesses-and each business is treated as strategic business unit (SBU). The fundamental concept in SBU is to identify the discrete independent product/market segments served by an organization. Since each product/market segment has a distinct environment, a SBU is created for each such segment. There-fore, it requires different strategies for its different product groups. Thus, where SBU concept is applied, each SBU sets its own strategies to make the best use of its resources (its strategic advantages) given the environment it faces. At such a level, strategy is a comprehensive plan providing objectives for SBUs, allocation of re-sources among functional areas and coordination between them for making optimal contribution to the achievement of corporate-level objectives. Such strategies operate within the overall strategies of the organization. The corporate strategy sets the long-term objectives of the firm and the broad constraints and policies within which a SBU operates. The corporate level will help the SBU define its scope of operations and also limit or enhance the SBUs operations by the resources the corporate level assigns to it. For example, in an organization of any size or diversity, corporate strategy usually applies to the whole enterprise, while business strategy, less comprehensive, defines the choice of product or service and market of individual business within the firm. In other words, business strategy relates to the how and corporate strategy to the what. Corporate strategy defines the business in which a company will compete preferably in a way that focuses resources to convert distinctive competence into competitive advantage.

Corporate strategy is not the sum total of business strategies of the corporation but it deals with different subject matter. While the corporation is concerned with and has impact on business strategy, the former is concerned with the shape and balancing of growth and renewal rather than in market execution. Q.10 (a) ROE = PAT________ x 100 Total Equity = (39/300) x 100 = 13% ROCE = PBIT__________ x 100 Capital Employed = (80/400) x 100 = 20%

After expansion Sales Variable cost Contribution Less: Fixed Cost Less: Depreciation PBIT Less: Interest PBT Tax @ 40% PAT Working: 1. Sales = 500 * 1.4 = 700 2. Variable cost to selling price in 2009-10 = (350/500)* 100 = 70 700 455 245 42 40 163 30 133 53.2 79.8

Therefore new Variable cost to selling price = 70 5 = 65% 3. Depreciation in 2009-10 = 30 (10% of 300 lakhs) Depreciation after expansion = 10% of (300 + 100) = 40 4. Interest in 2009-10 = 15 (15% of 100) Interest after expansion = 15% of (100 + 100) = 30 ROE = (79.8/400) * 100 = 19.95% ROCE = (163/600) * 100 = 27.17% Asset Turnover ratio = Sales/Total Assets = 700/600 = 1.17 times

Q.12 50% (10000 units) 20,00,000 10,00,000 3,00,000 1,80,000 1,00,000 4,20,000 1,20,000 1,00,000 2,00,000 60% (12000 units) 23,52,000 12,24,000 3,60,000 2,16,000 1,20,000 4,32,000 1,20,000 1,00,000 2,12,000 80% (16000 units) 30,40,000 16,80,000 4,80,000 2,88,000 1,60,000 4,32,000 1,20,000 1,00,000 2,12,000

Sales Less: Variable Cost Materials Labour Factory Overheads Admin Overheads Contribution Less: Fixed cost Factory Overheads Admin Overheads Profit Working:

Units @ 60% = (10000/50%) * 60% = 12,000 Units @ 80% = (10000/50%) * 80% = 16,000 Q.13 Budget In Rs. cr 40 Actual In Rs. cr 34 Variance In Rs. cr 6

Sales

Less: Operating costs Factory costs Marketing costs Freight Admin expenses Operating profits Operating assets Accounts receivable Cash Inventory Fixed assets ROI=Operating profit/Operating assets

21 7 1 3 32 8 8 4 18 20 50 16%

17 3 .8 3.2 24 10 8.5 2 21.5 20 52 19.23%

4 4 .2 -.2 8 -2 -.5 2 -3.5 0 -2 -3.23

(b) Budgeted ROI was 16% whereas actual ROI for the first quarter 1s 19.23% which is higher. Thus, the performance of the division is satisfactory. The second quarter budget is favorable because the sales are budgeted less and cost is also less as compared to the first quarter. However. The amount of fixed assets is rs. 2 crores as against Rs 20 crores for fixed assets. The second revision may be for sales of Rs 36vcrores which may be higher and should be reduced to some extent because all other costs are also reduced. Moreover budgeted sales for the first quarter were Rs 40 crores and actual sales realized were only Rs 34 crores.

MAY 2009 Q1 Stages in Management control process The control process involves carefully collecting information about a system, process, person, or group of people in order to make necessary decisions about each. Managers set up control systems that consist of four key steps: 1. Establish standards to measure performance. Within an organization's

overall strategic plan, managers define goals for organizational departments in specific, operational terms that include standards of performance to compare with organizational activities. 2. Measure actual performance. Most organizations prepare formal reports of performance measurements that managers review regularly. These measurements should be related to the standards set in the first step of the control process. For example, if sales growth is a target, the organization should have a means of gathering and reporting sales data. 3. Compare performance with the standards. This step compares actual activities to performance standards. When managers read computer reports or walk through their plants, they identify whether actual performance meets, exceeds, or falls short of standards. Typically, performance reports simplify such comparison by placing the performance standards for the reporting period alongside the actual performance for the same period and by computing the variancethat is, the difference between each actual amount and the associated standard. 4. Take corrective actions. When performance deviates from standards, managers must determine what changes, if any, are necessary and how to apply them. In the productivity and quality-centered environment, workers and managers are often empowered to evaluate their own work. After the evaluator determines the cause or causes of deviation, he or she can take the fourth stepcorrective action. The most effective course may be prescribed by policies or may be best left up to employees' judgment and initiative. These steps must be repeated periodically until the organizational goal is achieved.

Q 2What is a responsibility centre? List and explain different types of Responsibility Centres with sketches. Responsibility centres: A responsibility centre is an organization unit that is headed by a manager who is responsible for its activities. In a sense, a company is a collection of responsibility centres. Each of which is represented by box on the on the organization are responsibility centres for section work shifts or other small organization units. At a higher level are departments or business units that consist of several of these smaller units plus staff and management people these larger units are also responsibility centre. And from the stand point of senior management and the board of directors, the whole company is responsibility centre although the term is usually used to refer to unit within the company. Types of Responsibility Centres Cost Centre Cost centers are divisions that add to the cost of the organization, but only indirectly add to the profit of the company. Typical examples include Research and Development, Marketing and Customer service. Companies may choose to classify business units as cost centers, profit centers, or investment centers. Profit Centre A responsibility centre is called a profit centre when the manager is held responsible for both costs (inputs) and revenues (outputs) and thus for profit. Despite the name, a profit centre can exist in nonprofits organizations (though it might not be referred to as such) when a responsibility centre receives revenues for its services. A profit centre is a big segment of activity for which both revenues and costs are accumulated:

Investment Centre An investment centre goes a step further than a profit centre does. Its success is measured not only by its income but also by relating that income to its invested capital, as in a ratio of

income to the value of the capital employed. In practice, the term investment centre is not widely used. Instead, the term profit centre is used indiscriminately to describe centres that are always assigned responsibility for revenues and expenses, but may or may not be assigned responsibility for the capital investment. It is defined as a responsibility centre in which inputs are measured in terms of cost / expenses and outputs are measured in terms of revenues and in which assets employed are also measured. Nature of responsibility centres A responsibility centre exist one or more purpose are its objectives. The company as a whole has goals, and senior management has decided on a set of strategies to accomplish these goals. The objectives of responsibility centres are to help implement these strategies Q3) Every SBU is a profit center but every profit center is not a SBU? What are the conditions that should be fulfill for an organization unit to be converted into a profit center? What are the different ways to measure the performance of profit center? Discuss their relevant merits and demerits. Conditions for an organization to be converted into a profit centre: Many management decisions involve proposals to increase expenses with the expectation of an even greater increase in sales revenue. Such decisions are said to involve expense/revenue trade-offs. Additional advertising expense is an example. Before it is safe to delegate such a trade-off decision to a lower-level manager, two conditions should exist. The manager should have access to the relevant information needed for making such a decision. There should be some way to measure the effectiveness of the trade-offs the manager has made. A major step in creating profit centres is to determine the lowest point in an organization where these two conditions prevail. All responsibility centres fit into a continuum ranging from those that clearly should be profit centres to those that clearly should not. Management must decide whether the advantages of giving profit responsibility offset the disadvantages, which are discussed below. As with all management control system design choices, there is no clear line of demarcation.

Ways to Measure Performance: There are two types of profitability measurements used in evaluating a profit centre, just as there are in evaluating an organization as a whole. First, there is the measure of management performance, which focuses on how well the manager is doing. This measure is used for planning, coordinating, and controlling the profit center's day-to-day activities and as a device for providing the proper motivation for its manager. Second, there is the measure of economic performance, which focuses on how well the profit centre is doing as an economic entity. The messages conveyed by these two measures may be quite different from each other. For example, the management performance report for a branch store may show that the store's manager is doing an excellent job under the circumstances, while the economic performance report may indicate that because of economic and competitive conditions in its area the store is a losing proposition and should be closed. . The necessary information for both purposes usually cannot be obtained from a single set of data. Because the management report is used frequently, while the economic report is prepared only on those occasions when economic decisions must be made, considerations relating to management performance measurement have first priority in systems design-that is, the system should be designed to measure management performance routinely, with economic information being derived from these performance reports as well as from other sources. Types of Profitability Measures A profit center's economic performance is always measured by net income (i.e., the income remaining after all costs, including a fair share of the corporate overhead, have been allocated to the profit centre). The performance of the profit centre manager, however, may be evaluated by five different measures of profitability: (1) contribution margin, (2) direct profit, (3) controllable profit, (4) income before income taxes, or (5) net income (1) Contribution Margin: Contribution margin reflects the spread between revenue and variable expenses. The principal argument in favour of using it to measure the performance of profit centre managers is that since fixed expenses are beyond their control, managers should focus their attention on maximizing contribution. The problem with this argument is that its premises are inaccurate; in fact, almost all fixed expenses are at least partially controllable by the manager, and some are

entirely controllable. Many expense items are discretionary; that is, they can be changed at the discretion of the profit centre manager. Presumably, senior management wants the profit centre to keep these discretionary expenses in line with amounts agreed on in the budget formulation process. A focus on the contribution margin tends to direct attention away from this responsibility. Further, even if an expense, such as administrative salaries, cannot be changed in the short run, the profit centre manager is still responsible for controlling employees' efficiency and productivity. (2) Direct Profit: This measure reflects a profit center's contribution to the general overhead and profit of the corporation. It incorporates all expenses either incurred by or directly traceable to the profit centre, regardless of whether or not these items are within the profit centre manager's control. Expenses incurred at headquarters, however, are not included in this calculation. A weakness of the direct profit measure is that it does not recognize the motivational benefit of charging headquarters costs. (3) Controllable Profit: Headquarters expenses can be divided into two categories: controllable and non controllable. The former category includes expenses that are controllable, at least to a degree, by the business unit manager-information technology services, for example. If these costs are included in the measurement system, profit will be what remains after the deduction of all expenses that may be influenced by the profit centre manager. A major disadvantage of this measure is that because it excludes non controllable headquarters expenses it cannot be directly compared with either published data or trade association data reporting the profits of other companies in the industry. (4) Income before Taxes: In this measure, all corporate overhead is allocated to profit centres based on the relative amount of expense each profit centre incurs. There are two arguments against such allocations. First, since the costs incurred by corporate staff departments such as finance, accounting, and human resource management are not controllable by profit centre managers, these managers should not be held accountable for them. Second, it may be difficult to allocate corporate staff

services in a manner that would properly reflect the amount of costs incurred by each profit centre. There are, however, three arguments in favour of incorporating a portion of corporate overhead into the profit centres' performance reports. First, corporate service units have a tendency to increase their power base and to enhance their own excellence without regard to their effect on the company as a whole. Allocating corporate overhead costs to profit centres increases the likelihood that profit centre manager will question these costs, thus serving to keep head office spending in check. (Some companies have actually been known to sell their corporate jets because of complaints from profit centre managers about the cost of these expensive items.) Second, the performance of each profit centre will become more realistic and more readily comparable to the performance of competitors who pay for similar services. Finally, when managers know that their respective centres will not show a profit unless all-costs, including the allocated share of corporate overhead, are recovered, they are motivated to make optimum long-term marketing decisions as to pricing, product mix, and so forth, that will ultimately benefit (and even ensure the viability of) the company as a whole. If profit centres are to be charged for a portion of corporate overhead, this item should be calculated on the basis of budgeted, rather than actual, costs, in which case the "budget" and "actual" columns in the profit center's performance report will show identical amounts for this particular item. This ensures that profit centre managers will not complain about either the arbitrariness of the allocation or their lack of control over these costs, since their performance reports will show no variance in the overhead allocation. Instead, such variances would appear in the reports of the responsibility centre that actually incurred these costs. . (5) Net Income: Here, companies measure the performance of domestic profit centres according to the bottom line, the amount of net income after income tax. There are two principal arguments against using this measure: (1) after tax income is often a constant percentage of the pretax income, in which case there would be no advantage in incorporating income taxes, and (2) since many of the decisions that affect income taxes are made at headquarters, it is not appropriate to judge profit centre managers on the consequences of these decisions. There are situations, however, in which the effective income tax rate does vary among profit centres. For example, foreign subsidiaries or business units with foreign operations may have different effective income tax

rates. In other cases, profit centres may influence income taxes through their instalment credit policies, their decisions on acquiring or disposing of equipment, and their use of other generally accepted accounting procedures to distinguish gross income from taxable income. In these situations, it may be desirable to allocate income tax expenses to profit centres not only to measure their economic profitability but also to motivate managers to minimize tax liability. Merits:

The quality of decisions may improve because they are being made by managers closest to the point of decision. The speed of operating decisions may be increased since they do not have to be referred to corporate headquarters. . Headquarters management, relieved of day-to-day decision making, can concentrate on broader issues.

Managers, subject to fewer corporate restraints, are freer to use their imagination and initiative. Because profit centres are similar to independent companies, they provide an excellent training ground for general management. Their managers gain experience in managing all functional areas, and upper management gains the opportunity to evaluate their potential for higher-level jobs.

Profit consciousness is enhanced since managers who are responsible' for profits will constantly seek ways to increase them. (A manager responsible for marketing activities, for example, will tend to authorize promotion expenditures that increase sales, whereas a manager responsible for profits will be motivated to make promotion expenditures that increase profits.).

Profit centres provide top management with ready-made information on the profitability of the company's individual components. . Because their output is so readily measured, profit centres are particularly responsive to pressures to improve their competitive performance.

Demerits: Decentralized decision making will force top management to rely more on management control reports than on personal knowledge of an operation, entailing some loss of control.

If headquarters management is more capable or better informed than the average profit centre manager, the quality of decisions made at the unit level may be reduced. Friction may increase because of arguments over the appropriate transfer price, the assignment of common costs, and the credit for revenues that were formerly generated jointly by two or more business units working together. Organization units that once cooperated as functional units may now be in competition with one another. An increase in profits for one manager may mean a decrease for another. In such situations, a manager may fail to refer sales leads to another business unit better qualified to pursue them; may hoard personnel or equipment that, from the overall company standpoint, would be better off used in another unit; or may make production decisions that have undesirable cost consequences for other units. Divisionalization may impose additional costs because of the additional management, staff personnel, and record keeping required, and may lead to task redundancies at each profit centre. Q.4 (a) Transfer Pricing is not an accounting tool comment with an illustration If a group has subsidiaries that operate in different countries with different tax rates, manipulating the transfer prices between the subsidiaries can scale down the overall tax bill of the group. For example the tax rate in Country A is 20% and is 50% in Country B. In the larger interest of the group, it would be advisable to show lower profits in Country B and higher profits in Country A. For this, the group can adjust the transfer price in such a way that the profits in Country An increase and that in Country B get reduced. For this the group should fix a very high transfer price if the Division in Country A provides goods to the Division in Country B. This will maximize the profits in Country A and minimize the profits in Country B. The reverse will be true if the Division in Country A acquires goods from the Division in Country B. Q.4.( b) Market Price is ideal transfer price even in limited markets. Comments

By limited market it means that the markets for buying and selling profit centres may be limited. Even in case of limited market the transfer price that is ideal or satisfies the requirement of a profit centre system is the competitive price. In case if a company is not buying or selling its product in an outside market there are some ways to find the competitive price. They are as follows: 1. If published market prices are available, they can be used to establish transfer prices. However, these should be prices actually paid in the market-place and the conditions that exist in the outside market should be consistent with those existing within the company. For example, market prices that are applicable to relatively small purchases are not valid in this case. 2.Market prices are set by bids. This generally can be done only if the low bidder has a reasonable chance of obtaining the business. One company accomplishes this by buying about one-half of a particular group of products outside the company and one-half inside the company 3.If the production profit centre sells similar products in outside markets, it is often possible to replicate a competitive price on the basis of the outside price. 4.If the buying profit centre purchases similar products from the outside market, it may be possible to replicate competitive prices for its proprietary products. Q 5 Sum done in class Q6) Enumerate the differences among the following types of audits: a) Financial Audit (Statutory) b) Cost Audit c) Efficiency Audit d) Management Audit A6) Pg 243 onwards in the book a) Financial Audit:

Financial Audit is a historically oriented, independent evaluation performed by internal auditor or external auditor for the purpose of attesting to the fairness, accuracy and reliability of the financial data, providing protection for the entity's assets; evaluating the adequacy and accomplishment of the system (internal control) designed, provide for the aforementioned Fairness and Protection, Financial data, while not being the only source of evidence, are the primary evidential source. The evaluation is performed on a planned basis rather than a request". Financial audit takes care of the protective aspect of the business and it does not normally carry out constructive appraisal function of the business operations. It helps in detection and prevention of fraud. It also verifies whether documentation and flow of activities arc in conformity with the internal control system introduced and developed within the organization. It helps coordinating with statutory auditor to help them in proper discharge of their function. Besides, financial audit also ensures compliance with statutory laws especially in financial and accounting matters. Objectives of Financial Audit-To see that established accounting systems and procedures have been complied with -To see that proper records have been maintained for the fixed assets of the Concern to look into correctness of the financial data and records along with correctness of the accounting procedure followed. -To see whether scrap, salvage and surplus materials have been properly accounted for etc. -To see that internal control system has been working properly. -To see that any abrupt variation in sales, purchases etc.; with respect to immediate previous year are not due to any irregularity -To see that the credit control has been strictly followed. -To see that all payments have been made with proper authorization and approval. . -To see that preparation of salary and wage pay roll has been properly done. The opinion expressed by the auditors shall be based on verified data, reference to which shall also be made here and, if practicable, included after the company has been forded on opportunity to comment on them. b) Cost Audit:

Cost Audit is defined as the verification of correctness of cost records and check on the adherence to the cost accounting plan. It is an audit process for the verification of the cost of manufacture or production of any article on the basis of accounts relating to utilization of material, labour and other items of cost maintained by the company with the accepted principles of cost accounting. Cost Audit is an audit of efficiency. Objectives of Cost Audit: -Verification of Cost Accounts with a view to ascertain that those have been properly maintained and compiled according to the cost accounting system. - Ensure that the prescribed procedure of cost accounting is duly adhered to. - Detection of errors and frauds. - Determination of inventory valuation - Facilitating the fixation of prices and goods or services - Periodical reconciliation between cost accounts and financial accounts - Ensuring optimal utilization of human, physical and financial resources of the company. - Detection and correction of abnormal losses - Inculcation of cost consciousness - Advising management as regards the areas where performance calls for improvement. c) Efficiency Audit: Efficiency Audit ensures application of the basic economic principles i.e. resources flow into the most remunerative channels. The main purpose of Efficiency audit is to ensure that: (a) Every rupee invested in capital or in other resources gives the optimum returns, and (b) The planning of investment between the different functions and aspects is designed to give optimum results. From the above point of view, cost audit can be appropriately called as Efficiency Audit. Efficiency audit is thus defined as systematic analysis of activities to assess the efficiency with which resources are utilized. The purpose of efficiency audit is to control rising costs and inflation. By adherence to efficiency, audit norms will enable the company to sell its products in the competitive

market. It can even suggest ways and means for the execution of plans and projects with greater expedition, efficiency and economy. d) Management Audit: It is the Comprehensive examination of an enterprise to appraise its organisational structure, policies and procedures in order to determine whether sound management exists at all levels, ensuring effective relationship with the outside world and internal efficiency. It is concerned with review of the past performance to ascertain whether it is in tune with the objective, policies and procedures. It is the method to evaluate the efficiency of the management at all levels throughout the organization. Hence, it can also be called as Efficiency Audit. There is no limitation as to the period to be covered for management audit. There is no legal compulsion either, as compared to the others, where it is present. Therefore the company can appoint any person to undertake this task.

Objectives of Management Audit: -

Whether the basic aims and objectives of the enterprise are being fulfilled in practice. Whether the company is being successful in adopting itself to technological change. Whether management is efficient at all levels and the extent to which economies are possible. Whether the management structure is suitable Whether the policies with regard to staff recruitment and training are adequate Whether there is a proper communication system both upwards and downwards throughout the enterprise. Whether ROCE is adequate and its comparison with other companies in the same industry. Whether the companys share of the market is increasing and comparison with its main competitor. Whether its relationships with the outside world are effective and whether its corporate image is satisfactory.

Q7) Organizations with Business Divisions (Profit Centre) format have observed that Divisional Controllers experience divided loyalty in carrying out their functions, causing a possible dysfunction. How could such a situation be resolved? Define role of controller which suits your suggestion.

A7) To the extent the decision are decentralized top management may lose some control. Relying on control reports is not as effective as personal knowledge of an operation. With profit centre, top management must change its approach to control. Instead of personal direction senior management must rely to a considerable extent on management control reports. Competent units that were once cooperating as functional units may now compete with one another disadvantageously. An increase in one managers profit may decrease those of another. This decrease in cooperation may manifest itself in a manager unwillingness to refer sales lead to another business unit, even though that unit is better qualified to follow up on the lead in production decision that have undesirable cost consequence on other units or in the hoarding of personnel or equipment that from the overall company standpoint would be better off used in another units. There may be too much emphasis on short run profitability at the expense of long run profitability. In the desire to report high current profits, the profit centre manager may skip on R&D, training, maintenance. This tendency is especially prevalent when the turnover of profit centre managers is relatively high. In these circumstances, manager may have good reason to believe that their action may not affect profitability until after they have moved to other job. There is no complete satisfactory system for ensuring that each profit centre by optimizing its own profit will optimize company profits. If headquarter management is more capable or has better information then the average profit centre manager the quality of some of the decision may be reduced. Divisionalization may cause additional cost because it may require additional management staff personnel and recordkeeping and may lead to redundant at each profit centre. Business units as profit centres:

Business units are usually set up at profit centres. Business unit managers tend to control product development, manufacturing, and marketing resources. They are in a position to influence revenue and cost and as such can be held accountable for the bottom line. However as pointed out in the next section a business unit manager authority may be constrained such constrained should be incorporated in designing and operating profit centre. Constraint on business unit authority To realize fully the advantage of the profit centre concept the business unit manger would have to be as autonomous as the president of the independent company. As a practical matter however such autonomy is not feasible. If a company were divided into completely independent units the organization would be giving up the advantage of size and synergism. Also senior management authority that a board of director gives to the chief executive. Consequently business unit structure represents trade off between business unit autonomy and corporate constraint. The effectiveness of a business units organization is largely dependent on how well these trade off are made. The performance of a profit centre is appraised by comparing actual results for one or more orf these measures with budgeting amounts. In addition, data on competitors and the industry provide a good cross check on the appropriate of the budget. Data for individual companies are available from the securities and exchange commission for about key business ratios; standard & poor computer services, Inc; Robert Morris associates annual statement studies; and annual survey published in fortune, business week, and Forbes. Trade associations publish data for the companies in their industries. Revenues: choosing the appropriate revenue recognition method is important. Should revenue be recognized at the time as order is received, at the time an order is shipped, or at the time cash is received? In addition to that decision, issues related to common revenues may need to be considered. There are some situations in which two or more profit centres participate in the sales effort that results in a sale; ideally, each should be given appropriate credit for its part in this transaction. Many companies have not given much attention to the solution of these common revenue problems. They take the position that the identification of price responsibility for revenue generation is too complicated to be practical and that sale personnel must recognize they are working not only for their own

profit centre but also for the overall good of the company. They for example, may credit the business unit that takes an order for a product handled by the another unit with the equivalent of a brokerage commission or a finder fee. In the case of a bank the branch performing a service may be given explicit credit for that service even though the customer account is maintained in another branch. Role of controller It should publish procedure and forms for the preparation of the budget. It should provide assistance to budgetees in the preparation of their budget. It should administer the process of making budget revision during the year. It should coordinate the work of budget departments in lower echelons It should analyze reported performance against budget, interprets the result, and prepares summary report for senior management. (Taken from the link sent by Sheldon) Q8) What do you understand by Goal Congruence? What are the informal factors that influence goal congruence? A8) Goal congruence means that the goals of an organizations individual members should be consistent with the goals of the organization itself. According to Chris Argyris, goal congruence in a process means that actions it leads people to take in accordance with their perceived self-interest are also in the best interest of the organization. Informal factors that influence goal congruence Work ethic Work ethic is a set of attitudes which are manifested in employees loyalty to the organization, their diligence spirit and pride in doing a good job. Some of these attitudes are local or specific to the city or region in which the organization does its work. Some attitudes and norms are industry-specific. India and China have a reputation for excellent work ethics. Within India, the work ethic in Western India is better than the work ethic in Eastern India. Work culture The common beliefs, shared values, norms of behaviour and assumptions that are implicitly accepted and explicitly manifested throughout the organization are known as work culture. Organization culture is a very important factor which influences human behaviour in an informal manner. The culture of an organization is ever-lasting.

Managers may come and go but the culture continues forever. The Chief Executive Officer and managers working under him have a major influence on the culture of an organization. An organizations culture has a major influence on the design of management control systems. Organizations may have really good formal management control systems but the one which has the desired culture is in a position to exercise better control. Cultural norms are very important because they explain why two organizations, with identical formal management control systems, vary in terms of actual control. A companys culture usually exists unchanged for many years. Organizational culture is also influenced strongly by the personality and policies of the CEO and middle managers with respect to the areas they control. If there is a union in the organization, the rules and norms accepted by the union also have a major influence on the organizations culture. Management style Management style is the attitude of a managers superior towards control. This gets reflected to some extent in the attitude of the subordinates. Management style has the most influence on management control. An institution is the lengthened shadow of a man. Managers come in all shapes and sizes. Some managers are charismatic and outgoing, while others are less enthusiastic. Some of them spend much time looking and talking to people while others rely more heavily on written reports. Perception and communication Managers are informed about the goals of the organization and the actions they are expected to take in order to achieve these goals through a number of channels. These channels may be formal or informal. Formal means of communication include the corporate plan, budgets and other documents. The informal channels include conversation, discussion etc. An organization is a complicated entity and the actions that should be taken by any one part to further the common goals cannot be stated with absolute clarity even in the best of circumstances. Moreover, the messages received from different sources may conflict with one another or may be subject to different interpretations. The information operating managers receive as to what they are supposed to do is vastly less clear than the information they actually receive.

The informal organization The official authority-responsibility relationship of each manager is shown in the line chart of an organization. However, in an informal organization, there is no clear-cut authority-responsibility relationship. The finance managers interaction with the managers in the marketing or production department constitutes the informal organization. It is important in enabling the managers to comprehend the realities underlying the process of management control. Thus, the realities of the management control process cannot be understood without recognizing the importance of the relationships that constitute the informal organization. Cooperation and conflict The top management is primarily concerned with the responsibility of achieving the goals of the organization. The management makes decisions from time to time and the same is communicated to responsibility centre managers lower down the hierarchy for implementation. Responsibility centre managers have their personal goals. The interactions among managers have an effect on the manner in which the plans are executed. There may be a strained relationship between the production departments and service departments. The circulars carrying instructions to responsibility centre managers have adverse reactions on the middle managers. These factors affect the personal needs of the managers. Thus, there are conflicts between the managers in the organization. Those conflicts are part and parcel of organizational life. Cooperation is opposite of conflict. Many a times we find that there is effective cooperation between the people in the organization and the responsibility centre managers. They have to achieve the organizations goals. The cooperation may arise out of personal relations or mutual independence. There should be a proper balance between cooperation and conflicts. Conflicts resulting from competition among managers for increments or promotions are healthy but there should be some limit. On the other hand, there should be some amount of cooperation for the smooth functioning of an organization. Q9) Write Short Notes on any 2 of the following: i) Zero Based Budgeting ii) Free Cash Flow iii) MCS in the Matrix Organization

A9) (i) Zero Based Budgeting: Zero-based budgeting is a technique of planning and decision-making which reverses the working process of traditional budgeting. In traditional incremental budgeting, departmental managers justify only increases over the previous year budget and what has been already spent is automatically sanctioned. No reference is made to the previous level of expenditure. By contrast, in zero-based budgeting, every department function is reviewed comprehensively and all expenditures must be approved, rather than only increases.[1] Zero-based budgeting requires the budget request be justified in complete detail by each division manager starting from the zero-base. The zero-base is indifferent to whether the total budget is increasing or decreasing. The term "zero-based budgeting" is sometimes used in personal finance to describe the practice of budgeting every dollar of income received, and then adjusting some part of the budget downward for every other part that needs to be adjusted upward. It would be more technically correct to refer to this practice as "active-balanced budgeting". ii) Free Cash Flow: Pg. 27 in the book A companys value depends on its free cash flow (FCF) which is defined as FCF= Net op profit after tax Net Inv in Op. Cap

If a company can reduce its inventories, its cash holding or even its receivables, then its net inv in op profit will go down. If these actions do not harm op profit then free cash flows will increase, which will lead to a higher stock price. Cash conversion Cycle: Companies typically follow a cycle in which they purchase inventory, sell goods on credit and then collect accounts receivables. This cycle is called the Cash conversion cycle. Inventory conversion period: Average Time required to convert materials into finished goods and then to sell those goods.

Receivables collection period: Average time required to convery the cos. Receivables into cash that is to collect cash following a sale. Also called Days Sales Outstanding (DSO).

Payables Deferred period: Avg lth of time between the purchase of materials and payment of cash for them. Cash Conversion cycle: Avg lth of time a rupee is tied up in current assets.

iii) MCS in the matrix organisation: Pg 66 & 107 in the book The matrix organization is a combination or merger of the functional and divisional structures. Product lines are arranged along one arm of the matrix, across the other arm could be arranged either functions or geographical divisions. Features of a Matrix Organization: Built around a specific project Project mgr draws personnel from various functional departments Project and Functional mgrs play different roles Personnel assigned to project have 2 bosses.

A matrix form allows new businesses to plug into existing functional resources. Q10) Pritam Engineering manufactures a variety of metal products at many factories. Currently, it is experiencing a crisis. Management has therefore decided to install detailed expense control system including responsibility budgets for overhead expense items at each factory. From historical data, Controller developed a standard for each overhead expense item (relating expense to vol of activity). Summarised expenses for Nov 2005 given to concerned Production Supervisor for comments is tabulated. All fig are in Rs. 000.

Item

Standard at Normal Value (1)

Budgeted at Actual Volume (2) 720

Actual

Management Supervision Indirect Labour Idle time Materials, Tools Maintenance, Scrap Allocated expenses Total per ton (Rs.)

720

582

12706 420 3600 14840 21040 2133.04

11322 361 3096 13909 21040 2103.39

12552 711 3114 17329 21218 2413.30

a) Explain with full justification which of the 2 standards (1) or (2) is more meaningful for expense control. b) Can the supervisor be held responsible for all overhead expenses included? Why or why not?

Q11)
D ivision A B C D E Profit Fixed Assets C urrent Assets R T = A /T C of C O A P T A ost apita E A l V 30 0 80 0 10 6 3 .2 % 1 5 9 7 .1 % 22 1 20 2 40 0 10 60 1 .0 % 1 0 6 0 .0 % 10 0 10 0 60 0 10 00 6 5 .2 % 6 8 .8 % -1 .0 0 0 10 1 40 0 80 0 9 7 .1 % 6 7 .6 % 3 0 10 8 20 0 80 0 1 .0 % 8 0 6 0 .0 % 10 2

Based on Return on assets Division A has performed the best followed by E, B, D and C. The divisions rank in the same order when evaluated based on the EVA as well. However, it is to be noted that Division C though showing a return of 6.25% has a negative EVA and the company could review its investment decision.

Question 12 Refer Solution on Page 151 of Text Book Question 13 Refer Page 201 of text book

MAY 2008 Q1 a Internal Audit is one of the techniques of control available to the top management for ensuring continuous development for better performance. Comment upon this statement Ans: Internal auditing may be called as a special segment of the broad field of accounting. The techniques and methods of auditing are the same as in external auditing. However, the main concern of an internal auditor is to ensure that there is proper compliance with policies, rules and procedures if the enterprise such as good business practices, generally accepted accounting principles, laws of the land and government regulations. Internal audit helps an organisation accomplish its objectives by bringing a systematic, disciplined approach to evaluate and improve effectiveness of risk management, control and governance process. It is an independent, objective assurance consulting activity designed to add value and improve an organizations operations. Internal audit helps in the following ways to ensure continuous development for better performance: By studying and evaluating the adequacy and effectiveness of accounting, financial and operating controls. By ascertaining the extent to which business assets are accounted for and safeguarded from losses. By evaluating the quality of performance in carrying out assigned responsibilities. By ascertaining the degree of compliance with predetermined policies, plans and procedures. By ascertaining the authenticity of accounting and other data compiled within the organisation. By furnishing the members of management with the objective analysis, comments and recommendations as regards activities of the business so as to help them in efficient and reflective discharge of their responsibilities.

Q1b What are the considerations involved in regulating R&D function by the top management especially in view of challenges faced on account of globalisation? Ans: Research and development is an important function in modern organisation. Liberalisation and globalisation have thrown up a number of opportunities and challenges. This has increased the importance of R&D activities.

Type of financial control: The financial control exercised in a discretionary expense centre is quite different from that in engineered centre the latter attempts to minimize operating cost by setting a standard and reporting actual costs against this standards. The main purpose of a discretionary expense budget on the other hand is to allow the manager to control Cost for particular in the planning. Costs are controlled primarily by deciding what task should be undertaken and what level of effort is appropriate for each. Thus in a discretionary expense centre financial control is primary exercised at the planning stage before the amount are incurred.

Measurement of performance: The primary job of the manager of a discretionary expense centre is to accomplish the desired output spending an amount that is on budget is satisfactory. This is in contrast with the report in an engineered expense centre which helps higher management to evaluate the manger efficiency. If these two types of responsibility centre are carefully distinguished management may treat the performance report for the discretionary expense centre as if it were an indication of efficiency Control over spending can be exercised by requiring that the manger approved be obtain before the budget is over sometimes a certain percentage of overrun is permitted without additional approval if the budget really set forth the best estimate of actual cost there is 50 percent probability that it will overrun and this is the reason that some latitude is often permitted.

Control problems: The control of R & D centres, which are also discretionary expense centre is difficult for the following at least a semi tangible output reasons.

The considerations involved in regulating R&D functions:

Unrealistic expectations: The head of R&D centre generally feels inclined to have an excellent department which may be beyond the companys means.

Lack of business insight: It is also found that the persons engaged in R&D occasionally do not have interests in the business. They may have inadequate knowledge of the business in order to provide direction to research.

Difficultly in measuring output: It is difficult to measure the output of R&D centres as the output are semi-intangible in the form of patents, new product, new designs, processes, etc.

Input-output relationship: It is also difficult to establish relationship between the inputs and outputs of the R&D activities. A product may take several years of efforts to evolve and as a result the inputs provided for in the annual budget may bear no relations to output.

May not be possible to control annually: As research efforts take several years to bear fruit, it is difficult to control R&D in an effective manner on an annual basis.

Manpower: The main element of expenditure is manpower cost and getting highly skilled personnel is difficult.

The goal congruence problem in R&D centre is similar to that in administrative centres. The research managers typically want to build the best research organization that money can buy, even though this is more expensive than the company can afford. A further problem is that research people often may not have sufficient knowledge of the business to determine the optimum direction of the research efforts.

Research and development can seldom be controlled effectively on an annual basis. A research project may take year s to reach fruition, and the organization must be built up slowly over a long time period. The principal cost is for the work force obtaining highly skilled scientific talented is often difficult, and short term fluctuation in the work force are in efficient. It is not reasonable, therefore to reduce R&D costs in years when profits are low and increase them in year when profits are high. R&D should be looked at as a long term investment not as an activity that varies with short run corporate profitability.

The R&D continuum: Activities conducted by R&D organization lie along a continuum. At one extreme is basic research; the other extreme is product testing. Basic research has two characteristics: first, it is unplanned management at most can specify the general area that is to be explored second there is often a very long time lag before basic research result in successful new product introductions. Financial control system has little value in managing basic research activities. In some companies, basic research in included as a lump sum in the research program and budget. In others, no specific allowance is made for basic research as such; there is an understanding that scientists and engineers can devote part of their time to explorations in whatever direction they find most interesting, subject only to informal agreement with their supervisor. For product testing projects, on the other hand, the time and financial requirement can be estimated, not as accurately as production activities. Q 2a Answer was not available in any of the library books or online Q2 b What are the challenges faced in pricing corporate services provided to Business Units operating as profit centres? Business Units as Profit Centres Most business units are created as profit centres since managers in charge of such units typically control product development, manufacturing, and marketing resources. These managers are in a position to influence revenues and costs and as such can be held accountable for the "bottom line." However, as pointed out in the next section, a business unit manager's authority may be constrained in various ways, which ought to be reflected in a profit centres design and operation. Constraints on Business Unit Authority To realize fully the benefits of the profit centre concept, the business unit manager would have to be as autonomous as the president of an independent company. As a practical matter, however, such autonomy is not feasible. If a company were divided into completely independent units, the organization would lose the advantages of size and synergy. Furthermore in delegating to business unit management all the authority that the board of directors has given to the CEO, senior management would be abdicating its own responsibility. Consequently, business unit structures represent trade-offs between business unit autonomy and corporate constraints. The effectiveness of a business unit organization is largely dependent on how well these trade-offs are made.

Constraints from Other Business Units. One of the main problems occurs when business units must deal with one another. It is useful to think of managing a profit centre in terms of control over three types of decisions: (1) The product decision (what goods or services to make and sell), (2) The marketing decision (how, where, and for how much are these goods or services to be sold?), and (3) The procurement or sourcing decision (how to obtain or manufacture the goods or services). If a business unit manager controls all three activities, there is usually no difficulty in assigning profit responsibility and measuring performance. In general, the greater the degree of integration within a company, the more difficult it becomes to assign responsibility to a single profit centre for all three activities in a given product line; that is, if the production, procurement, and marketing decisions for a single product line are split among two or more business units, separating the contribution of each business unit to the overall success of the product line may be difficult. Constraints from Corporate Management The constraints imposed by corporate management can be grouped into three types: (1) Those resulting from strategic considerations, (2) Those resulting because uniformity is required, and (3) Those resulting from the economies of centralization. Most companies retain certain decisions, especially financial decisions, at the corporate level, at least for domestic activities. Consequently, one of the major constraints on business units results from corporate control over new investments. Business units must compete with one another for a share of the available funds. Thus, a business unit could find its expansion plans thwarted because another unit has convinced senior management that it has a more attractive program. Corporate management also imposes other constraints. Each business unit has a "charter" that specifies the marketing and/or production activities that it is permitted to undertake, and it must refrain from operating beyond its charter, even though it sees profit opportunities in

doing so. Also, the maintenance of the proper corporate image may require constraints on the quality of products or on public relations activities. Companies impose some constraints on business units because of the necessity for Uniformity. One-constraint is that business Units must conform to corporate accounting and MCS This constraint is especially troublesome for units that have been acquired from another company and that have been accustomed to using different systems.

Q 3a) What is two step transfer pricing and profit sharing approach? Narrate Merits and demerits Transfer pricing: If two or more profit centers are jointly responsible for product development manufacturing and marketing, each should share in the revenue that is generated when the product is finally sold. The transfer price is not primarily an accounting tool; rather, it is a behavioral tool that motivates manager to make the right decisions. In particular the transfer price should be designed so that it accomplishes the following objective: It should provide each segment with the relevant information required to determine the optimum tradeoff between company cost and revenues It should induce goal congruent decisions that is the system should be so designed that decision improve business unit to earn more profit It should help measure the economic performance of the individual profit center Two step pricing: First, a charge is made for each unit sold that is equal to the standard variable cost of production. Second a periodic charge is made for the buying unit. One or both of these components should include a profit margin. The two step pricing method correct this problem by transferring variable cost on a per unit basis, and transferring fixed cost and profit on a lump sum basis under this method the transfer price for product A would be 5$ for each unit that unit Y purchases plus $20000 per month for fixed cost. Plus $10000 per month for profit: if transfer of product A in a certain month are at the expected amount 5000 units then under the two step method unit y will pay the variable cost of $25000 plus $30000 for the fixed cost and profit a total of $55000 .this is the same amount as the amount it would pay unit x if the transfer price is less than 5000 units say 4000unoits.unit y would pay $50000 under the two step methods compared with the $44000 it would pay if the transfer price

were $11 per unit. The difference is their transfer prices were for not using a portion of unit X capacity that it has reserved. Note that fewer than two step methods the company variable cost for product A is identifiable to unit Y variable cost for the product, and unit Y will make the correct short term marketing decisions. Unit Y also has information on upstream fixed costs and profit related to product A and it can use these data for long term decision. The fixed cost calculation in the two step pricing method is based on the capacity that is reserved for the production of product A that is sold to unit Y the investment represented by this capacity is allocated to product A. The return on investment that unit X earns on competitive product is calculated and multiplied by the investment assigned to the product. In the example we calculated the profit allowance as a fixed monthly amount. It would be appropriate under some circumstance to divide the investment into variable and fixed component. Then, a profit allowance based on a return on investment on variable assets would be added to the standard variable cost for each unit sold.

Profit sharing: If the two step pricing system just described is not feasible, a profit sharing system might be used to ensure congruence of business unit interest with company interest. This system operates somewhat as follows. 1. The product is transferred to the marketing unit at standard variable cost. 2. After the product is sold, the business units share the contribution earned which is selling price minus the variable manufacturing and marketing costs. This method of pricing may be appropriate if the demand for the manufactured product is not steady enough to warrant the permanent assignment of facilities as in the two step method. In general, this method accomplished the purpose of making the marketing units interest congruent with the companies. There are several practical problems in implementing such profit sharing system. First, there can be arguments over the way contribution is divided between the two profit centres. Which is costly, time consuming and work against basic reason for decentralization namely autonomy of the business units mangers. Second, arbitrarily divided up the profit between units does not give valid information on the profitability of each segment of the organization.

Third since the contribution is not allocated until after the sale has been made the manufacturing units contribution depends upon the marketing units ability to sell and on the actual selling price. Manufacturing units may perceive this situation to be unfair

Q3b) "Adopting profit centre approach may not be an appropriate solution always". Do you agree? Give reasons for answer quoting various situations in the business. A profit centre is a unit of a company that generates revenue in excess of its expenses. It is expected that, through the sale of goods or services, the unit will turn a profit. This is in contrast to a cost centre, which is a unit inside a company that generates expenses with no responsibility for creating revenue. The only expectation a cost centre has is to lower expenses whenever possible while staying with a specific budget that is determined at the corporate level. Beyond that simple definition, the term "profit centre" has also come to represent a form of management accounting that is organized around the profit centre concept. Companies that have adopted the profit centre system have organized all of their business units as either profit Centerior cost centres, and all company financial results are reported in that manner. Adopting a profit enter system often requires a radical shift in corporate philosophy and culture, but it can yield great returns in net before tax (NBT) profits. According to an article in Business Solutions, The data collection company Data Recognition, Inc. made the shift to a profit centre-based system and was pleased with the results. The profit centres allows bettering identifying specific gains and losses. And that's critically important for a growing business. All companies, no matter what size, have both cost and profit For example, in most companies, units such as human resources and purchasing are strictly cost centres. The company has to spend money to operate those units, and neither has any means of producing a profit to offset those expenses. They exist solely to make it possible for other areas of the company to make money. However, without those two departments, the company could not survive. All companies have profit centres and cost centres, but not all companies organize their accounting practices around the profit centre concept. In fact, most companies do things the time-honoured way, producing overall profit and loss statements for the company as a whole, without making each business unit accountable for generating a profit. A cost centre may actually provide services that could generate a profit if they were offered on the open market. But in most corporate environments, cost centres are not expected

to generate a profit and operation costs are treated as overhead. Departments that are typically cost centres include information technology, human resources, accounting, and others. However, the satisfied acceptance that some departments will always be cost centres and can never generate a profit has changed at some companies. They recognize that cost centres can turn into profit centres by taking the services they used to automatically provide to the company's other business units and making those services available for a fee. In a business, every cost is supposed to have an ROI, and the cost is incurred only when the ROI promises to be more than the investment itself .As an example of how a cost centre may be turned into a profit center,Consider a company's information technology (IT) department. This department may provide such services as computer-aided design, network administration, or database development toothier units of the company. These services have value, and they are important to the companys overall success, but they do not generate a profit. IT may charge the "cost" of its services back t0 the department that requested them, but it does not make a profit because it charges only for its actual costs incurred, without adding an extra margin for profit. The unit that requested the services absorbs the cost as part of its overhead; or, in some companies, the cost is not charged back and is simply part of the company's overall overhead. There are two ways that the IT department could make the switch from cost centre to profit centre. First, instead of writing off its services to overhead or charging them at cost, the IT department could be allowed to bill other departments for its services at going market rates. The second way the IT department could become a profit centre is if the company determined that the department was one of the best in the industry, better in fact than some companies that existed just to provide IT services. The company could then allow the department the freedom to sell its services to outside customers. Thus, the department would still operate as a cost centre in its dealings with other units inside the company, but it would operate as a profit centre when it provided services to outside companies. Just as the company's senior management could decide that the IT department was good enough to operate as a profit centre by soliciting outside clients, so too could it decide that the department is behind the times and is not providing adequate services. This would result in management choosing to shut down the department and contract with an outside vendor for the companys IT needs. Considering that HR is a crucially important function but alas, is usually bogged down by routine things like

recruitment etc., and not in value-addition activities like training and development. When this change occurs, HR can be an indirect profit centre through the following actions: 1. Savings through: a. Negotiation during recruitment: Trying to get the best for the least; b. Training: For example healthcare training to save on health-related downtime and absentism Medical bills etc.; reducing wastages through job-oriented hard skills training etc. Reducing attrition, thereby cost of recruitment. 2. Raised productivity through value addition such as Hard Skills & Soft Skills training for augmenting proper job-related skills, motivation, engagement and focus etc. Thus If both the department exists only as a cost centre, it faces enormous pressure to provide services at the lowest possible costs. Because it does not generate profits, it must constantly fight to remain in existence and must fight off attempts to slash its budget to free up cash for the companys profit centres. Q.4)What is Balance Scorecard? Describe steps involved in implementation thereof, difficulties and reasons for failure thereof, if any The Balanced Scorecard (BSC) is a performance management tool which began as a concept for measuring whether the smaller-scale operational activities of a company are aligned with its larger-scale objectives in terms of vision and strategy. By focusing not only on financial outcomes but also on the operational, marketing and developmental inputs to these, the Balanced Scorecard helps provide a more comprehensive view of a business, which in turn helps organizations act in their best long-term interests. Organizations were encouraged to measurein addition to financial outputswhat influenced such financial outputs. For example, process performance, market share / penetration, long term learning and skills development, and so on. The underlying rationale is that organizations cannot directly influence financial outcomes, as these are "lag" measures, and that the use of financial measures alone to inform the strategic control of the firm is unwise. Organizations should instead also measure those areas where direct management intervention is possible. In so doing, the early versions of the Balanced Scorecard helped organizations achieve a degree of "balance" in selection of performance measures. In practice, early Scorecards achieved this balance by encouraging managers to

select measures from three additional categories or perspectives: "Customer," "Internal Business Processes" and "Learning and Growth." The balance scorecard suggests that we view the organization from four perspectives, and to develop metrics, collect data and analyze it relative to each of these perspectives: The learning and growth perspective: To achieve our vision, how will we sustain our ability to change and improve? The business process perspective : To satisfy our shareholders and customers what business processes must we excel at? The customer perspective : To achieve our vision, how should we appear to our customer? The financial perspective : To succeed financially, how should we appear to our shareholders? Implementing a Balanced Scorecard We can summarize the implantation of a balanced scorecard in four general steps; 1. Define strategy. 2. Define measure of strategy. 3. Integrate measures into the management system. 4. Review measures and result frequently. Each of these steps is iterative, requiring the participation of senior executive and employees throughout the organization 1 Define Strategy The balance scorecard builds a link between strategy and operational action. As a result it is necessary to begin the process of defining a balanced scorecard by defining the organization goals are explicit and what that targets have been developed. 2 Define Measures of Strategy The next step is to develop measures in support of the articulate strategy. It is imperative that the organization focuses on a few critical measures at this point; otherwise management will be overloaded with measures. Also, it is important that the individual measures be linked with each other in a cause effect manner 3 Integrated Measures into the management system

The balanced scorecard must be integrated with the organization formal and informal structure, its culture, and its human resources practice. While the balanced Scorecard gives some means for balancing measures, the measures can still become unbalanced by others system in the organization such as compensation policies that compensate the manager strictly based on financial performance. 4 Review Measures and result Frequently Once the balance scorecard is up and running it must be consistently reviewed by senior management. The organization should be looking for the following How do the outcome measures say the organization is doing? How do the driver measures say the organization is doing? How has the organizations strategy changed since the last review? How has the scorecard measures changed?

The most important aspects of these reviews are as follows; They tell management whether the strategy is being implemented correctly and They show that management is serious about the importance of these measures. They maintain alignment of measure to ever changing strategies.

how successfully the strategy is working.

Difficulties in implementing Balanced Scorecard The following problems unless suitably dealt with, could limit the usefulness of the balanced scorecard approach: Poor correlation between no financial measures and result. Fixation on financial result. No mechanism for improvement. No mechanism for improvement. Measures overload. Poor Correlation between No financial measures and result Simply put there is no guarantee that future profitably will allow targets achievement in any no financial area. This is probably the biggest problem with the balanced scorecard because there is an inherent assumption that future profitability does follow from achieving the scorecard

measures, identifying the cause effect relationships among the different measures is easier said than done. This will be a problem with any system that is trying to develop proxy measures for future performance. While this does not mean that the balanced Scorecard should be abandoned it is imp that comp adopting such a system understand that the links between no financial measures and financial performance are still poorly understood. Fixation on Financial Results As previously discussed not only are most senior managers well trained and very adept with financial measures but they also most keenly feel pressure regarding the financial performance of their comp. Shareholder are vocal and the board of directors often applies pressure on the stakeholders behalf .this pressure often overwhelms the long term uncertain payback of the no financial measures. Non mechanism for Improvement One of the most overlooked pitfalls of the balanced scorecard is that a company cannot achieve Stretch goals if the Company has no mechanism for improvement .Unfortunately achieving many of these goals require complete shifts in the way that business is done yet the company often does not have mechanism to make those shifts . The mechanism available takes additional resource and requires a changed in the company culture. These changes do not happen overnight nor do they respond automatically to a new stretch targets. Inertia often works against the company employees are accustomed to a self limited cycle of setting targets, missing those targets and readjusting the targets to reflect what was actually achieved. Without a method for making improvement, improvements are unlikely to consistently happen no matter how good the stretch goal sound. Measurement overload How many critical measures can one manager track at one time without losing? Unfortunately there is no right answer to this question except it is more than 1 and less than 50. It too few then the manager is ignoring measures that are critical to creating success. If it too many then the manager may risk losing focus and trying to do too many things at once.

Q5. Economic Value Added(EVA) is a technique of Management Control, considered by some as superior to that of ROI. Analyse the statement and give your comments quoting illustrations prevailing in the Business World. Text book Pg168 Q6 does not exist in this paper..Neil Q7 Explain Responsibility Centre and map the process of evaluation thereof from one stage to another with the help of illustrations and experiences Responsibility centres: A responsibility centre is an organization unit that is headed by a manager who is responsible for its activities. In a sense, a company is a collection of responsibility centres. Each of which is represented by box on the on the organization are responsibility centres for section work shifts or other small organization units. At a higher level are departments or business units that consist of several of these smaller units plus staff and management people these larger units are also responsibility centre. And from the stand point of senior management and the board of directors, the whole company is responsibility centre although the term is usually used to refer to unit within the company. Types of Responsibility Centres Cost Centre. Profit Centre Investment Centre Revenue Centres Contribution centres Budget centres Got the latter part from the bog
http://mcs20112010.blogspot.in/2011_04_01_archive.html Neil

Process of evaluation of Responsibility Centre.

1. The organization is divided into various responsibility centres. Each responsibility centre is put under the charge of a responsibility manager. 2. The targets or budgets of each responsibility centre are set in consultation with the manager of responsibility centre, so that he may be able to give full information about his department. The manager of responsibility centre should know as what is expected of him - each centre should have a clear set of goals. The responsibility and authority of each centre should be well defined. 3. Managers are charged with the items and responsibility, over which they can exercise a significant degree of direct control. 4. Goals defined for each area of responsibility should be attainable with efficient and effective performance. 5. The actual performance is communicated to the managers concerned. If it falls short of the standards, the variances are conveyed to the top management. The names of persons responsible for the variances are also conveyed so that responsibility may be fixed. The purpose of all these steps is to assign responsibility to different individuals so that their performance is improved and costs are controlled. The personal factor in Responsibility Accounting is most important. The management may prepare the best plan or the budget and put up before its staff, but its success depends upon the initiative and the will of the workers to execute it Example of Responsibility Centre The Sarva Shiksha Abhiyan emphasizes quality improvement in elementary education for which it deems necessary that resource groups and responsibility centres from national to subdistrict levels are identified. These groups would oversee the policy, planning, implementation and monitoring of all quality related interventions. Their major role would be to advise and assist at various levels in curriculum development, pedagogical improvement, teacher education/training and activities related to classroom transaction. In order to facilitate a decentralized mode of education, these groups would need to be constituted at various operational levels, namely - national, state, district and sub district. The following could be involved in the groups: National level - NCERT, NIEPA, Universities, NGOs, experts and eminent educationists.

State level - SCERT, SIEMAT, Universities, IASEs/CTEs, NGOs, experts and eminent educationists. District level - DIETs, representatives from DPEP District Resource Group, higher educational institutions, innovative teachers from the districts, NGOs. Sub-district - BRC/BEO, representatives from CRCs, innovative teachers.

Q.8. Draw a plan of an ideal management control system in a corporate engaged in any of the segments of services sector stating the segment specifically (Im putting down points. You can apply them to any sector) Factors that impact management control 1. Goals 2. Budget 3. Professionals (labour intensive) 4. Output/Input measurement 5. Financial Report analysis 6. Size 7. Managing Marketing 8. Managing performance appraisals 9. Inventory 10. Quality of Service 11. Key performance reports Q9. Strategy Formulation/Task Control/Management Control/

(All differences and similarities can be picked up from the following tables) Characteristics a) Focus of plan b) Complexities c) Nature of information Strategy Formulation On one aspect at a time Many variables hence complex Tailor-made for the issue, more external and d) Structure predictive, less accurate. Unstructured and irregular, each problem being different e) Communication of information Relatively simple f) Purpose of estimates Show expected results g) Persons involved Staff and top management h) No. of persons involved Small i) Mental activity Creative, analytical j) Planning and control k) Time horizon l) End result m) Appraisal of job done Planning dominant but some control Tends to be long Policies and precedents Extremely difficult Management Control On entire organisation Less complex Integrated, more internal and historical, more accurate. Rhythmic, definite pattern, set procedure Relatively difficult Lead to desired result Line and top management Large Administrative, persuasive Emphasis on both planning and control Tends to be short Action within policies laid Less difficult

Characteristics a) Focus of plan b) Nature of information

Task Control Single task or transaction Tailor-made to operation, specific, often non- financial, real time

Management control On entire organisation Integrated, more internal and historical, more accurate Line and top management Administrative, persuasive

c) Persons involved d) Mental activity

Supervisors Follow directives or none as in case of machines or set objectives

e) Time horizon

Day to day

Tends to be short

f) Type of cost

Engineered- Existence of objective standard against which actual can be compared makes control easier.

Discretionary- Control is more difficult due to subjective consideration.

Q. 10 Manejay Ltd: Changes needed in the control system, practices, and procedures are as follows: Sometimes, excessive diversification and that too in unrelated lines of business causes failure in the business operations. One of the major reason for failures of many Mergers and Diversification is excessive diversification. As, excessive diversification is ominous especially, in unrelated lines of business. As, there may be no advantage of operating synergy. Neither through: I) Sharing common resources nor II) Sharing common core competencies Therefore, it may be a strategic decision by the promoters and directors of the company to sell one of its divisions. As, this may be impacting their core business. Sometimes, your core business tends to get neglected mainly due to excessive diversification. As, the division is being sold to its own company managers. There, might not be major changes in management control and systems. As, most of its managers will be the same. But, they will have more autonomy to take decisions independently after acquisition. Now there will be less red tapes and managers can take more risk. The managers will manage the firm in their own style. As, they are not answerable to their superiors. Currently, they are answerable to their stake-holders. As, the management is completely in their hands and that too with full autonomy. The management might have identified the flaws in the previous controls and systems of the company because of which the company might not be so effective and efficient. As, they have been associated with the company over a period of time. They have a better understanding about the business dynamics and environment in which the firm operates. So, they can take necessary steps to overcome the flaws and improve the management control and systems.

So, that is why there will be some significant changes in the management control and systems and procedures if there is further scope for improvement.

Q11 is a numerical solved it in class For part 2 of Q.10 and both parts of Q12. (Basically for any of the answers that are about one company taking over another, or diversifying, or going from closed to professionally managed etc. certain management control changes are required. Depending on the differences (manufacturing/service, national/international, closely-held/professional) and the situation (who is taking over whom), choose the relevant ones that are likely to change from this list of factors.

1. Strategy 2. Budgeting 3. Operations 4. Reporting 5. Pricing 6. Government Regulations 7. Responsibility Centres 8. Taxation 9. Management Style 10. Internal and External controls 11. Communication Process 12. Organizational Culture 13. Financial Statement Accounting

14. International Transactions 15. Markets and Industries that the company belongs to

MAY 2007 1) What do you understand by goal congruence? What are the informal factors that influence goal congruence?

Solution:

Goal congruence means that the goals of an organizations individual members should be consistent with the goals of the organization itself. According to Chris Argyris, goal congruence in a process means that actions it leads people to take in accordance with their perceived self-interest are also in the best interest of the organization.

Informal factors that influence goal congruence

Work ethic Work ethic is a set of attitudes which are manifested in employees loyalty to the organization, their diligence spirit and pride in doing a good job. Some of these attitudes are local or specific to the city or region in which the organization does its work. Some attitudes and norms are industry-specific. India and China have a reputation for excellent work ethics. Within India, the work ethic in Western India is better than the work ethic in Eastern India.

Work culture The common beliefs, shared values, norms of behaviour and assumptions that are implicitly accepted and explicitly manifested throughout the organization are known as work culture. Organization culture is a very important factor which influences human behaviour in an informal manner. The culture of an organization is ever-lasting. Managers may come and go but the culture continues forever. The Chief Executive Officer and managers working under him have a major influence on the culture of an organization. An organizations culture has a major influence on the design of management control systems. Organizations may have really good formal management control systems but the one which has the desired culture is in a position to exercise better control. Cultural norms are very important because they explain why two organizations, with identical formal

management control systems, vary in terms of actual control. A companys culture usually exists unchanged for many years. Organizational culture is also influenced strongly by the personality and policies of the CEO and middle managers with respect to the areas they control. If there is a union in the organization, the rules and norms accepted by the union also have a major influence on the organizations culture.

Management style Management style is the attitude of a managers superior towards control. This gets reflected to some extent in the attitude of the subordinates. Management style has the most influence on management control. An institution is the lengthened shadow of a man. Managers come in all shapes and sizes. Some managers are charismatic and outgoing, while others are less enthusiastic. Some of them spend much time looking and talking to people while others rely more heavily on written reports.

Perception and communication Managers are informed about the goals of the organization and the actions they are expected to take in order to achieve these goals through a number of channels. These channels may be formal or informal. Formal means of communication include the corporate plan, budgets and other documents. The informal channels include conversation, discussion etc. An organization is a complicated entity and the actions that should be taken by any one part to further the common goals cannot be stated with absolute clarity even in the best of circumstances. Moreover, the messages received from different sources may conflict with one another or may be subject to different interpretations. The information operating managers receive as to what they are supposed to do is vastly less clear than the information they actually receive.

The informal organization The official authority-responsibility relationship of each manager is shown in the line chart of an organization. However, in an informal organization, there is no clear-cut authority-responsibility relationship. The finance managers interaction with the managers in the marketing or production department constitutes the informal organization. It is important in enabling the managers to comprehend the realities underlying the process of management control. Thus, the realities of the management control process cannot be understood without

recognizing the importance of the relationships that constitute the informal organization.

Cooperation and conflict The top management is primarily concerned with the responsibility of achieving the goals of the organization. The management makes decisions from time to time and the same is communicated to responsibility centre managers lower down the hierarchy for implementation. Responsibility centre managers have their personal goals. The interactions among managers have an effect on the manner in which the plans are executed. There may be a strained relationship between the production departments and service departments. The circulars carrying instructions to responsibility centre managers have adverse reactions on the middle managers. These factors affect the personal needs of the managers. Thus, there are conflicts between the managers in the organization. Those conflicts are part and parcel of organizational life. Cooperation is opposite of conflict. Many a times we find that there is effective cooperation between the people in the organization and the responsibility centre managers. They have to achieve the organizations goals. The cooperation may arise out of personal relations or mutual independence. There should be a proper balance between cooperation and conflicts. Conflicts resulting from competition among managers for increments or promotions are healthy but there should be some limit. On the other hand, there should be some amount of cooperation for the smooth functioning of an organization.

(Answer taken from pages 60-63 of MCS text book By K.C. Pandey, P.K. Bandgar & S.P. Das)

2) Briefly define Discretionary Expense Centre, Engineered Expense Centre, Profit Centre and Investment Centre? How is budget prepared in Discretionary Expense Centre? How is performance of the manager evaluated in a Discretionary Expense Centre?

Solution:

Discretionary

expense

centre

Discretionary

expense

centres

include

administrative and support units, R&D and marketing activities. For example, accounting legal, industrial relations, public relations and human resources. The output of these centres cannot be measured in monetary terms. The term discretionary does not mean that managements judgement as to optimum cost is capricious or haphazard. In case of discretionary expense centres, an optimal relationship cannot be established between inputs and outputs. The difference between budgeted input and actual input measures the efficiency of discretionary expense centres. This does not measure the value of output. The characteristics of discretionary expense centres are (figure 4.3 on page 79 of text).

Engineered expense centre In an engineered expense centre, an optimal relationship can be established between the inputs and the outputs. Engineered expense centres are generally found in manufacturing operations. Some responsibility centres within administrative and support departments have also engineered expenses. Engineered expense centres have the following characteristics: a) Inputs can be measured in terms of money. b) Outputs can be measured in physical terms. c) Optimum amount of inputs which are required to produce one unit of output can be established. The output of an engineered expense centre gives the standard cost of finished product if it is multiplied by the standard cost of each unit manufactured. The difference between the engineered cost and actual costs represents the efficiency of the organizational unit. There are other important tasks in a responsibility centre which cannot be measured solely on the basis of cost. The effectiveness of these performance aspects should be controlled. The quality is an important aspect of production in addition to volume and cost efficiency. Induction and training are other responsibilities of managers of engineered expense centres.

The characteristics of engineered expense centres are (figure 4.2 on page 79 of text).

Profit centre Profit is the difference between revenue and expenses. A profit centre is an organizational unit in which both revenues and expenses are measured in monetary terms. Profit is a useful performance measure of a responsibility centre. It allows senior management to use one comprehensive indicator rather than several. Thus, the performance in a responsibility centre is measured in terms of revenue it earns and the cost it incurs, it is called a profit centre. In management accounting, revenue is treated as value of the output of the centre whether it is realized or not. The factory may be a profit centre selling its production to the sales department or service department. Profit as a measure of performance is especially useful for the top level management. Therefore, the profit centre concept is a powerful concept. There are two types of profitability measurements used in evaluating a profit centre. There is the measure of management performance which focuses on how well the manager is doing his business operations. It can be used as a device for providing the proper motivation for the managers. There is also a measure of economic performance which focuses on how well the profit centre is doing as an economic activity of the business. These measures are quite different from each other. The characteristics of a profit centre are (figure 4.4 on page 80 of text). Profit measures both effectiveness and efficiency. In case of non-profit making organizations the term financial performance centres is used as an alternative concept.

Investment centre An investment centre is a responsibility centre in which the manager is held responsible for the use of assets as well as for revenues and expenses of the centre. The manager is expected to earn a satisfactory return on capital employed in the business units. Measurement of the investment base or capital employed gives rise to many difficult problems. This is a new idea in the management control system. The characteristics of investment centre are (figure 4.5 on page 80 of text). Investment centres have the measurement problems involved in defining expenses and revenues. These centres also raise some problems regarding the measurement of the assets employed, valuation of fixed assets and current assets and valuation of liabilities. An important goal of a company is to optimize return on shareholders equity. It may not be practical to use such a measure to evaluate the performance of the business unit managers on a regular

basis. For this purpose, accounting rate of return is the best measure of business managers performance. However, economic value added is conceptually superior to return on investment (ROI) in evaluating business unit managers. Reporting on the economic performance of an investment centre is quite different from reporting on the performance of a manager of the investment centre. (Answer to part 1 of the question is taken from pages 78-81 of MCS text book By K.C. Pandey, P.K. Bandgar & S.P. Das. Answers to part 2 & 3 of the question are unclear and need to be checked).

3) Every SBU is a profit centre but every profit centre is not a SBU. What are the conditions that should be fulfilled for an organization unit to be converted into a Profit Centre? What are the different ways to measure the performance of Profit Centres? Discuss their relative merits and demerits.

Solution:

Conditions for an organization to be converted into a profit centre: Many management decisions involve proposals to increase expenses with the expectation of an even greater increase in sales revenue. Such decisions are said to

involve expense/revenue trade-offs. Additional advertising expense is an example. Before it is safe to delegate such a trade-off decision to a lower-level manager, two conditions should exist.

The manager should have access to the relevant information needed for making such a decision. There should be some way to measure the effectiveness of the trade-offs the manager has made.

A major step in creating profit centres is to determine the lowest point in an organization where these two conditions prevail. All responsibility centres fit into a continuum ranging from those that clearly should be profit centres to those that clearly should not. Management must decide whether the advantages of giving profit responsibility offset the disadvantages, which are discussed below. As with all management control system design choices, there is no clear line of demarcation.

Ways to Measure Performance: There are two types of profitability measurements used in evaluating a profit centre, just as there are in evaluating an organization as a whole. First, there is the measure of management performance, which focuses on how well the manager is doing. This measure is used for planning, coordinating, and controlling the profit centres day-today activities and as a device for providing the proper motivation for its manager. Second, there is the measure of economic performance, which focuses on how well the profit centre is doing as an economic entity. The messages conveyed by these two measures may be quite different from each other. For example, the management performance report for a branch store may show that the store's manager is doing an excellent job under the circumstances, while the economic performance report may indicate that because of economic and competitive conditions in its area the store is a losing proposition and should be closed.

The necessary information for both purposes usually cannot be obtained from a single set of data. Because the management report is used frequently, while the economic report is prepared only on those occasions when economic decisions must be made, considerations relating to management performance measurement have first priority in systems design i.e. the system should be designed to measure management performance routinely, with economic information being derived from these performance reports as well as from other sources. Types of Profitability Measures: A profit centres economic performance is always measured by net income (i.e. the income remaining after all costs, including a fair share of the corporate overhead, have been allocated to the profit centre). The performance of the profit centre manager, however, may be evaluated by five different measures of profitability: (1) Contribution margin, (2) Direct profit, (3) Controllable profit, (4) Income before income taxes, or (5) Net income (1) Contribution margin: Contribution margin reflects the spread between revenue and variable expenses. The principal argument in favour of using it to measure the performance of profit centre managers is that since fixed expenses are beyond their control, managers should focus their attention on maximizing contribution. The problem with this argument is that its premises are inaccurate; in fact, almost all fixed expenses are at least partially controllable by the manager, and some are entirely controllable. Many expense items are discretionary; that is, they can be changed at the discretion of the profit centre manager. Presumably, senior management wants the profit centre to keep these discretionary expenses in line with amounts agreed on in the budget formulation process. A focus on the contribution margin tends to direct attention away from this responsibility. Further, even if an expense, such as administrative salaries, cannot be changed in the short run, the profit centre manager is still responsible for controlling employees' efficiency and productivity.

(2) Direct profit:

This measure reflects a profit centres contribution to the general overhead and profit of the corporation. It incorporates all expenses either incurred by or directly traceable to the profit centre, regardless of whether or not these items are within the profit centre manager's control. Expenses incurred at headquarters, however, are not included in this calculation. A weakness of the direct profit measure is that it does not recognize the motivational benefit of charging headquarters costs.

(3) Controllable profit: Headquarters expenses can be divided into two categories: controllable and noncontrollable. The former category includes expenses that are controllable, at least to a degree, by the business unit manager-information technology services, for example. If these costs are included in the measurement system, profit will be what remains after the deduction of all expenses that may be influenced by the profit centre manager. A major disadvantage of this measure is that because it excludes non-controllable headquarters expenses it cannot be directly compared with either published data or trade association data reporting the profits of other companies in the industry.

(4) Income before taxes: In this measure, all corporate overhead is allocated to profit centres based on the relative amount of expense each profit centre incurs. There are two arguments against such allocations. First, since the costs incurred by corporate staff departments such as finance, accounting, and human resource management are not controllable by profit centre managers, these managers should not be held accountable for them. Second, it may be difficult to allocate corporate staff services in a manner that would properly reflect the amount of costs incurred by each profit centre. (5) Net income: Here, companies measure the performance of domestic profit centres according to the bottom line, the amount of net income after income tax. There are two principal arguments against using this measure: (1) after tax income is often a constant percentage of the pre-tax income, in which case there would be no advantage in

incorporating income taxes, and (2) since many of the decisions that affect income taxes are made at headquarters, it is not appropriate to judge profit centre managers on the consequences of these decisions. There are situations, however, in which the effective income tax rate does vary among profit centres. For example, foreign subsidiaries or business units with foreign operations may have different effective income tax rates. In other cases, profit centres may influence income taxes through their instalment credit policies, their decisions on acquiring or disposing of equipment, and their use of other generally accepted accounting procedures to distinguish gross income from taxable income. In these situations, it may be desirable to allocate income tax expenses to profit centres not only to measure their economic profitability but also to motivate managers to minimize tax liability.

Merits: The quality of decisions may improve because they are being made by managers closest to the point of decision. The speed of operating decisions may be increased since they do not have to be referred to corporate headquarters. Headquarters management, relieved of dayto-day decision making, can concentrate on broader issues. Managers, subject to fewer corporate restraints, are freer to use their imagination and initiative. Because profit centres are similar to independent companies, they provide an excellent training ground for general management. Their managers gain experience in managing all functional areas, and upper management gains the opportunity to evaluate their potential for higher-level jobs. Profit consciousness is enhanced since managers who are responsible for profits will constantly seek ways to increase them. Profit centres provide top management with ready-made information on the profitability of the company's individual components. Because their output is so readily measured, profit centres are particularly responsive to pressures to improve their competitive performance.

Demerits: Decentralized decision making will force top management to rely more on management control reports than on personal knowledge of an operation, entailing some loss of control. If headquarters management is more capable or better informed than the average profit centre manager, the quality of decisions made at the unit level may be reduced. Friction may increase because of arguments over the appropriate transfer price, the assignment of common costs, and the credit for revenues that were formerly generated jointly by two or more business units working together. Organization units that once cooperated as functional units may now be in competition with one another. An increase in profits for one manager may mean a decrease for another. In such situations, a manager may fail to refer sales leads to another business unit better qualified to pursue them; may hoard personnel or equipment that, from the overall company standpoint, would be better off used in another unit; or may make production decisions that have undesirable cost consequences for other units. Divisionalization may impose additional costs because of the additional management, staff personnel, and record keeping required, and may lead to task redundancies at each profit centre. (Answer to the above question has been taken from the following link:

http://mcs20112010.blogspot.in/2011_04_01_archive.html).

4) What are the objectives of Transfer Pricing? What is ideal transfer price in the situations of Limited Market Shortage of capacity in the industry

When do you use Cost Based Transfer Prices? Solution: Objectives of transfer pricing To foster a commercial attitude in those who are responsible for the performance of profit centres. The main emphasis should be on profitability. It will force the units to improve their profit position. To optimize the profit of the company over a given period of time. For this purpose the resources should be utilized to the maximum extent. To make optimum use of companys financial resources. It should be based on relative performance of various profit centres, which are influenced by transfer pricing policies. To enable the performance of a division to be evaluated by compensating it for benefits provided for other divisions and changing it for benefits received by the division. To motivate divisional manager for maximizing the profitability of their divisions acting in the best interests of the company as a whole. To manage transfer prices between countries in order to minimize overall tax burden by the international companies/ groups.

(Answer to part 1 of the question is taken from pages 130-131 of MCS text book By K.C. Pandey, P.K. Bandgar & S.P. Das. Answers to part 2 & 3 of the question are unclear and need to be checked. The answers that seem closest to part 2&3 are on pages 143&144 6.6 Procedure for Transfer Pricing which gives ideal transfer price in different situations and on page 131 6.4 Methods of Transfer Pricing (A) Cost-based Transfer Pricing).

Q 5 What are the different methods to evaluate performance of Investment Centres? Discuss the merits and demerits. Which method would you recommend? ROI

Refer pg 166 from Bandgar Advantages 1. Easily Understood 2. Relative measure


3. Further ratios can be worked out {DU Pont Ratio}

4. Inter firm or Intra Firm Valuation can be worked out Disadvantage 1. Confusion between gross and net profit 2. Confusion in capital employee Current or fixed 3. Dissimilar accounting practices make ROI non Comparable 4. Project efficiency may be miscalculated 5. It is a good performance measure but not a good decision making tool EVA refer pg 164 from Bandgar Advantages
1. Opportunity cost of fund is known to the divisional manager. 2. Divisional decisions are compatible with organisational decision.

3. No challenges in decision making. 4. EVA > GDP i.e. wealth acceleration can be compared with economic progress. 5. Enterprise value can be calculated using EVA 6. Withdrawal of activity and diversification decision can be calculated using EVA.
7. Measures managerial and entrepreneurial activity can be easily measured.

Disadvantages

1. 2. 3. 4.
5.

No definite capital cost Assets are not defined i.e. net, gross Controllable and non controllable capital not identified No benchmark for EVA Shareholders contribution not measured

EVA v/s ROI which is better Refer page 168 from Bandgar Q.6) a) What are the Special Characteristics of Professional Service Organization? Answer: A professional organization has relatively few tangible assets; its principal asset is the skill of its professional staff, which 'doesn't appear on its balance sheet. Return on assets employed, therefore, is essentially meaningless in such organizations. Their financial goal is to provide adequate compensation to the professionals. In many organizations, a related goal is to increase their size. In part, this reflects the natural tendency to associate success with large size. In part, it reflects economies of scale in using the efforts of a central personnel staff and units responsible for keeping the organization up-todate. Large public accounting firms need to have enough local offices to enable them to audit clients who have facilities located throughout the world. Professionals Professional organizations are labour intensive, and the labour is of a special type. Many professionals prefer to work independently, rather than as part of a team. Professionals who are also managers tend to work only part time on management activities; senior partners in an accounting firm participate actively in audit engagements; senior partners in law firms have clients. Education for most professions does not include education in management, but quite

naturally stresses the skills of the profession, rather than management; for this and other reasons, professionals tend to look down on managers. Professionals tend to give inadequate weight to the financial implications of their decisions; they want to do the best job they can, regardless of its cost. This attitude affects the attitude of support staffs and non-professionals in the organization; it leads to inadequate cost control.

How is Marketing done in them? In a manufacturing company there is a clear dividing line between marketing activities and production activities; only senior management is concerned with both. Such a clean separation does not exist in most professional organizations. In some, such as law, medicine, and accounting, the profession's ethical code limits the amount and character of overt marketing efforts by professionals (although these restrictions have been relaxed in recent years). Marketing is an essential activity in almost all organizations, however. If it can't be conducted openly, it takes the form of personal contacts, speeches, articles, conversations on the golf course, and so on. These marketing activities are conducted by professionals, usually by professionals who spend much of their time in production work-that is, working for clients. In this situation, it is difficult to assign appropriate credit to the person responsible for "selling" a new customer. In a consulting firm, for example, a new engagement may result from a conversation between a member of the firm and an acquaintance in a company, or from the reputation of one of the professionals as an outgrowth of speeches or articles. Moreover, the professional who is responsible for obtaining the engagement may not be personally involved in carrying it out. Until fairly recently, these marketing contributions were rewarded subjectively-that is, they were taken into account in promotion and compensation decisions. Some organizations now give explicit credit, perhaps as a percentage of the project's revenue, if the person who "sold" the project can be identified.

Q6.b) What is a Non - Profit Organization? How is the performance of this organization evaluated?

Answer: Introduction A non-profit organization, as defined by law, is an organization that cannot distribute assets or income to, or for the benefit of, its members, officers, or directors. The organization can, of course, compensate its employees, including officers and members, for services rendered and for goods supplied. This definition does not prohibit an organization from earning a profit; it prohibits only the distribution of profits. A non-profit organization needs to earn a modest profit, on average, to provide funds for working capital and for possible rainy days. Performance evaluation of non-profit organization For any organization, the most important reasons to measure performance are to improve effectiveness and to acquire information that will allow the organization to drive its agenda forward. If the motivation for doing evaluation remains outside an organization, the evaluation will have limited impact. To do performance assessment effectively, an organization must commit to adopting a culture of measurement, because acceptance must come from senior management, staff, funders, and board members alike. Board self-evaluation Members of the Board of Directors should regularly evaluate the quality of their activities on a regular basis. Activities might include staffing the Board with new members, developing the members into well-trained and resourced members, discussing and debating topics to make wise decisions, and supervising the CEO. Probably the biggest problem with Board selfevaluation is that it does not occur frequently enough. As a result, Board members have no clear impression of how they are performing as members of a governing Board. Poor Board operations, when undetected, can adversely affect the entire organization. Staff and volunteer (individual) performance evaluation Most of us are familiar with employee performance appraisals, which evaluate the quality of an individuals performance in their position in the organization. Ideally, those appraisals reference the individuals written job description and performance goals to assess the quality of the individuals progress toward achieving the desired results described in those documents. Continued problems in individual performance often are the results of poor strategic planning,

program planning and staff development. If overall planning is not done effectively, individuals can experience continued frustration, stress and low morale, resulting in their poor overall performance. Experienced leaders have learned that continued problems in performance are not always the result of a poor work ethic the recurring problems may be the result of larger, more systemic problems in the organizations. Program evaluation Program evaluations have become much more common, particularly because donors demand them to ensure that their investments are making a difference in their communities. Program evaluations are typically focused on the quality of the programs process, goals or outcomes. An ineffective program evaluation process often is the result of poor program planning programs should be designed so they can be evaluated. It can also be the result of improper training about evaluation. Sometimes, leaders do not realize that they have the responsibility to verify to the public that the non-profit is indeed making a positive impact in the community. When program evaluations are not performed well, or at all, there is little feedback to the strategic and program planning activities. When strategic and program planning are done poorly, the entire organization is adversely effected. Evaluation of cross-functional processes Cross-functional processes are those that span several systems, such as programs, functions and projects. Common examples of major processes include information technology systems and quality management of services. Because these cross-functional processes span so many areas of the organization, problems in these processes can be the result of any type of ineffective planning, development and operating activities. Organizational evaluation Ongoing evaluation of the entire organization is a major responsibility of all leaders in the organization. Leaders sometimes do not recognize the ongoing activities of management to actually include organizational evaluations but they do. The activities of organizational evaluation occur every day. However, those evaluations usually are not done systematically. As a result, useful evaluation information is not provided to the strategic and program planning processes. Consequently, both processes can be ineffective because they do not focus on improving the quality of operations in the workplace.

Q7) What do you understand by balance score card? Why Balance Score Card is considered superior to other methods of Performance Appraisal? Prepare Balance Score Card for any organization you are familiar with. ANSWER: What is the Balanced Scorecard? The rationale for the development of the Balanced Scorecard was a growing dissatisfaction with traditional, financial measures of performance. These measures suffer from a number of serious drawbacks in that they take a short-term, lagged (i.e., historic) view of performance. The shift towards flexible, lean production/service systems in many firms has strengthened the requirement for performance measurement systems to become more broadly based, incorporating both non-financial and external measures of performance. According to Kaplan and Norton, the Balanced Scorecard provides a better assessment of performance as it "enables companies to track financial results while simultaneously monitoring progress in building the capabilities and acquiring the intangible assets they need for future growth". The original scorecard designed by Kaplan and Norton contained four key groupings of performance measures. These four groupings, called perspectives by Kaplan and Norton were considered sufficient to track the key drivers of both current and future financial performance of the firm. The perspectives focused on the achievements of the firm in four areas: namely the financial, customer, internal business process and innovation/learning perspectives. The four perspectives can be represented as an interlinked hierarchy. The firms strategy underlies the whole scorecard, as the measures for each of the four perspectives are drawn from this strategy.

To obtain a satisfactory overview of performance, the scorecard will require a mix of lagging and leading (forward looking) measures. Financial measures tend to be lagged and consequently, the measures chosen for the other perspectives will need to include leading measures. In general, outcome measures tend to be lagged, for example, current market share is the result of past decisions and consequently is a lagging measure. Thus the challenge in designing a Balanced Scorecard is to choose driver measures which lead changes in the outcome measures in the non-financial perspectives and which ultimately drive the financial measures. Once the firms objectives have been agreed and the appropriate outcome and driver measures chosen for each of the perspectives, firm and managerial performance is assessed by comparing actual attainment on each measure with the target set for that measure. Objective Measure Target Actual

Benefits from adopting the Balanced Scorecard There are several benefits from implementing a Balanced Scorecard. Originally the Balanced Scorecard was seen as a useful tool for performance measurement. In this role, the Balanced Scorecard was seen as integrating financial/non-financial, internal/external and leading /lagging information on firm performance in a coherent fashion. Later it was realised that the Balanced Scorecard could play a pivotal role in the strategic management process. Because the Balanced Scorecard requires management to clarify and obtain consensus on the strategic objectives of the firm, it can assist in the communication of the chosen strategy, consequently aligning the efforts both of individuals and of departments. In this role, there is a clear link between the Balanced Scorecard and management by objectives (MBO). Effective implementation of a Balanced Scorecard project will generally involve the development of a series of hierarchical (cascaded) scorecards. Given the overall corporate scorecard, supporting scorecards can be developed for each department within the firm. Within each department, a scorecard can be developed for each manager (or perhaps even for each individual member of staff) which links the objectives on each perspective for that manager back to the objectives for each perspective outlined in the scorecard for the department and finally, back to the objectives listed in the firms overall scorecard.

The Balanced Scorecard could be used to assist in corporate restructuring. In recent years, many firms have migrated away from a traditional hierarchical structure to a flatter, teambased organisational structure. The Balanced Scorecard can support such changes, as it can help clarify the objectives and the critical success factors for the newly formed teams. Apart from the communication and co-ordination roles of the Balanced Scorecard in strategic implementation, the Balanced Scorecard can be used to link strategy to specific critical success factors in the customer, internal business process and growth/learning perspectives. By setting both short and long-term targets for driver and outcome measures and by comparing actual attainment against target, feedback is obtained on how well the strategy is being implemented and on whether the strategy is working. Building on the Balanced Scorecards use as a strategic management tool, it has been suggested that the Balanced Scorecard can play a role in the investment appraisal process(5). Traditional methods of investment appraisal such as discounted cash flow do not cope well with investments which generate indirect rather than direct financial returns. Examples of these include investments which enhance the future flexibility of a firm or investments in the firms infrastructure, such as an enhanced management information system. The Balanced Scorecard can assist managements investment appraisal decisions as it provides managers with a mechanism to incorporate the strategic aspects of the investment into the appraisal process. This could be achieved by using a weighting system developed from a firms Balanced Scorecard measures to evaluate new projects. An index score would be calculated for each investment opportunity and projects would then be ranked and selected based on this score. Q8. What are different types of Strategic Missions at SBU level? How do these missions affect Strategic Planning process and Budgeting at SBU Level? Different Types of Strategic Missions: Business Unit Mission: In a diversified firm one of the important tasks of senior management is resource deployment, that is, make decisions regarding the use of the cash generated from some business units to finance growth in other business units. Several planning models have been developed to help

corporate level managers of diversified firms to effectively allocate resources. These models suggest that a firm has business units in several categories, identified by their mission; the appropriate strategies for each category differ. Together, the several units make up a portfolio, the components of which differ as to their risk/reward characteristics just as the components of an investment portfolio differ. Both the corporate 'office and the business unit general manager are involved in identifying the missions of individual business units. Of the many planning models, two of the most widely used are Boston Consulting Group's two-by-two growth-share matrix and General Electric Company/McKinsey & Company's three-by-three industry attractiveness-business strength matrix. While these models differ in the methodologies they use to develop the most appropriate missions for the various business units, they have the same set of missions from which to choose: build, hold, harvest, and divest. Build: This mission implies an objective of increased market share, even at the expense of short-term earnings and cash flow (e.g., Merck's bio-technology, Black and Decker's handheld electric tools). Hold: This strategic mission is geared to the protection of the business unit's market share and competitive position (e.g.: IBM's mainframe computers). Harvest: This mission has the objective of maximizing short-term earnings and cash flow, even at the expense of market share (e.g., American Brands' tobacco products, General Electric's and Sylvania's light bulbs) Divest: This mission indicates a decision to withdraw from the business either through a process of slow liquidation or outright sale. While the planning models can aid in the formulation of missions, they are not cook books. A business unit's position on a planning grid should not be the sole basis for deciding its mission. Business Unit Competitive Advantage: Every business unit should develop a competitive advantage in order to accomplish its mission. Three interrelated questions have to be considered in developing the business unit's competitive= advantage. First, what is the structure of the industry in which the business unit operates? Second, how should the business unit exploit the industry's structure? Third, what will be the basis of the business unit's competitive advantage? Industry Analysis: Research has highlighted the important role industry conditions play in the performance of individual firms. Studies have shown that average industry profitability is, by far, the most significant predictor of firm performance. According to Porter, the structure of an industry should be analyzed in terms of the collective strength of five competitive forces.

1. The intensity of rivalry among existing competitors. Factors affecting direct rivalry are industry growth, product differentiability, number and diversity of competitors, level of fixed costs, intermittent overcapacity, and exit barriers. 2. The bargaining power of customers. Factors affecting buyer power are number of buyers, buyer's switching costs, buyer's ability to integrate backward, impact of the business unit's product on buyer's total costs, impact of the business unit's product on buyer's product quality/ performance, and significance of the business unit's volume to buyers. 3. The bargaining power of suppliers. Factors affecting supplier power are number of suppliers, supplier's ability to integrate forward, presence of substitute inputs, and importance of the business unit's volume to suppliers. 4. Threat from substitutes. Factors affecting substitute threat are relative price/performance of substitutes, buyer's switching costs, and buyer's propensity to substitute. 5. The threat of new entry. Factors affecting entry barriers are capital requirements, access to distribution channels, economies of scale, product differentiation, technological complexity of product or process, expected retaliation from existing firms, and government policy. We make three observations with regard to the industry analysis: 1. The more powerful the five forces are, the less profitable an industry is likely to be. In industries where average profitability is high (such as soft drinks and pharmaceuticals), the five forces are weak (e.g., in the soft drink industry, entry barriers are high). In industries where the average profitability is low (such as steel and coal), the five forces are strong (e.g., in the steel industry, threat from substitutes is high). 2. Depending on the relative strength of the five forces, the key strategic issues facing the business unit will differ from one industry to another. 3. Understanding the nature of each force helps the firm to formulate effective strategies. Supplier selection (a strategic issue) is aided by the analysis of the relative power of several supplier groups; the business unit should link with the supplier group for which it has the best competitive advantage. Similarly, analyzing the relative bargaining power of several buyer groups will facilitate selection of target customer segments. Generic Competitive Advantage:

The five-force analysis is the starting point for developing a competitive advantage since it helps to identify the opportunities and threats in the external environment. With this understanding, Porter claims that the business unit has two generic ways of responding to the opportunities in the external environment and developing a sustainable competitive advantage: low cost and differentiation. Low Cost: Cost leadership can be achieved through such approaches as economies of scale in production; experience curve effects, tight cost control, and cost minimization (in such areas as research and development, service, sales force, or advertising). Some firms following this strategy include Charles Schwab in discount brokerage, Wal-Mart in discount retailing, Texas Instruments in consumer electronics, Emerson Electric in electric motors, Hyundai in automobiles, Dell in computers, Black and Decker in machine tools, Nucor in steel, Lincoln Electric in arc welding equipment, and BIC in pens. Differentiation: The primary focus of this strategy is to differentiate the product offering of the business unit, creating something that is perceived by customers as being unique. Approaches to product differentiation include brand loyalty (Coca-Cola and Pepsi Cola in soft drinks), superior customer service (Nordstrom in retailing), dealer network (Caterpillar Tractors in construction equipment), product design and product features (Hewlett-Packard in electronics), and technology (Cisco in communications infrastructure). Other examples of firms following a differentiation strategy include BMW in automobiles; Stouffer's in frozen foods, NeimanMarcus in retailing, Mont Blanc in pens, and Rolex in wristwatches. Value Chain Analysis: Business units can develop competitive advantage based on low cost, differentiation, or both. The most attractive competitive position is to achieve cost-cum-differentiation. Q9.a,b -repeated in set2, c)repeated in set-9, d) Internal Control: Internal control is defined as a process affected by an organization's structure, work and authority flows, people and management information systems, designed to help the

organization accomplish specific goals or objectives.[1] It is a means by which an organization's resources are directed, monitored, and measured. It plays an important role in preventing and detecting fraud and protecting the organization's resources, both physical (e.g., machinery and property) and intangible (e.g., reputation or intellectual property such as trademarks). At the organizational level, internal control objectives relate to the reliability of financial reporting, timely feedback on the achievement of operational or strategic goals, and compliance with laws and regulations. At the specific transaction level, internal control refers to the actions taken to achieve a specific objective (e.g., how to ensure the organization's payments to third parties are for valid services rendered.) Internal control procedures reduce process variation, leading to more predictable outcomes Objective categorization Internal control activities are designed to provide reasonable assurance that particular objectives are achieved, or related progress understood. The specific target used to determine whether a control is operating effectively is called the control objective. Control objectives fall under several detailed categories; in financial auditing, they relate to particular financial statement assertions,[5] but broader frameworks are helpful to also capture operational and compliance aspects: 1. Existence (Validity): Only valid or authorized transactions are processed (i.e., no invalid transactions)
2. Occurrence (Cut-off): Transactions occurred during the correct period or were

processed timely. 3. Completeness: All transactions are processed that should be (i.e., no omissions) 4. Valuation: Transactions are calculated using an appropriate methodology or are computationally accurate. 5. Rights & Obligations: Assets represent the rights of the company, and liabilities its obligations, as of a given date. 6. Presentation & Disclosure (Classification): Components of financial statements (or other reporting) are properly classified (by type or account) and described. 7. Reasonableness-transactions or results appear reasonable relative to other data or trends. Activity categorization

Control activities may also be described by the type or nature of activity. These include (but are not limited to):

Segregation of duties - separating authorization, custody, and record keeping roles to limit risk of fraud or error by one person. Authorization of transactions - review of particular transactions by an appropriate person. Retention of records - maintaining documentation to substantiate transactions. Supervision or monitoring of operations - observation or review of ongoing operational activity. Physical safeguards - usage of cameras, locks, physical barriers, etc. to protect property. Analysis of results, periodic and regular operational reviews, metrics, and other key performance indicators (KPIs). IT Security - usage of passwords, access logs, etc. to ensure access restricted to authorized personnel. S

Q10) Part of a multinational group, Sundaram Shoe Company(SSC), established its own facilities in India over 75 years ago and enjoyed an excellent record-high market share for its diverse range of shoes, growth and profits. SC markets its products through company owned shops and its own personnel. Organization structure is functional. Since 2001, profitability, market share are slipping. Pressure from cheap Chinese shoes and also premium shoes like Nike has made the company think< of organizational restructuring and introducing Comensurate Control System to regain its position. Although SSC outsources, 30% of products, it is seen as a production oriented company. SSC wants to adopt measures to reduce costs, strengthen marketing and be in a position to produce and meet unexpected and unusual customer demands. How should the company reorganize to achieve Goal Congruence. Define Performance Metric? In a goal congruent process, the actions people are led to take in accordance with their perceived self-interest are also in the best interest of the organization. A firms strategy has a major influence on its structure. The type of structure in turn influences the design of the organizations management control systems. Sundaram Shoe Companys (SSC) organization structure is functional which involves the notion of a manager who brings specialized knowledge to bear on decisions related to a specific function, vis--vis a general purpose

manager who lacks the specialized knowledge. A skilled marketing and production manager would be able to make better decisions in their respective fields. He would also be able to supervise workers in the same function better than the generalist would. Thus an important advantage of the functional structure is efficiency. A major disadvantage of this structure is that there is no unambiguous way of determining the effectiveness of the separate functional managers because each function contributes jointly to the organizations final output. Therefore, there is no way of determining how much of the profit was earned respectively by the several production departments. Sundaram Shoe Company which was a market leader for a period of over 75 years has been losing market share, which has impacted its profitability. Also it needs to be seen that the company outsourcers about 30% of its products. The company aims to strengthen marketing, reduce costs and wants to be in a position to customize products as per the demands of the customer. Thus, Sundaram needs to re-organize its organization structure which is functional to a Business Unit form of organization. The benefits of the re-organization would be that the business unit or the division would be responsible for all the functions involved in producing and marketing a specified product line. The business managers act almost as if their units are separate companies. They are responsible for planning and co-coordinating the work of the separate functions. Their performance is measured by the profitability of the business unit. This is a valid criterion because profit reflects the activities of both marketing and production. Though business unit managers exercise broad authority over their units, headquarters reserves certain key prerogatives. Headquarters are responsible for obtaining funds for the company as a whole and allocating it to the business unit, as well as approving budgets and judging the performance of business unit managers, setting their compensation. A major advantage of the Business unit structure of organization is that because it is close to the market for its products than the headquarters, its manager may make sounder production and marketing decisions than headquarters might and the unit as a whole reacts to new threats or opportunities quickly. This re-organization would help in achieving goal congruence in the organization. Performance Metrics are high-level measures what you are doing; that is, they assess your overall performance in the areas you are measuring. They are external in nature and are most closely tied to outputs, customer requirements, and business needs for the process. The performance measurement system should cover the following areas at a minimum:

Customers 1. Performance against customer requirements 2. Customer satisfaction Performance of internal work processes 1. Cycle times 2. Product and service quality 3. Cost performance (could be productivity measures, inventory, etc.) Suppliers 1. Performance of suppliers against your requirements Financial 1. Profitability (could be at the company, product line, or individual level) 2. Market share growth and other standard financial measures Employee 1. Associate satisfaction Q.12 Suresh Ltd. (Numerical)

(a) Define profit in this case and prepare a statement for both divisions and overall company. Solution: i) Profitability statement of Division A:Particulars Amount(Rs.) Selling price p.u. 35 Variable Cost p.u. 11 Contribution p.u. 24 Contribution Expected sales Total Total Fixed Net profit (Rs.) p.u. (no. of units) contribution cost (Rs.) 24 2000 48000 60000 (12000) 24 3000 72000 60000 12000 24 6000 144000 60000 84000

ii) Profitability statement of Division B:Selling Total Contribution Expected Total Total p.u. variable p.u. sales (no. contribution Fixed cost p.u. of units) cost (Rs.) 90 42 48 2000 96000 90000 80 42 38 3000 114000 90000 50 42 8 6000 48000 90000 Net profit (Rs.) 6000 24000 (42000)

[Note: Total Variable cost p.u. = Variable cost p.u. (Rs.7) + Transfer price of intermediate product (Rs.35)] iii) Profitability statement of Company as a whole:Expected sales 2000 3000 6000 Net profit of Net profit of Total Net profit division A (Rs.) Division B (Rs.) (12000) 6000 (6000) 12000 24000 36000 84000 (42000) 42000

(b) State the selling price which maximizes profits for division B and company as a whole. Comment on why the latter price is unlikely to be selected by division B. Solution: As per the calculation in part (a), selling price p.u. of Rs.80 maximizes profit for division B whereas selling price p.u. of Rs.50 maximizes profit for the Company as a whole. However, if Division B opts for selling price p.u. of Rs.50 in order to maximize Companys profit, it would suffer a loss of Rs.42000. Therefore, Division B would not select Selling price p.u. of Rs.50.

MAY 2006 1) Explain briefly the likely features of an IDEAL Management Control System in Organisations. Management control is a process of assuming that resources are obtained and used effectively and efficiently in the accomplishment of the organisational objectives. It is a fundamental necessity for the success of a business and hence from time to time, the current performance of the various operations is compared to a predetermined standard or ideal performance and in case of variance, remedial measures are adopted to confirm operations to a set plan or policy. Features of an ideal MCS include: Total system: MCS is an overall process of the enterprise which aims to fit together the separate plans for various segments to assure that each harmonizes with the others and that the aggregate effort of all of them on the entire enterprise is satisfactory.

Monetary Standard: MCS is built around a financial structure and all the resources and outputs are measured and expressed in terms of money.

Definite pattern: It follows a definite pattern and time table. It is a continuous process.

Coordinated System: It is a fully coordinated and integrated system.

Emphasis: It requires emphasis both on search for planning as well as control.

Function of every manager: Managers at every level have to focus towards future operational and accounting data, taking into consideration past performance, present trends and anticipated economic and technological changes. The nature, scope and level of control will be governed by the level of managers exercising it.

Existence of goals and plans: MCS is not possible without predetermined goals and plans. These two provide a link between future anticipations and actual performance.

Forward Looking: MCS is on the basis of past performance so that the future plans or guidelines can be laid down.

Continuous Process: It is a continuous process over human and material resources. It demands vigilance at every step. Deciding, planning and regulating the activities of people associated in the common task of attaining the objectives of the organization is the primary aim of MCS.

People oriented: It is the managers, engineers and operators who implement the ideas and objectives of the management. Coordination of the main divisions of the organization helps in smooth operations and less friction which results in the achievement of predetermined objectives.

2) What is Balanced Scorecard? Describe the process of implementation and possible difficulties? The Balance Scorecard is a performance Management tool which began as a concept for measuring whether the smaller scale operational activities of a company are aligned with its large scale objectives in terms of vision and strategy.

By focusing not only on financial outcomes but also on operational, marketing and development inputs to these, the Balanced Scorecard helps provide a more comprehensive view of a business, which in turn helps organisations act in their best long term interests. Organisations were encouraged to measure- in addition to financial outputs, what influenced such financial outputs. For eg., process performance, market share/penetration, long term learning and skills development and so on. The underlying rationale is that organisations cannot directly influence financial outcomes as these are lag measures and that the use of financial measures alone to inform the strategic control of the firm is unwise. Organisations should instead also measure those areas where direct management intervention is possible. In doing so, the early versions of the Balanced Scorecard helped organisations achieve a degree of Balance in the selection of performance measures. In practice, early scorecards achieved this balance by encouraging managers to select measures from three additional perspectives: Customer, Internal Business Processes, Innovation and Learning. The Balance Scorecard suggests that we view the organisation from 4 perspectives and to develop metrics, collect data and analyse it relative to each of these perspectives.

Implementation of Balance Scorecard: Consists of four stages. Define the Strategy Measures for achievement of strategy Integrate financial/non financial measures in the control system Review the measures and results. Change the strategy if required.

Difficulties in implementation There exists poor correlation between financial and non financial measures

Managers are obsessed with monetary measures Measurement overload or underload of each parameter is possible and the management may lose focus

Difficult to establish a trade off between financial and non financial measures/parameters

Performance measures must be updated periodically which is not done Quality measures are difficult to establish.

3) Organizations with Business Divisions (Profit Centre) format have observed that Divisional Controllers experience divided loyalty in carrying out their functions, causing a possible dysfunction. How could such a situation be resolved? Define role of controller which suits your suggestion. To the extent the decision are decentralized top management may lose some control. Relying on control reports is not as effective as personal knowledge of an operation. With profit center, top management must change its approach to control. Instead of personal direction senior management must rely to a considerable extent on management control reports. Competent units that were once cooperating as functional units may now compete with one another dis advantageously. An increase in one managers profit may decrease those of another. This decrease in cooperation may manifest itself in a manager unwillingness to refer sales lead to another business unit, even though that unit is better qualified to follow up on the lead in production decision that have undesirable cost consequence on other units or in the hoarding of personnel or equipment that from the overall company standpoint would be better off used in another units. There may be too much emphasis on short run profitability at the expense of long run profitability. In the desire to report high current profits, the profit center manager may skip on R&D, training, maintenance. This tendency is especially prevalent when the

turnover of profit center managers is relatively high. In these circumstances, manager may have good reason to believe that their action may not affect profitability until after they have moved to other job. There is no complete satisfactory system for ensuring that each profit center by optimizing its own profit , will optimize company profits. If headquarter management is more capable or has better information then the average profit center manager the quality of some of the decision may be reduced. Divisionalization may cause additional cost because it may require additional management staff personnel and recordkeeping and may lead to redundant at each profit center. Business units as profit centers: Business units are usually set up at profit centers. Business unit managers tend to control product development, manufacturing, and marketing resources. They are in a position to influence revenue and cost and as such can be held accountable for the bottom line. However as pointed out in the next section a business unit manager authority may be constrained such constrained should be incorporated in designing and operating profit center. Constraint on business unit authority To realize fully the advantage of the profit center concept the business unit manger would have to be as autonomous as the president of the independent company. As a practical matter however such autonomy is not feasible. If a company were divided into completely independent units the organization would be giving up the advantage of size and synergism. Also senior management authority that a board of director gives to the chief executive. Consequently business unit structure represents trade off between business unit autonomy and corporate constraint. The effectiveness of a business units organization is largely dependent on how well these trade off are made. The performance of a profit center is appraised by comparing actual results for one or more orf these measures with budgeting amounts. In addition, data on competitors and the industry provide a good cross check on the appropriate of the budget. Data for individual companies are available from the securities and exchange commission for

about key business ratios; standard & poor computer services, Inc; Robert Morris associates annual statement studies; and annual survey published in fortune, business week, and Forbes. Trade associations publish data for the companies in their industries. Revenues: choosing the appropriate revenue recognition method is important. Should revenue be recognized at the time as order is received, at the time an order is shipped, or at the time cash is received? In addition to that decision, issues related to common revenues may need to be considered. There are some situations in which two or more profit centers participate in the sales effort that results in a sale; ideally, each should be given appropriate credit for its part in this transaction. Many companies have not given much attention to the solution of these common revenue problems. They take the position that the identification of price responsibility for revenue generation is too complicated to be practical and that sale personnel must recognize they are working not only for their own profit center but also for the overall good of the company. They for example, may credit the business unit that takes an order for a product handled by the another unit with the equivalent of a brokerage commission or a finder fee. In the case of a bank the branch performing a service may be given explicit credit for that service even though the customer account is maintained in another branch. Role of controller It should publish procedure and forms for the preparation of the budget. It should provide assistance to budgetees in the preparation of their budget. It should administer the process of making budget revision during the year. It should coordinate the work of budget departments in lower echelons It should analyze reported performance against budget, interprets the result, and prepares summary report for senior management.

Q4) Explain briefly the various stages of management control process citing salient features of each.

Found the answer at the bog: http://mcs20112010.blogspot.in/2011_04_01_archive.html However, not too sure of the same. Q5) What is a responsibility centre? List and explain different types of responsibility centres with sketches. (Taken from Govindarajan and http://mcs20112010.blogspot.in/2011_04_01_archive.html) A responsibility centre is an organization unit that is headed by a manager who is responsible for its activities. In a sense, a company is a collection of responsibility centres. Each of which is represented by box on the on the organization are responsibility centres for section work shifts or other small organization units. At a higher level are departments or business units that consist of several of these smaller units plus staff and management people these larger units are also responsibility centre. And from the stand point of senior management and the board of directors, the whole company is responsibility centre although the term is usually used to refer to unit within the company. Revenue Centre
-

In revenue centre, out (revenue) is measured in monetary terms, but no formal attempt is made to relate input (expense/cost) to output. (If expense was matched with revenue, the unit would be a profit centre).

Typically revenue centres are marketing/sales units that do not have authority to set selling prices and are not charges for the cost of the goods they market. Actual sales or orders booked are measured against budgets or quotas, and the manager is held accountable for the expenses incurred directly within the unit, but the primary measurement is revenue.

Expense Centre
-

Expense centres are responsibility centres whose inputs are measured in monetary terms, but whose outputs are not. There are two general types of expense centres: Engineered and Discretionary.

Engineered Expense Centre: Their input can be measured in monetary term. Their output can be measured in physical terms. The optimum dollar amount of input required

to produce one unit of output can be determined. Usually found in manufacturing operations.
-

Discretionary Expense Centre: The output of these centres cannot be measured in monetary terms. These include administrative and support units (IR, HR, PR, Legal, accounts), research and development operations and most marketing activities.

Profit Centre A responsibility centre is called a profit centre when the manager is held responsible for both costs (inputs) and revenues (outputs) and thus for profit. Despite the name, a profit centre can exist in nonprofits organizations (though it might not be referred to as such) when a responsibility centre receives revenues for its services. A profit centre is a big segment of activity for which both revenues and costs are accumulated: A centre, whose performance is measured in terms of both - the expense it incurs and revenue it earns, is termed as a profit centre. The output of a responsibility centre may either be meant for internal consumption or for outside customers. In the latter case, the revenue is realized when the sales are made. That is, when the output is meant for outsiders, then the revenue will be measured from the price charged from customers. If the output is meant for other responsibility centre, then management takes a decision whether to treat the centre as profit centre or not. In fact, any responsibility centre can be turned into a profit centre by determining a selling price for its outputs. For instance, in case of a process industry, the output of one process may be transferred to another process at a profit by taking into account the market price. Such transfers will give some profit to that responsibility centre. Although such transfers do not increase the Companys assets, they help in management control process.

Investment Centre An investment centre goes a step further than a profit centre does. Its success is measured not only by its income but also by relating that income to its invested capital, as in a ratio of income to the value of the capital employed.

A responsibility centre is called an investment centre, when its manager is responsible for costs and revenues as well as for the investment in assets used by his centre. He is responsible for maintaining a satisfactory return on investment i.e. asset employed in his responsibility centre. The investment centre manager has control over revenues, expenses and the amounts invested in the centres assets.

For diagrams, refer to page 132 Govindarajan (12th Edition) or page 77,78,79,80 Pandey, Bandgar, Das Q6) (a) Explain the concept of Return on Investment (ROI). What are its advantages? (Taken from Pandey, Bandgar, Das) ROI is a percentage of return on the total capital employed in the business. It is also referred to as return on capital employed. It provides an easily calculated and acceptable measure of economic performance of the business. It is calculated as follows: ROI = Operating profit X 100 Capital employed The term capital employed is used in the following way: Capital employed = (shared capital + reserve surplus + long term loans) (fictitious assets and non-business assets) Advantages: It has the following purposes: - to measure the operating performance of an organization - to evaluate and control the capital expenditure projects - to make profit-planning - to analyse the profit by operating divisions - to analyse the profit by product line - to analyse major cost areas in a cost reduction programme

- to determine the relative profitability of different projects Importance of ROI: Page 38-39 of Pandey, Bandgar, Das (b) Many experts regard Economic Value Added (EVA) as a concept superior to ROI and yet in certain cases, EVA does not do justice to the evaluation of investment centres. Explain this phenomenon with illustrations

Q7) Explain various features of Financial, Operational and Management Audit. Illustrate with one example. (Taken from http://mcs20112010.blogspot.in/2011_04_01_archive.html) Financial AuditFinancial Audit is a historically oriented, independent evaluation performed by internal auditor or external auditor for the purpose of attesting to the fairness, accuracy and reliability of the financial data, providing protection for the entity's assets; evaluating the adequacy and accomplishment of the system (internal control) designed, provide for the aforementioned Fairness and Protection, Financial data, while not being the only source of evidence, are the primary evidential source. The evaluation is performed on a planned basis rather than a request". Objectives of Financial Audit: -To see that established accounting systems and procedures have been complied with -To see that proper records have been maintained for the fixed assets of the Concern to look into correctness of the financial data and records along with correctness of the accounting procedure followed. -To see whether scrap, salvage and surplus materials have been properly accounted for etc. -To see that internal control system has been working properly. -To see that any abrupt variation in sales, purchases etc.; with respect to immediate previous year are not due to any irregularity -To see that the credit control has been strictly followed.

-To see that all payments have been made with proper authorization and approval. . -To see that preparation of salary and wage pay roll has been properly done. Management Audit It is a complex task closely related with the process of management. It is highly result oriented. It requires inter/multi-disciplinary approach as it involves examination, review and appraisal of various policies and actions of management on the basis of certain norms/standards. It undertakes comprehensive and critical review of all organizational activities with wider perspective. It goes beyond conventional audit and audits the efficacy of the management itself. Definition: It's a comprehensive and constructive examination of an organization, the structure of a company, institution or branch of government or of any components thereof, such as division or department and its plans, objectives, its means of operations and its use of human and physical facilities. Objectives To ascertain the provision of proper control at different levels, their effectiveness I in accomplishing management goals. Ascertain objectives of the organization are properly communicated and understood at all levels. To reveal defects or irregularities in any of the elements examined and to indicate what improvements are possible to obtain the best results of the operations of the company. To assist the management to achieve the most efficient administration of its operations. To suggest to the management the ways and means to achieve the objectives if the management of the organization itself lacks the knowledge of efficient management. It aims to achieve the efficiency of management and assess the strength and weaknesses of the organization structure, its management team and its corporate culture.

To ascertain the provision of proper control at different levels, their effectiveness in accomplishing management goals.

Ascertain objectives of the organization are properly communicated and understood at all levels.

To reveal defects or irregularities in any of the elements examined and to indicate what improvements are possible to obtain the best results of the operations of the company.

To assist the management to achieve the most efficient administration of its operations. To suggest to the management the ways and means to achieve the objectives if the management of the organization itself lacks the knowledge of efficient management.

Operational Audit Not too sure about where this is. But if it is called Efficiency Audit, find the same on Page 246 Pandey, Bandgar and Da.s Q 8a) Describe the factors which impact service organisation Refer Q 9a May 2001 Q 8b) Explain special characteristics of Professional Services Organizations which would have a bearing on their control systems Refer Q 9b May 2001 Q 9 (a) 1. If two or more profit centres are jointly responsible for product development manufacturing and marketing, each should share the revenue generated when the product is finally sold. 2. Transfer price is a mechanism for distributing this revenue. 3. The transfer price should be designed so that it accomplishes the following objectives: 4. It should provide each business unit with relevant information it needs to determine the optimum tradeoff between company cost and revenue. 5. It should induce goal congruence decisions.

6. The systems should be designed so that the decisions that improve business unit profit also improve company profits. 7. It should help measure the economic performance of the individual business unit.

Q 9 (b) 1. A market price based transfer price will induce goal congruence if the following conditions exist: 1. Competent people 2. Good atmosphere 3. Market price 4. Freedom to source 5. Full information 6. Negotiations 2. The ideal transfer price is based on a well established, normal market price for the identical product being transferred, that is a market price reflecting the same conditions ( quantity quality delivery time) as the product to which the transfer price applies. 3. The market price may be adjusted downwards reflect savings accruing to the selling unit from dealing inside the company. E.g. there would be a bad debt expense, and advt. and selling expense would be smaller when products are transferred from one business unit to the other one within the company. 4. A market price for a similar but not identical product is better than no market price at all. 5. The market price represents the opportunity cost to the seller of selling the product inside.

6. This is so because if the product were not sold inside then it would be sold outside. 7. From a company point of view therefore the relevant cost of product is the market price because hat is the amount of cash that has been forgone by selling inside. 8. The transfer price represents the opportunity cost to the company.
Q10 (A) Particulars Division A Division B Comme

a.)

Return on sales = Profit/Sales * 100

4,00,000/40,00,00 0*100

6,40,000/96,00,00 0*100

10.00% Return on Investments = Profit/ Capital Employed*100 4,00,000/20,00,00 0*100 20% 40,00,000/20,00,0 00 2 times

6.67% 6,40,000/32,00,00 0*100 20% 96,00,000/32,00,0 00 3 times

nt The return on sales of division A is higher than division B. This indicates that Division A has been operatin g well Even though the Return on Capital Employ ed is the same for both the compani es, sales of Division B are higher than that of Division A. Similarl y the capital turnover of Division B is higher than that of Division A. Hence the Division

b.)

c.)

Capital Turnover = Sales/Capital Employed

B has a better marketin g excellen

Q11 Could not solve Question 12 Refer Solution on Page 50 and 51 of Text Book

May 2005 Q 1 Awaited Q2. Discuss the concept of free cash flow and describe the process of its computation. 1. A measure of financial performance calculated as operating cash flow minus capital expenditures. Free cash flow (FCF) represents the cash that a company is able to generate after laying out the money required to maintain or expand its asset base. 2. Free cash flow is important because it allows a company to pursue opportunities that enhance shareholder value. Without cash, it's tough to develop new products, make acquisitions, pay dividends and reduce debt. FCF is calculated as: 3. EBIT(1-Tax Rate) + Depreciation & Amortization - Change in Net Working Capital Capital Expenditure or FCF = Operating Cash Flow - Capital Expenditures 4. The data needed to calculate a company's free cash flow is usually on its cash flow statement. For example, if Company XYZ's cash flow statement reported $15 million of cash from operations and $5 million of capital expenditures for the year, then Company XYZ's free cash flow was $15 million - $5 million = $10 million Q 3. Describe and illustrate significance of human behaviour patterns in management control system. The significance of human behaviour patterns in management control system can be explained with the help of Informal Factors that influence Goal Congruence. In the informal forces both internal and external factors play a key role. A. External Factors External factors are norms of desirable behaviour that exist in the society of which the organization is a part. Illustration: A set of attitudes, often collectively referred to as the work ethic, which is manifested in employees' loyalty to the organization, their diligence, their spirit, and their pride in doing a good job (rather than just putting in time). Some of these attitudes are local that is, specific to the city or region in which the organization does its work. In encouraging companies to locate in their city or state, chambers of commerce and other promotional organizations often claim that their locality has a loyal, diligent workforce.

B. Internal Factors 1. Culture

The most important internal factor is the organization's own culture-the common beliefs, shared values, norms of behaviour and assumptions that are implicitly and explicitly manifested throughout the organization. Cultural norms are extremely important since they explain why two organizations with identical formal management control systems, may vary in terms of actual control. 2. Management Style

The internal factor that probably has the strongest impact on management control is management style. Usually, subordinates' attitudes reflect what they perceive their superiors' attitudes to be, and their superiors' attitudes ultimately stem from the CEO. 3. The Informal Organization

An informal organization is one which is not reliant on a hierarchical structure, typical of large-scale companies. It is not typical for an entire organization to be informal, as this could cause problems which are discussed in a moment, but formal organizations do tend to have informal ones within them.

4 (a) Differences in budgeting perspective of engineered and discretionary expense centre

1. Relationship of output and input In case of a discretionary expense budget relationship cannot be made between input and output. In engineered expense centre, relationship can be made between input and output.

2. Measuring efficiency

In engineered expense centre, difference between engineered costs and actual costs represent the efficiency of the organizational unit. In case of a discretionary expense, difference between budgeted input and actual input measures the efficiency of the organizational unit.

3. Costs Variability In discretionary expense centre costs tend to vary with volume from one year to the next but they tend not to vary with short run fluctuation in volume within a given year. By contrast costs in engineering expense centre are expected to vary with short run changes in volume.

Q 4 (b) Explain problems faced in pricing corporate services provided to business units organized as Profit Centres 1. Services are intangible in nature. This characteristic of services makes it difficult for pricing. Charging business units for services furnished by corporate staff units becomes challenging work due to intangibility of services. 2. While pricing corporate services, we exclude the cost of central service staff units over which business units have no control (e.g., central accounting, public relations, and administration). If these costs are charged at all, they are allocated, and the allocations do not include a profit component. The allocations are not transfer prices. 3. We need to consider two types of transfers: O For central services that the receiving unit must accept but can at least partially control the amount used. O For central services that the business unit can decide whether or not to use.

4.

Business units may be required to use company staffs for services such as

information technology and research and development. In these situations, the business unit manager cannot control the efficiency with which these activities are performed but can control the amount of the service received. There are three schools of thought about such services. One school holds that a business unit should pay the standard variable cost of the discretionary services. A second school of thought advocates a price equal to the standard variable cost plus a fair share of the standard fixed costs-that is, the full cost. A third school advocates a price that is equivalent to the market price, or to standard full cost plus a profit margin.

Q 5 (a) Describe inherent difficulties creation of profit centres may cause and advantages possible? Q 5(b) Under which situation creation of profit centre is not advisable.

Difficulties; 1. 2.
3.

Decentralized decision making will force top management to rely more on management Organization units that once cooperated as functional units may now be in competition Divisionalization may impose additional costs because of the additional management,

control reports than on personal knowledge of an operation, entailing some loss of control. with one another. An increase in profits for one manager may mean a decrease for another. staff personnel, and record keeping required, and may lead to task redundancies at each profit centre.
4.

There may be too much emphasis on short-run profitability at the expense of long-run

profitability. In the desire to report high current profits, the profit centre manager may skimp on R&D, training programs, or maintenance.

Advantages: 1. The top management is relieved of day to day affairs. They can concentrate on other

important issues.

2. 3. 4.

The top management gets ready made information on the profitability of individual The quality of decisions may improve because theyre being made by people closest to The speed of operating decisions can be increased since they do not have to be referred

components. the point of decision making. to the corporate headquarters.

Q.6 Which management control practices, if followed, in performance measurement of investment centres are likely to induce goal congruence, in respect of following assets a. (i) Idle assets (ii) Intangible assets (iii) Leased assets b. (i) Cash (ii) Receivables (iii) Inventories i)Idle Assets If a business unit has idle asset that can be used by other units, the business unit may be permitted to exclude them from the investment base if it classifies them as available. The purpose of this permission is to encourage business unit managers to release underutilized assets to units that may have better use for them. However, if the fixed assets cannot be used by other units, permitting the business unit manager to remove them from the investment base could result in dysfunctional actions

ii) Intangible Assets Some companies tend to be R&D intensive others tend to be marketing intensive. There are advantages to capitalizing intangible assets such as R&D and marketing and then amortizing them over a selected life. This method should change how the business unit manager views these expenditures. By accounting for these assets as long-term investments, the business unit manager will gain less short-term benefit from reducing out lays on such item,. iii) Leased Assets Suppose the business unit sold its fixed assets for their book value of $300,000, returned the proceeds of the sale to corporate headquarters, and then leased back the assets at a rental rate of

$60,000 per year. The business unit's income before taxes would decrease because the new rental expense would be higher than the depreciation charge that was eliminated. Nevertheless, economic valued added would increase because the higher cost would be more than offset by the decrease in the capital charge. Because of this, business unit managers are induced to lease, rather than own, assets whenever the interest charge that is built into the rental cost is less than the capital charge that is applied to the business unit's investment base b. i) Cash Most companies control cash centrally because central control permits use of a smaller cash balance than would be the case if each business unit held the cash balances it needed to weather the unevenness of its cash inflows and outflows. Business unit cash balances may well be only the "float" between daily receipts and daily disbursements. Consequently, the actual cash balances at the business unit level tend to be much smaller than would be required if the business unit were an independent company. ii) Receivables Business unit managers can influence the level of receivables, not only indirectly by their ability to generate sales, and directly, by establishing credit terms by approving individual credit accounts and credit limits, and by the collecting overdue amount. In the interest of simplicity, receivable included at the actual end-.of-period balances, although the average of intra-period balances is conceptually a better measure of the am should be related to profits. The usual practice is to take the simpler alternative-that is, receivables at the book If the business unit does not control credits and collections, receivables may be amount, which is the selling price less an allowance for bad debts. calculated on a formula basis. iii) Inventories Inventories ordinarily are treated in a manner similar to receivables that is they are often recorded at end-of-period amounts even though intraperiod averages would be preferable conceptually.

In these circumstances where inventory balances tend to be unrealistically low in

periods of inflation they should be valued at standard or average costs, and these same costs should be used to measure cost of sales on the business unit income statement If work-in-process inventory is financed by advance or progress payments from the customer, in case long manufacturing period, these payments either are subtracted from the gross inventory amounts or reported as liabilities. The corporate capital required for inventories is only the difference between the gross

inventory amount and accounts payable. If the business unit can influence the payment period allowed by vendors, then

including accounts payable in the calculation encourages the manager to seek the most favorable terms 7a) Explain the reasons for failure of balance score card? There is poor correlation measure between financial and non financial parameters. Most managers are trained to take only finances into account (i.e. money0 and not other factors. Measurement overloads or under load of each parameter (i.e. finances, customer relations, innovation, internal improvement) and management may lose focus.

Difficulty in establishing tradeoffs between financial and non financial parameters. The performance measurement ratios and bench marks must be updated periodically which is not done.

Quality measures are difficult to establish.

7b) Discuss special challenges faced in controlling R and D activities and possible management initiatives. Challenges faced in controlling R and D The results of R and D activities are difficult to measure quantitatively. The output is semi tangible in the form of patents, new products, and new designs, processes thus the output and input is difficult to appraise on an annual basis.

It is not possible to estimate the value of input to output. This is due to the technical nature which does not allow the management to measure its efficiency.

Financial control systems cannot be used to manage research activities thus alternative procedures have to be used.

There are problems associated with the budget and actual amount used for research.

Management initiatives

The company must not spend beyond its means in research. The R and D should be in the interest of the company. The performance of the R and D centre can be measured by preparing and presenting financial reports to appropriate levels of management. These reports can be prepared on a monthly or quarterly basis.

Comparison must be made between the actual and budget expenditure. The following is an explanation for better understanding. (Type of financial control: The financial control exercised in a discretionary expense centre is quite different from that in engineered centre the latter attempts to minimize operating cost by setting a standard and reporting actual costs against this standards. The main purpose of a discretionary expense budget on the other hand is to allow the manager to control Cost for particular in the planning. Costs are controlled primarily by deciding what task should be undertaken and what level of effort is appropriate for each. Thus in a discretionary expense centre financial control is primary exercised at the planning stage before the amount are incurred.

Measurement of performance: The primary job of the manager of a discretionary expense centre is to accomplish the desired output spending an amount that is on budget is satisfactory. This is in contrast with the report in an engineered expense centre which helps higher management to evaluate the manger efficiency. If these two types of responsibility centre are carefully distinguished management may treat the performance report for the discretionary

expense centre as if it were an indication of efficiency Control over spending can be exercised by requiring that the manger approved be obtain before the budget is over sometimes a certain percentage of overrun is permitted without additional approval if the budget really set forth the best estimate of actual cost there is 50 percent probability that it will overrun and this is the reason that some latitude is often permitted.

Control problems: The control of R & D centres, which are also discretionary expense centre is difficult for the following at least a semi tangible output reasons.

1. Results are difficult to measure quantitatively. As contrasted with administrative activities, R&D usually has at least a semi tangible output in patent, new products, or new processes. Nevertheless, the relationship of these outputs to inputs is difficult to measure and appraise. A complete product of an R&D group may require several year of effort; consequently input as stated in an annual budget may be unrelated to outputs. Even if an output can be identified a reliable estimate of its value often cannot be made. Even if the value of the output can be calculated, it is usually not possible for management to evaluate the efficiency of the R&D effort because of its technical nature. A brilliant effort may come up against an insuperable obstacle, whereas a mediocre effort may, by luck result in a bonanza.
2. The goal congruence problem in R&D centre is similar to that in administrative

centres. The research managers typically want to build the best research organization that money can buy, even though this is more expensive than the company can afford. A further problem is that research people often may not have sufficient knowledge of the business to determine the optimum direction of the research efforts. 3. Research and development can seldom be controlled effectively on an annual basis. A research project may take year s to reach fruition, and the organization must be built up slowly over a long time period. The principal cost is for the

work force obtaining highly skilled scientific talented is often difficult, and short term fluctuation in the work force are in efficient. It is not reasonable, therefore to reduce R&D costs in years when profits are low and increase them in year when profits are high. R&D should be looked at as a long term investment not as an activity that varies with short run corporate profitability.

The R&D continuum: Activities conducted by R&D organization lie along a continuum. At one extreme is basic research; the other extreme is product testing. Basic research has two characteristics: first, it is unplanned management at most can specify the general area that is to be explored second there is often a very long time lag before basic research result in successful new product introductions. Financial control system has little value in managing basic research activities. In some companies, basic research in included as a lump sum in the research program and budget. In others, no specific allowance is made for basic research as such; there is an understanding that scientists and engineers can devote part of their time to explorations in whatever direction they find most interesting, subject only to informal agreement with their supervisor. For product testing projects, on the other hand, the time and financial requirement can be estimated, not as accurately as production activities)

8a) What are the advantages in conducting an internal audit? It helps to study and evaluate the adequacy and effectiveness of accounting, financial and operating controls. It ascertains the extent to which the business assets are accounted for and safeguarded from losses. It evaluates the quality of performance in carrying out assigned responsibilities. It ascertains the degree of compliance with pre determined policies, plans and procedures.

It ascertains the authenticity of accounting and other data complied within the organization.

It tells the members of the management the objective analysis, comments and recommendations with respect to the activities of the business, which helps them in the efficient and reflective discharge of their responsibilities.

It helps to improve and add value to organizations operations. It helps to evaluate and improve the effectiveness of risk management, control and governance processes.

It provides inputs for the formulation and implementation of strategy.

8b) Illustrate the difference between financial and management audits with an example? Financial audit The accounts have been drawn up with reference to entries in the book of accounts. The entries in the book of accounts are adequately supported by underlying papers. No entry in the book of accounts must be omitted and anything that is not there in the book of account must not be present in the accounting statement. The information conveyed in the statement must be clear and unambiguous. The amounts in the financial statement must be properly classified, described and disclosed in accordance with the accounting standards. The statement of accounts must present a true and fair picture of the operational results and the assets and liabilities. Management audit It is conducted to see if the basic aims and objectives of the enterprise are being fulfilled. To see if the company is successful in adopting technological change.

Whether the management is efficient at all levels and the extent to which economies are possible. To see if the management structure is suitable. To see if the policies with regard to staff recruitment and training are adequate. To see if there is a proper communication system both upwards and downwards in the enterprise. To see if the return on capital is adequate and is comparable with other companies in the same industry. To see if the companys markets share is increasing and its comparison with its main competitor. To see the companys relationship with the outside world and whether its corporate image is satisfactory. Financial Audit It is concerned with financial aspects of business transactions of the year under audit Management Audit It is concerned with the review of the past Performance to ascertain whether it is in tune with the objectives, policies and procedures of the

enterprise. The auditor examines the past financial records to The management auditor reports on performance report his opinion on the truth and fairness of the representations made in the financial statements. Examination of the performance of the management is beyond his scope Past year '(Financial) transactions are Covered Enterprises such as companies, trust and societies etc. Financial audit is compulsory in the case of certain enterprises such as companies, trust and societies etc. The auditor reports to the owner, i.e. shareholders in the Case of a company There is legal compulsion as regards management audit. The auditor reports to the management of the management during a particular period and suggest ways to remedy the deficiencies, including modification of objectives, policies etc. No limit as to the period to be covered

9a) Explain some factors which may influence top management style and the implications of top management style on management control. The management control function in an organization is influenced by the style of senior management. The style of the chief executive officer affects the management control process in the entire organization. Similarly, the style of the business unit manager affects the units management control process, and the style of functional department managers affects the management control process in their functional areas. Differences in Management Styles Managers manage differently. Some rely heavily on reports and certain formal documents; others prefer conversations and informal contacts. Some are analytical; others use trial and error. Some are risk takers; others are risk averse. Some are process oriented; others are results oriented. Some are long-term oriented; others are short-term oriented. Some emphasize monetary rewards; others emphasize a broader set of rewards. Management style is influenced by the manager's background and personality. Background includes things like age, formal education, and experience in a given function, such as manufacturing, technology, marketing, or finance. Personality characteristics include such variables as the manager's willingness to take risks and his or her tolerance for ambiguity. Implications for Management Control The various dimensions of management style significantly influence the operation of the control systems. Even if the same reports with the same set of data go with the same frequency to the CEO, two CEOs with different styles would use these reports very differently to manage the business units. Style affects the management control process how the CEO prefers to use the information, conducts performance review meetings, and so on which in turn affects how the control system actually operates, even if the formal structure does not change under a new CEO. In fact, when CEOs change, subordinates typically infer what the new CEO really wants based on how he or she interacts during the management control process. Personal versus Impersonal Controls Presence of personal versus impersonal controls in organizations is an aspect of managerial style. Managers differ on how much importance they attach to formal budgets and reports as well as informal conversations and other personal contacts. Some managers are "numbers

oriented"; they want a large flow of quantitative information, and they spend much time analyzing this information and deriving tentative conclusions from it. Other managers are "people oriented"; they look at a few numbers, but they usually arrive at their conclusions by talking with people, judging the relevance and importance of what they learn partly on their appraisal of the other person. They visit various locations and spend time talking with both supervisors and staff to get a sense of how well things are going. Managers' attitudes toward formal reports affect the amount of detail they want, the frequency of these reports, and even their preference for graphs rather than tables of numbers, and whether they want numerical reports supplemented with written comments. Designers of management control systems need to identify these preferences and accommodate them. Tight versus Loose Controls A manager's style affects the degree of tight versus lose control in any situation. The manager of a routine production responsibility center can be controlled relatively tightly or loosely, and the actual control reflects the style of the manager's superior. Thus, the degree of tightness or looseness often is not revealed by the content of the forms or aspects of the formal control documents, rules, or procedures. It is a factor of how these formal devices are used. The degree of looseness tends to increase at successively higher levels in the organization hierarchy: higher-level managers typically tend to pay less attention to details and more to overall results. The style of the CEO has a profound impact on management control. If a new senior manager with a different style takes over, the system tends to change correspondingly. It might happen that the manager's style is not a good fit with the organization's management control requirements. If the manager recognizes this incongruity and adapts his or her style accordingly, the problem disappears. If, however the manager is unwilling or unable to change, the organization will experience performance problems. The solution in this case might be to change the manager.

9b) what are the advantages and disadvantages of two step transfer pricing and profit sharing methods. Two step pricing First, a charge is made for each unit sold that is equal to the standard variable cost of production. Second a periodic charge is made for the buying unit. One or both of these components should include a profit margin. The two step pricing method correct this problem

by transferring variable cost on a per unit basis, and transferring fixed cost and profit on a lump sum basis under this method the transfer price for product A would be 5$ for each unit that unit Y purchases plus $20000 per month for fixed cost. Plus $10000 per month for profit: if transfer of product A in a certain month are at the expected amount 5000 units then under the two step method unit y will pay the variable cost of $25000 plus $30000 for the fixed cost and profit a total of $55000 .this is the same amount as the amount it would pay unit x if the transfer price is less than 5000 units say 4000unoits.unit y would pay $50000 under the two step methods compared with the $44000 it would pay if the transfer price were $11 per unit. The difference is their transfer prices were for not using a portion of unit X capacity that it has reserved. Note that fewer than two step methods the company variable cost for product A is identifiable to unit Y variable cost for the product, and unit Y will make the correct short term marketing decisions. Unit Y also has information on upstream fixed costs and profit related to product A and it can use these data for long term decision. The fixed cost calculation in the two step pricing method is based on the capacity that is reserved for the production of product A that is sold to unit Y the investment represented by this capacity is allocated to product A. The return on investment that unit X earns on competitive product is calculated and multiplied by the investment assigned to the product. In the example we calculated the profit allowance as a fixed monthly amount. It would be appropriate under some circumstance to divide the investment into variable and fixed component. Then, a profit allowance based on a return on investment on variable assets would be added to the standard variable cost for each unit sold. Profit sharing If the two step pricing system just described is not feasible, a profit sharing system might be used to ensure congruence of business unit interest with company interest. This system operates somewhat as follows. 1. The product is transferred to the marketing unit at standard variable cost. 2. After the product is sold, the business units share the contribution earned which is selling price minus the variable manufacturing and marketing costs. This method of pricing may be appropriate if the demand for the manufactured product is not steady enough to warrant the permanent assignment of facilities as in the two step method. In general, this method accomplished the purpose of making the marketing units interest congruent with the companies. There are several practical problems in implementing such profit sharing system. First, there can be arguments over the way contribution is divided

between the two profit centers. Which is costly, time consuming and work against basic reason for decentralization namely autonomy of the business units mangers. Second, arbitrarily divided up the profit between units does not give valid information on the profitability of each segment of the organization. Third since the contribution is not allocated until after the sale has been made the manufacturing units contribution depends upon the marketing units ability to sell and on the actual selling price.

Manufacturing units may perceive this situation to be unfair

Two set of price: in this method, the manufacturing units revenue is credited at the outside sales price, and the buying unit is charged the total standard costs. The difference is changed to a headquarter account and eliminated when the business unit statement are consolidated, this transfer pricing method is sometimes used when there are frequent conflict between the buying and selling units that cannot be resolved by one of the other method both the buying and selling

There are several disadvantages to the system of having two set of transaction prices, however the sum of the business unit profit is greater than overall company profits, senior management must be aware of this situation in approved budget for the business units and in subsequent evaluation of performance against these budget. Also, this system create an illusion feeling that business units are making money while in fact the overall company might be losing after taking account of the debits to headquarter. Further this system might motivate business unit to concentrate more on internal transfers at the expense of outside sales

Question 10 and 11 are sums and will be sent later because I am not sure of the answers. Question 12 Refer Solution on Page 180 of Text Book

May 2004 Q1. Features of an ideal management control system Total system: MCS is an overall process of the enterprise which aims to fit together the separate plans for various segments to assure that each harmonizes with the others and that the aggregate effort of all of them on the entire enterprise is satisfactory. Monetary Standard: MCS is built around a financial structure and all the resources and outputs are measured and expressed in terms of money. Definite pattern: It follows a definite pattern and time table. It is a continuous process. Coordinated System: It is a fully coordinated and integrated system. Emphasis: It requires emphasis both on search for planning as well as control. Function of every manager: Managers at every level have to focus towards future operational and accounting data, taking into consideration past performance, present trends and anticipated economic and technological changes. The nature, scope and level of control will be governed by the level of managers exercising it. Existence of goals and plans: MCS is not possible without predetermined goals and plans. These two provide a link between future anticipations and actual performance. Forward Looking:

MCS is on the basis of past performance so that the future plans or guidelines can be laid down.

Continuous Process: It is a continuous process over human and material resources. It demands vigilance at every step. Deciding, planning and regulating the activities of people associated in the common task of attaining the objectives of the organization is the primary aim of MCS.

People oriented: It is the managers, engineers and operators who implement the ideas and objectives of the management. Coordination of the main divisions of the organization helps in smooth operations and less friction which results in the achievement of predetermined objectives.

Q 2. What the concept of free cash flow is as applied to an organization? Explain process of computation. (Ans.Text Bk pg 174-176 ) Q.3 Explain different organizational goals. Comment on goal of shareholder value maximization in particular. Goals Corporate goals are determined by the chief executive officer (CEO) of the corporation, with the advice of other members of senior management, and they are usually ratified by the board of directors. In many corporations, the goals originally set by the founder persist for generations. Examples are Henry Ford, Ford Motor Company; Alfred P. Sloan, General Motors Corporation; Walt Disney, Walt Disney Company; George Eastman, Eastman Kodak; and Sam Walton, Wal-Mart. 1. Profitability In a business, profitability is usually the most important goal. Return on investment can be found by simply dividing profit (i.e., revenues minus expenses) by investment, but this method does not draw attention to the two principal components: profit margin and investment

turnover."Profitability" refers to profits in the long run, rather than in the current quarter or year. Much current expenditure (e.g., amounts spent on advertising or research and development) reduce current profits but increase profits over time. 2. Maximizing Shareholder value

Achieving satisfactory profit is a better way of stating a corporation's goal, for two reasons. First, "maximizing" implies that there is a way of finding the maximum amount that a company can earn. This is not the case. In deciding between two courses of action, management usually selects the one it believes will increase profitability the most. But management rarely, if ever, identifies all the possible alternatives and their respective effects on profitability. Furthermore, profit maximization requires that marginal costs and a demand curve be calculated, and managers usually do not know what these are. Second, although optimizing shareholder value may be a major goal, it is by no means the only goal for most organizations. Certainly a business that does not earn a profit at least equal to its cost of capital is not doing its job; unless it does so, it cannot discharge any other responsibilities. But economic performance is not the sole responsibility of a business, nor is shareholder value. Most managers want to behave ethically, and most feel an obligation to other stakeholders in the organization in addition to shareholders. However certain valid principles hold true. A course of action that decreases expenses without affecting another element, such as market share, is sound. So is a course of action that increases expenses with a greater than proportional increase in revenues, such as expanding the advertising budget. So, too, are actions that increase profit with a less than proportional increase in shareholder investment, such as purchasing a cost-saving machine. These principles assume, in all cases, that the course of action is ethical and consistent with the corporation's other goals. 3. Minimizing Risk An organization's pursuit of profitability is affected by management's willingness to take risks. The degree of risk-taking varies with the personalities of individual managers. Nevertheless there is always an upper limit; some organizations explicitly state that management's primary responsibility is to preserve the company's assets, with profitability considered a secondary

goal. The Asian .financial crisis during 1996-1998 is traceable, in large part, to the fact that banks in Asia's emerging markets made what appeared to be highly profitable loans without paying adequate attention to the level of risk involved. 4. Multiple Stakeholder Approach

Organizations participate in three markets: the capital market, the product market, and the factor market. A firm raises funds in the capital market, and the public stockholders are therefore an important constituency. The firm sells its goods and services in the product market, and customers form a key constituency. It competes for resources such as human capital and raw materials in the factor market and the prime constituencies are the company's employees and suppliers and the various communities in which the resources and the company's operations are located. The firm has a responsibility to all these multiple stakeholders-shareholders, customers, employees, suppliers, and communities. Ideally, its management control system should identify the goals for each of these groups and develop scorecards to track performance. Q 4. Short Notes a) Concept of profit centre in NPO- (Ans.Text be-pg 218-220) b) Impact of top management style on Management control- (Ans. Text bk-pg237-239) c) Management control in matrix structures-(Ans.Text be-pg 66-68) d) Implications of differentiated strategies on controls-(Ans.Text bk-pg 230-232) 5)a) Transfer Pricing is not an accounting tool. Comment with Illustrations If a group has subsidiaries that operate in different countries with different tax rates, manipulating the transfer prices between the subsidiaries can scale down the overall tax bill of the group. For example the tax rate in Country A is 20% and is 50% in Country B. In the larger interest of the group, it would be advisable to show lower profits in Country B and higher profits in Country A. For this, the group can adjust the transfer price in such a way that the profits in Country An increase and that in Country B get reduced. For this the group should fix a very high transfer price if the Division in Country A provides goods to the Division in Country B. This will maximize the profits in Country A and minimize the profits in Country B.

The reverse will be true if the Division in Country A acquires goods from the Division in Country B. There is also a temptation to set up marketing subsidiaries in countries with low tax rates and transfer products to them at a relatively low transfer price. Transfer price is viewed as a major international tax issue. While companies indulge in all types of activities to lower their tax liability, the tax authorities monitor transfer prices closely in an attempt to collect the full amount of tax due. For this they enter into agreements whereby tax is paid on specific transactions in one country only. But if companies set unrealistic transfer price to minimize their tax liabilities and the same is spotted by the tax authority, then the company is forced to pay tax in both countries leading to double taxation. There have been instances where companies have fixed unrealistic transfer prices. The first case relates to Hoffman La Roche that imported two drugs Librium and Valium into UK at prices of 437 pounds and 979 pounds per kilo respectively. While the tax authorities in UK accepted the price, the Monopolies Commission did not accept the company's argument, since the same drugs were available from an Italian firm for 9 pounds and 28 pounds per kilo. The company's lawyers argued the case before the Commission on two grounds viz. 1. The price was not set on cost but on what the market would bear and 2. The company had incurred an R&D cost that was included in the price. These arguments did not go well with the Commission and the company was fined 1.85 million pounds for the manipulative practices adopted while fixing the transfer price. The second case is of Nissan. The company had falsely inflated freight charges by 40-60% to reduce the profits. The manipulation helped the company to hide tax to the tune of 237 million dollars. The next year Nissan was made to pay 106 million dollars in unpaid tax in the USA because the authorities felt that part of their US marketing profits were being transferred to Japan, as transfer prices on import of cars and trucks were too high. Interestingly the Japanese tax authorities took a different view and returned the double tax.

With a view to avoid such cases from recurring, Organisation for Economic Cooperation and Development issued some guidelines in 1995. These guidelines aim at encouraging world trade. They evolved what came to be known as the arm's length price. The principle states that the transfer price would be arrived at on the basis as if the two . Companies are independent and unrelated. The price is determined through: Comparable Price Method where the price is fixed on the basis of prices of similar products or an approximation to one. Gross Margin Method where a gross margin is established and applied to the seller's manufacturing cost. In spite of all these efforts, it has to be admitted that setting a fair transfer price is not easy. So the onus of proving the price has been put on the taxpayer who is required to produce supporting documents. If the taxpayer fails to do this he is required to pay heavy penalty. For example, in USA, failure to provide documentary evidence results in a 40% penalty on the arm's length price. In UK the penalty is to the tune of 100% of any tax adjustment. Other countries are also in the process of evolving tight norms for the same. Countries across the globe also allow the taxpayer to enter into an Advance Pricing Agreement whereby dispute can be avoided and so also the costly penalty of double taxation and penalty. 5) b) Market Price is ideal transfer price even in limited markets. Comment By limited market it means that the markets for buying and selling profit centres may be limited. Even in case of limited market the transfer price that is ideal or satisfies the requirement of a profit centre system is the competitive price. In case if a company is not buying or selling its product in an outside market there are some ways to find the competitive price. They are as follows: 1. If published market prices are available, they can be used to establish transfer prices. However, these should be prices actually paid in the market-place and the conditions that exist in the outside market should be consistent with those existing within the company. For example, market prices that are applicable to relatively small purchases are not valid in this case. 2.Market prices are set by bids. This generally can be done only if the low bidder has a reasonable chance of obtaining the business. One company accomplishes this by buying about one-half of a particular group of products outside the company and one-half inside the company.

The company then puts all of the products out to bid, but selects one-half to stay inside. The company obtains valid bids, because low bidders can expect to get some of the business. By contrast, if a company requests bids solely to obtain a competitive price and does not award the contracts to the low bidder, it will soon find that either no one bids or that the bids are of questionable value. 3.If the production profit centre sells similar products in outside markets, it is often possible to replicate a competitive price on the basis of the outside price. 4.If the buying profit centre purchases similar products from the outside market, it may be possible to replicate competitive prices for its proprietary products. This can be done by calculating the cost of the difference in design and other conditions of sale between the competitive products and the proprietary products. So we see from the above arguments that market price is ideal transfer price even in limited markets 6) Explain and illustrate with one example differences between 3 forms of internal auditFinancial, Operational & Management. Financial AuditFinancial Audit is a historically oriented, independent evaluation performed by internal auditor or external auditor for the purpose of attesting to the fairness, accuracy and reliability of the financial data, providing protection for the entity's assets; evaluating the adequacy and accomplishment of the system (internal control) designed, provide for the aforementioned Fairness and Protection, Financial data, while not being the only source of evidence, are the primary evidential source. The evaluation is performed on a planned basis rather than a request". Financial audit takes care of the protective aspect of the business and it does not normally carry out constructive appraisal function of the business operations. It helps in detection and prevention of fraud. It also verifies whether documentation and flow of activities arc in conformity with the internal control system introduced and developed within the organization. It helps coordinating with statutory auditor to help them in proper discharge of their function. Besides, financial audit also ensures compliance with statutory laws especially in financial and accounting matters.

Objectives of Financial Audit -To see that established accounting systems and procedures have been complied with -To see that proper records have been maintained for the fixed assets of the Concern to look into correctness of the financial data and records along with correctness of the accounting procedure followed. -To see whether scrap, salvage and surplus materials have been properly accounted for etc. -To see that internal control system has been working properly. -To see that any abrupt variation in sales, purchases etc.; with respect to immediate previous year are not due to any irregularity -To see that the credit control has been strictly followed. -To see that all payments have been made with proper authorization and approval. . -To see that preparation of salary and wage pay roll has been properly done.

Management Audit It is a complex task closely related with the process of management. It is highly result oriented. It requires inter/multi-disciplinary approach as it involves examination, review and appraisal of various policies and actions of management on the basis of certain norms/standards. It undertakes comprehensive and critical review of all organizational activities with wider perspective. Objectives To ascertain the provision of proper control at different levels, their effectiveness I in accomplishing management goals. Ascertain objectives of the organization are properly communicated and understood at all levels. To reveal defects or irregularities in any of the elements examined and to indicate what improvements are possible to obtain the best results of the operations of the company. To assist the management to achieve the most efficient administration of its operations. To suggest to the management the ways and means to achieve the objectives if the management of the organization itself lacks the knowledge of efficient management.

It aims to achieve the efficiency of management and assess the strength and weaknesses of the organization structure, its management team and its corporate culture. To ascertain the provision of proper control at different levels, their effectiveness in accomplishing management goals. Ascertain objectives of the organization are properly communicated and understood at all levels. To reveal defects or irregularities in any of the elements examined and to indicate what improvements are possible to obtain the best results of the operations of the company. To assist the management to achieve the most efficient administration of its operations. To suggest to the management the ways and means to achieve the objectives if the management of the organization itself lacks the knowledge of efficient management. It aims to achieve the efficiency of management and assess the strength and weaknesses of the organization structure, its management team and its corporate culture. To help the management at all levels in the effective and efficient discharge of their duties and responsibilities.

Operational Audit An Operational audit tests a companys internal systems and procedures used to produce its goods and services sold to consumers. These audits test production operations for efficiency and effectiveness. Audits may be conducted by internal employees or external auditors with business experience relating to the company's operational procedures. Operational audits are usually a deeper review of company operations than a financial audit, which is conducted in an after-the-fact audit process. Benefits from operational audits include objective opinions, improved workflow or cost allocation processes and quicker turnaround times. Staff accountants or accountants from public accounting firms usually conduct operational audits. Using staff accountants for an internal operational audit allows companies to have an objective opinion on how well the company is using their business resources. Department managers may have a tendency to fudge their audit figures since they often receive compensation bonuses or pay increases from improved operations. Public accounting firms sometimes are used for operational audits to inform outside stakeholders on the operational strength of a companys operations. Objective audit opinions may lead companies to increase their production cost controls.

7) What is a Balance Score Card? What is the process of implementation and difficulties in implementation ? The Balanced Scorecard (BSC) is a performance management tool which began as a concept for measuring whether the smaller-scale operational activities of a company are aligned with its larger-scale objectives in terms of vision and strategy. By focusing not only on financial outcomes but also on the operational, marketing and developmental inputs to these, the Balanced Scorecard helps provide a more comprehensive view of a business, which in turn helps organizations act in their best long-term interests. Organizations were encouraged to measurein addition to financial outputswhat influenced such financial outputs. For example, process performance, market share / penetration, long term learning and skills development, and so on. The underlying rationale is that organizations cannot directly influence financial outcomes, as these are "lag" measures, and that the use of financial measures alone to inform the strategic control of the firm is unwise. Organizations should instead also measure those areas where direct management intervention is possible. In so doing, the early versions of the Balanced Scorecard helped organizations achieve a degree of "balance" in selection of performance measures. In practice, early Scorecards achieved this balance by encouraging managers to select measures from three additional categories or perspectives: "Customer," "Internal Business Processes" and "Learning and Growth." The balance scorecard suggests that we view the organization from four perspectives, and to develop metrics, collect data and analyze it relative to each of these perspectives: The learning and growth perspective: To achieve our vision, how will we sustain our ability to change and improve? The business process perspective: To satisfy our shareholders and customers what business processes must we excel at? The customer perspective: To achieve our vision, how should we appear to our customer? The financial perspective : To succeed financially, how should we appear to our shareholders? Implementing a Balanced Scorecard We can summarize the implantation of a balanced scorecard in four general steps; 1. Define strategy.

2. Define measure of strategy. 3. Integrate measures into the management system. 4. Review measures and result frequently. Each of these steps is iterative, requiring the participation of senior executive and employees throughout the organization 1 Define Strategy The balance scorecard builds a link between strategy and operational action. As a result it is necessary to begin the process of defining a balanced scorecard by defining the organization goals are explicit and what that targets have been developed. 2 Define Measures of Strategy The next step is to develop measures in support of the articulate strategy. It is imperative that the organization focuses on a few critical measures at this point; otherwise management will be overloaded with measures. Also, it is important that the individual measures be linked with each other in a cause effect manner 3 Integrated Measures into the management system The balanced scorecard must be integrated with the organization formal and informal structure, its culture, and its human resources practice. While the balanced Scorecard gives some means for balancing measures, the measures can still become unbalanced by others system in the organization such as compensation policies that compensate the manager strictly based on financial performance. 4 Review Measures and result Frequently Once the balance scorecard is up and running it must be consistently reviewed by senior management. The organization should be looking for the following How do the outcome measures say the organization is doing? How do the driver measures say the organization is doing? How has the organizations strategy changed since the last review? How has the scorecard measures changed? The most important aspects of these reviews are as follows; They tell management whether the strategy is being implemented correctly and how successfully the strategy is working. They show that management is serious about the importance of these measures.

They maintain alignment of measure to ever changing strategies. Difficulties in implementing Balanced Scorecard The following problems unless suitably dealt with, could limit the usefulness of the balanced scorecard approach:

Poor correlation between no financial measures and result. Fixation on financial result. No mechanism for improvement. No mechanism for improvement. Measures overload.

Poor Correlation between No financial measures and result Simply put there is no guarantee that future profitably will allow targets achievement in any no financial area. This is probably the biggest problem with the balanced scorecard because there is an inherent assumption that future profitability does follow from achieving the scorecard measures, identifying the cause effect relationships among the different measures is easier said than done. This will be a problem with any system that is trying to develop proxy measures for future performance. While this does not mean that the balanced Scorecard should be abandoned it is imp that comp adopting such a system understand that the links between no financial measures and financial performance are still poorly understood. Fixation on Financial Results As previously discussed not only are most senior managers well trained and very adept with financial measures but they also most keenly feel pressure regarding the financial performance of their comp. Shareholder are vocal and the board of directors often applies pressure on the stakeholders behalf .this pressure often overwhelms the long term uncertain payback of the no financial measures. Non mechanism for Improvement One of the most overlooked pitfalls of the balanced scorecard is that a company cannot achieve Stretch goals if the Company has no mechanism for improvement .Unfortunately achieving many of these goals require complete shifts in the way that business is done yet the company often does not have mechanism to make those shifts . The mechanism available takes additional resource and requires a changed in the company culture. These changes do not happen

overnight nor do they respond automatically to a new stretch targets. Inertia often works against the company employees are accustomed to a self limited cycle of setting targets, missing those targets and readjusting the targets to reflect what was actually achieved. Without a method for making improvement, improvements are unlikely to consistently happen no matter how good the stretch goal sound. Measurement overload How many critical measures can one manager track at one time without losing? Unfortunately there is no right answer to this question except it is more than 1 and less than 50. It too few then the manager is ignoring measures that are critical to creating success. If it too many then the manager may risk losing focus and trying to do too many things at once. 8) MCS designers apparently disagree whether a SINGLE measure to evaluate profit performance and capital Investment performance is preferable or SEPARATE measures for each are preferable- Comment

Solution to be sent later

9) What are the different methods to measure profits of a profit centre in organisations? Which different messages each type of measure is likely to convey to managers? When financial performance in a responsibility centre is measured in terms of profit, which is the difference between the revenues and expenses, the responsibility centre is called a profit center.Profit as a measure of performance is especially useful since it enables senior management to use one comprehensive measure instead of several measures that often point to different directions. There are two types of profitability measurements in a profit centre, just as there are for the organization as a whole. There is, first, a measure of management performance, in which the focus is on how well the manager is doing. This measure is used for planning, coordinating and controlling the day-to-day activities of the profit centre. Second, there is a measure of economic performance, in which the focus is on how well the profit centre is doing as an economic entity. The message given by these two measures may be quite different. Types of Profitability measures:

In order to evaluate the economic performance of a profit centre, one must use net income after allocating all costs. However, in evaluating the performance of manager, any of five different measures of profitability can be used. 1) Contribution Margin: The logic behind using contribution margin as a measure is that fixed expenses are not controllable by the manager, and therefore he should focus on maximizing the spread between revenue and expenses. But the problem with this is that some fixed costs are controllable and all fixed costs are partially controllable. A focus on the contribution margin tends to direct attention away from this responsibility. 2) Direct Profit: This measure shows the amount that the profit centre contributes to the general overhead and profit of the corporation. It incorporates all expenses incurred in or directly traced to the profit centre, regardless of whether these items are entirely controllable by the profit centre manager. A weakness of this measure is that it does not recognize the motivational benefit of charging headquarters costs. 3) Controllable Profit: Headquarters expenses are divided into two categories: controllable and non-controllable. The controllable expenses are controlled by business unit manager. Consequently, if these costs are included in the management system, the profit will be after the deduction of all expenses that are influenced by profit centre manager. 4) Income before Taxes: In this measure, all corporate overhead is allocated to profit centres. The basis of allocation reflects the relative amount of expense that is incurred for each profit centre. If corporate overheads are allocated to profit centres, budgeted costs, not actual costs, should be allocated. Then the performance report will show an identical amount in the budget and actual columns for such overheads. 5) Net Income: Here, companies measure performance of domestic profit centres at the bottom line, the amount of net income after income tax. There are two arguments 1) Income after tax is constant percentage of the pretax income, so there is no advantage in incorporating income taxes 2) many decisions that have impact on income taxes are made at headquarters, and it is believed that profit centre manager should not be judged by the consequences of these decisions.

Q10(a) Explain special characteristics of professional organizations which impact management control?

Research & development organizations, law firms, accounting firms, health care organizations, engineering firms, architectural firms, consulting firms, advertising agencies, symphony and other arts organizations, and sports organizations (such as baseball teams) are examples of organizations whose products are professional services Special characteristics: Goals: A dominant goal of manufacturing company is to earn a satisfactory profit, specifically a satisfactory return on asset employed. A professional organization has relatively few tangible asset; its principles asset is the skill of its professional staff which does not appear on its balance sheet. Returns on asset employed, therefore, is essentially meaningless in such organization. Their financial goal is to provide adequate compensation to the professional. Professionals: Professional organizations are labours intensive and labour are of a special type. Many professional prefer to work independently, rather than as apart of team. Professional who are also managers tend to work only part time on management activities; E.g. senior partners in an accounting firm participate actively in audit engagement; senior partners in law firms have clients. Education for most professional does include education management, but quite naturally stresses the skill of the profession, rather than management, for this and other reason, professional tend to look down on manager. Professional tends to give inadequate weight to the financial implication of their decision; they want to do the best job they can, regardless of its cost. This attitude affects the attitude of supports staff and non professional in the organization; it tends to inadequate cost control. Output and input measurement: The output of professional organization cant be measured in physical terms, such as units, tons, or gallons. We can measure the number of hours a lawyer spends on case, but this measure of input, not output. Output is the effectiveness of the lawyers works, and this is not measured by number of pages in brief or the number of hours in the court room. We can measure the number of patients a physician treats in a day, even classify this visit by type of complaint; but this by no means equivalent to measuring the amounts or the quality

of service the physician has provided. At most, what is measured is the physician's efficiency in treating patients. Revenues earned is one measure of output in some professional organizations, but these monetary amounts, at most, relate to the quantity of services rendered, not to their quality (although poor quality is reflected in reduced revenues in the long run). Small size:

With few exceptions, such as some law firms & accounting firms, professional organization are relatively small and operate at a single location. Senior management in such organization can personally observe what is going on and personally motivate employees. Thus, there is less need for sophisticated management control system, with profit centre and formal performance reports. Nevertheless, even a small organization needs a budget, a regular comparison of performance against budget, and away of relating compensation to performance. Performance appraisal: The performance of professionals is easy to judge. Appraisal of the large percentage of professionals who are within the extremes is much more difficult. For some professions, objective measures of performance are sometimes unavailable. The assessment of performance is finally a matter of human judgment by superiors, peers, self, subordinates, and clients. Some systems require numerical ratings of specified attributes of performance and provide for a weighted average of these ratings. Compensation may be tied, in part, to these numerical ratings. In Appraisals by a professional's peers, or by subordinates, is sometimes part of a formal control system. In some organizations, individuals may be asked to make a selfappraisal. Expressions of satisfaction or dissatisfaction from clients are also an important basis for judging performance, although such expressions may not always be readily forthcoming. The budget can be used as the basis for measuring cost performance, and the actual time taken can be compared with the planned time. Budgeting and control of discretionary expense: They are as important in a professional firm as in a manufacturing company. Such financial measures are relatively unimportant in assessing a professional's contribution to the firm's, profitability, however. The professional's major contribution is related to quantity and above all quality of work, and its appraisal must be largely subjective. Furthermore, the

appraisal must be made currently; it cannot wait until one learns whether a new building is well designed, a new control system actually works well, In some professions, internal audit procedures are used to control quality. In many accounting firms, the report of an audit is reviewed by a partner other than the one who is responsible for it, and the work of the whole firm is peer reviewed by another firm. [Q10(b)] What are interactive controls? Refer Pages 208-210 Bandgar Q11 Veena Pvt. Ltd., a small multiproduct company is taken over by a multinational company (e.g. Hindustan Lever.) What changes in the control system would you expect and why? Since Veena is a small multiproduct company it would require changes in control system which would be related to transfer pricing a, as this company would generally provide inputs to HUL. Thus the domestic operations generally involve transfer of goods and services only In view of this difference many other considerations, in addition to the criteria used in domestic operations for the determination of transfer price, are involved. These include: (a) Fair Price: This is an important factor one needs to consider while determining the transfer price for foreign operations. Companies that enter into joint ventures must ensure that the transfer price charged is fair. If such companies charge a higher transfer price, it would reduce the profits of the joint venture and as a result reduce the foreign partner's share of profits. (b) Government Regulations: All countries have a regulatory framework under which business units operate. Where government rules and regulations regarding transfer prices are lenient, the parent company should fix a higher transfer price for all transfers to countries with high income tax rates. This approach would enable the parent company to minimize taxes in such countries. (c) Exchange Control Restrictions: Every country has foreign exchange control regulations. These regulations impose a limit on the amount of foreign exchange available for the import of certain goods. To accommodate the foreign subsidiary the parent company may have a lower transfer price so that the subsidiary is able to import a larger quantity of required goods.

(d) Income Tax Regulations: The rates of income tax vary from country to country. To overcome this difference the transfer price should be so fixed that countries with low tax rates show profits while others end up with a loss. This helps the parent company to reduce its taxes on a global basis. (e) Desire to accumulate funds: A company that wishes to accumulate funds in a particular country may fix the transfer prices in such a manner that it facilitates shifting of funds into that country. (I) Tariffs and Duties: No country likes high imports. In order to restrict imports countries impose restrictions such as quantitative restrictions, high duties and tariffs and banning import of products. The general practice is to charge import duties as a percentage of the value of products imported, although a lower tariff may be levied if the import value is lower. It is seen that the impact of tariffs on the profitability of foreign operations is generally the reverse of the incidence of income taxes in transfer pricing. As such a low transfer price would lead to low import duties on transfer, the profit arising in that country would be high. This results in high income taxes in that country. It is therefore advisable that companies must compute the net effect of these factors while determining transfer prices. In designing performance evaluation systems for acquired Veena company, HUL could use the following guidelines: Subsidiary managers should not be held responsible for translation effects. The

simplest way to achieve this objective is to compare budgets and actual results using the same metric and isolate inflation-related effects through variance analysis. It is pointless for managers to worry about the appropriate metric. The MNE should choose whatever metric is more convenient. Transaction effects are best handled through centralized coordination of the MNE's overall hedging needs. This is likely to be cheaper and simpler, and it prevents the subsidiary manager from becoming a foreign exchange rate forecaster and speculator. The subsidiary manager should be held responsible for the dependence effects Evaluation of the subsidiary as a basis for a decision to locate operations in a of exchange rates resulting from economic exposure. country or to relocate operations from a country should reflect the consequences of translation. Transaction and economic exposures.

Q12-13 problems not done

MAY 2003 Q1a What are the objectives of Transfer prices ? Under which conditions a Transfer pricing mechanism is likely to induce goal congruence? Refer Page 130 Bandgar (6.3 for the first part of the answer and 6.2 for the second part of the answer) Q1(b)] Explain the advantages and disadvantages of two step pricing and profit sharing methods? Two step pricing: First, a charge is made for each unit sold that is equal to the standard variable cost of production. Second a periodic charge is made for the buying unit. One or both of these components should include a profit margin. The two step pricing method correct this problem by transferring variable cost on a per unit basis, and transferring fixed cost and profit on a lump sum basis under this method the transfer price for product A would be 5$ for each unit that unit Y purchases plus $20000 per month for fixed cost. Plus $10000 per month for profit: if transfer of product A in a certain month are at the expected amount 5000 units then under the two step method unit y will pay the variable cost of $25000 plus $30000 for the fixed cost and profit a total of $55000 .this is the same amount as the amount it would pay unit x if the transfer price is less than 5000 units say 4000unoits.unit y would pay $50000 under the two step methods compared with the $44000 it would pay if the transfer price were $11 per unit. The difference is their transfer prices were for not using a portion of unit X capacity that it has reserved. Note that under two step method the company variable cost for product A is identifiable to unit Y variable cost for the product, and unit Y will make the correct short term marketing decisions. Unit Y also has information on upstream fixed costs and profit related to product A and it can use these data for long term decision. The fixed cost calculation in the two step pricing method is based on the capacity that is reserved for the production of product A that is sold to unit Y the investment represented by this capacity is allocated to product A. The return on investment that unit X earns on competitive product is calculated and multiplied by the investment assigned to the product. In the example we calculated the profit allowance as a fixed monthly amount. It would be appropriate under some circumstance to divide the investment into variable and fixed component. Then, a profit allowance based on a return on investment on variable assets would be added to the standard variable cost for each unit sold.

Advantages - In this method, the manufacturing units revenue is credited at the outside sales price, and the buying unit is charged the total standard costs. The difference is changed to a headquarter account and eliminated when the business unit statement are consolidated, this transfer pricing method is sometimes used when there are frequent conflict between the buying and selling units that cannot be resolved by one of the other method both the buying and selling Disadvantages - There are several disadvantages to the system of having two set of transaction prices, however the sum of the business unit profit is greater than overall company profits, senior management must be aware of this situation in approved budget for the business units and in subsequent evaluation of performance against these budget. Also, this system create an illusion feeling that business units are making money while in fact the overall company might be losing after taking account of the debits to headquarter. Further this system might motivate business unit to concentrate more on internal transfers at the expense of outside sales The fact that the conflict between the business units would be lessened under this system could be viewed as a weakness. Sometime, it is better for the headquarter to be aware of the conflict arising out of transfer prices because such conflict may signal problem in either the organizational structure or In other management systems. Profit sharing: If the two step pricing system just described is not feasible, a profit sharing system might be used to ensure congruence of business unit interest with company interest. This system operates somewhat as follows. 1. The product is transferred to the marketing unit at standard variable cost. 2. After the product is sold, the business units share the contribution earned which is selling price minus the variable manufacturing and marketing costs. Advantages - This method of pricing may be appropriate if the demand for the manufactured product is not steady enough to warrant the permanent assignment of facilities as in the two step method. In general, this method accomplished the purpose of making the marketing units interest congruent with the companies.

Disadvantages - There are several practical problems in implementing such profit sharing system. First, there can be arguments over the way contribution is divided between the two profit centres. Which is costly, time consuming and work against basic reason for decentralization namely autonomy of the business units mangers. Second, arbitrarily divided up the profit between units does not give valid information on the profitability of each segment of the organization. Third since the contribution is not allocated until after the sale has been made the manufacturing units contribution depends upon the marketing units ability to sell and on the actual selling price. Manufacturing units may perceive this situation to be unfair 2 a. Management control lies between strategy formulation and operational control. MCS has certain limitations which are called as boundaries; that distinguish it from other planning and control systems namely strategy formulation, operational control and financial control. Strategy formulation has found to be the least systematic, operational control, the most systematic and MCS falling in between. Strategy formulation focuses on the long run, operational control on short term operating activities and MCS lies in between. Strategy formulation is based on rough approximations of the future, operational control makes use of current accurate data and MCS lies in between. The three concepts are explained as follows4. Strategy formulationIts a planning process used by firms to decide on the goals of the organization and strategies to be used for achieving these goals. Goals are the overall aims of the organisation. Strategies are the plans for achieving the goals. Once a firm has formulated its strategies, it operates in accordance with the strategies. They may be re examined during the annual strategic planning exercise and some of them may be changed or modified. Steps in strategy formulation are as followsg. h. i. Framing mission and objectives Analysis of internal environment Analysis of external environment

j. k. l.

Gap analysis Framing alternative strategies Choice of strategy

5.

Operational control-

Its a process used for ensuring the tasks which are specified are carried out efficiently and effectively. It involves the control of individual tasks. The rules to be followed for accomplishing the tasks are prescribed as part of the management control process. Many operational control activities are scientific in nature.

6.

Management Control-

It involves the process of implementing strategies. There is generally a fixed time table and a series of steps in accordance with which management control takes place. It is a type of planning and control activity that is done by the organization. It is a process by which management influences other members of the organisation to implement the strategies effectively. Thus management control involves the behaviour of managers as managers interact with other managers and this cannot be shown in the form of equations. b. Pg 218-220 c. Matrix Structure and control (Pg 66 & 107) The matrix organization is a combination or merger of the functional and divisional structures. Product lines are arranged along one arm of the matrix, across the other arm could be arranged either functions or geographical divisions. A matrix form allows new businesses to plug into existing functional resources. Features of a Matrix Organization: Built around a specific project Project mgr draws personnel from various functional departments

Project and Functional mgrs play different roles Personnel assigned to project have 2 bosses.

d. Free Cash Flow (Pg 174 & 27) Free cash flow is a measure of financial performance calculated as operating cash flow minus capital expenditures. FCF represents the cash that a company is able to generate after laying out the money required to maintain or expand its asset base. FCF is calculated as Net income + Amortization/Depreciation Changes in Working capital Capital Expenditure. If a company can reduce its inventories, its cash holding or even its receivables, then its net inv in op profit will go down. If these actions do not harm op profit then free cash flows will increase, which will lead to a higher stock price. Cash conversion Cycle: Companies typically follow a cycle in which they purchase inventory, sell goods on credit and then collect accounts receivables. This cycle is called the Cash conversion cycle. Inventory conversion period: Average Time required to convert materials into finished goods and then to sell those goods. Receivables collection period: Average time required to convert the Receivables into cash that is to collect cash following a sale. Payables Deferred period: Average length of time between the purchase of materials and payment of cash for them. Cash Conversion cycle: Average length of time a rupee is tied up in current assets.

a. Pitfalls of Balance Score card i. ii. iii. There exists poor correlation between financial and non financial measures Managers are obsessed with monetary measures Measurement overload or underload of each parameter is possible and the management may lose focus iv. Difficult to establish a trade off between financial and non financial measures/parameters v. vi. Performance measures must be updated periodically which is not done Quality measures are difficult to establish.

b. Pg 82-83 4 a. Economic Value added is the corporate surplus to be shared with the employees, management and shareholders. A business entity should earn enough to cover its cost of capital and surplus to grow. Economic Value Added (EVA) is calculated as NOPAT Cost of Capital. An investment centre is a responsibility centre in which the manager is held responsible for the use of assets as well as for revenues and expenses of the centre. The manager is expected to earn a satisfactory return on capital employed in the business units. Investment centres have the measurement problems involved in defining expenses and revenues. These centres also raise some problems regarding the measurement of the assets employed, valuation of fixed assets and current assets and valuation of liabilities. An important goal of a company is to optimize return on shareholders equity. It may not be practical to use such a measure to evaluate the performance of the business unit managers on a regular basis. For this purpose, accounting rate of return is the best measure of business managers performance. Economic value added is conceptually superior to return on investment (ROI) in evaluating business unit managers.

Its merits are 5. Helps measure corporate performance and performance of business

segment 6. 7. 8. Tells how managers are creating wealth Most appropriate determination of cost of capital Strong tool for business planning

Its demerits are 5. 6.


7.

failure to consider the future prospects of the company Requires a lot of adjustments to financial information company Requires a tradeoffs between accuracy and simplicity of calculation,

since very complicated adjustments result in a lack of credibility. 8. Doubt about universal suitability of EVA.

b. Pg 69 Management Control systems are formal control systems which include strategic plans and budgets and reports. Formal communication system is structured as per the 'hierarchy outlined in the organization chart. The system has the following four components: (a) Strategic plan and programme (b) Budgeting (c) Operations and measurement in responsibility centres (d) Reporting (a) Strategic Plan and Programme

The foundation of management control process lies in the organization's goals and its strategies for attaining these goals. A strategic plan is prepared in order to implement the strategies, after carefully considering opportunities and threats in the external environment as well as the strengths and weaknesses in the internal environment. Thus, a strategic plan and programme is prepared as a guideline to budgeting. (b) Budgeting The strategic plan is converted to an annual budget incorporating planned expenditure and revenues for individual responsibility centres. Expenses and revenues are marked for each responsibility centre period wise, say monthly, quarterly, half yearly, and annually. (c) Operations and Measurement Responsibility centres operate within the framework of the budget, established standards, standing instructions, practices and operating procedures embodied in 'rules', and 'manuals'. Thus, besides budget, the responsibility centres are also guided by a large number of rules. They record the resources actually used and revenue earned. They also classify the data by programmes as well as by responsibility centres for performance measurement. (d) Reporting Actual performance is analyzed, measured and reported against plan, indicating variances and highlighting areas of weaknesses. If the performance is satisfactory, feedback information is sent to the responsibility centre concerned for praise or reward. If the same is unsatisfactory feedback communication is sent to the responsibility centre concerned for corrective action. If such action requires to be included in the budget, then the latter is revised to give effect to the changed position. If required, then the plan itself can be revised and a new basis of control may be established. It is important for designers of formal systems to take informal factors into consideration. Informal factors include:Informal factors that influence goal congruence

Work ethic Work ethic is a set of attitudes which are manifested in employees loyalty to the organization, their diligence spirit and pride in doing a good job. Some of these attitudes are local or specific to the city or region in which the organization does its work. Some attitudes and norms are industry-specific. India and China have a reputation for excellent work ethics. Within India, the work ethic in Western India is better than the work ethic in Eastern India.

Work culture The common beliefs, shared values, norms of behaviour and assumptions that are implicitly accepted and explicitly manifested throughout the organization are known as work culture. Organization culture is a very important factor which influences human behaviour in an informal manner. The culture of an organization is ever-lasting. An organizations culture has a major influence on the design of management control systems. Organizations may have really good formal management control systems but the one which has the desired culture is in a position to exercise better control.

Management style Management style is the attitude of a managers superior towards control. This gets reflected to some extent in the attitude of the subordinates. Management style has the most influence on management control. An institution is the lengthened shadow of a man. Managers come in all shapes and sizes.

Perception and communication Managers are informed about the goals of the organization and the actions they are expected to take in order to achieve these goals through a number of channels. These channels may be formal or informal. Formal means of communication include the corporate plan, budgets and other documents. The informal channels include conversation, discussion etc.

The informal organization The official authority-responsibility relationship of each manager is shown in the line chart of an organization. However, in an informal organization, there is no clear-cut authority-responsibility relationship. The finance managers interaction with the managers in the marketing or production department constitutes the informal organization. It is important in enabling the managers to comprehend the realities underlying the process of management control. Thus, the realities of the management control process cannot be understood without

recognizing the importance of the relationships that constitute the informal organization.

Cooperation and conflict The top management is primarily concerned with the responsibility of achieving the goals of the organization. The management makes decisions from time to time and the same is communicated to responsibility centre managers lower down the hierarchy for implementation. Responsibility centre managers have their personal goals. The interactions among managers have an effect on the manner in which the plans are executed. There may be a strained relationship between the production departments and service departments. The circulars carrying instructions to responsibility centre managers have adverse reactions on the middle managers. These factors affect the personal needs of the managers. Thus, there are conflicts between the managers in the organization. Those conflicts are part and parcel of organizational life. Cooperation is opposite of conflict. Many a times we find that there is effective cooperation between the people in the organization and the responsibility centre managers. They have to achieve the organizations goals. The cooperation may arise out of personal relations or mutual independence. There should be a proper balance between cooperation and conflicts. Conflicts resulting from competition among managers for increments or promotions are healthy but there should be some limit. On the other hand, there should be some amount of cooperation for the smooth functioning of an organization.

5
a. This ans is from the bog http://mcs20112010.blogspot.in/2011_04_01_archive.html

Internal controls may be described in terms of: a) the objective they pertain to; and b) the nature of the control activity itself. Objective categorization Internal control activities are designed to provide reasonable assurance that particular objectives are achieved, or related progress understood. The specific target used to determine whether a control is operating effectively is called the control objective. Control objectives fall under several detailed categories; in financial auditing, they

relate to particular financial statement assertions,[5] but broader frameworks are helpful to also capture operational and compliance aspects: 1. 2. 3. 4. 5. 6. Existence (Validity): Only valid or authorized transactions are processed Occurrence (Cutoff): Transactions occurred during the correct period or Completeness: All transactions are processed that should be (i.e., no Valuation: Transactions are calculated using an appropriate Rights & Obligations: Assets represent the rights of the company, and Presentation & Disclosure (Classification): Components of financial

(i.e., no invalid transactions) were processed timely. omissions) methodology or are computationally accurate. liabilities its obligations, as of a given date. statements (or other reporting) are properly classified (by type or account) and described. 7. b. Pg 243 6 This ans is from the bog http://mcs20112010.blogspot.in/2011_04_01_archive.html To the extent the decision are decentralized top management may lose some control. Relying on control reports is not as effective as personal knowledge of an operation. With profit centre, top management must change its approach to control. Instead of personal direction senior management must rely to a considerable extent on management control reports. Competent units that were once cooperating as functional units may now compete with one another disadvantageously. An increase in one managers profit may decrease those of another. This decrease in cooperation may manifest itself in a manager unwillingness to refer sales lead to another business unit, even though that unit is better qualified to follow up on the lead in production decision that have undesirable cost consequence on other units or in the hoarding of personnel or equipment that from the overall company standpoint would be better off used in another units. Reasonableness-transactions or results appear reasonable relative to other data or trends

There may be too much emphasis on short run profitability at the expense of long run profitability. In the desire to report high current profits, the profit centre manager may skip on R&D, training, maintenance. This tendency is especially prevalent when the turnover of profit centre managers is relatively high. In these circumstances, manager may have good reason to believe that their action may not affect profitability until after they have moved to other job. There is no complete satisfactory system for ensuring that each profit centre by optimizing its own profit , will optimize company profits. If headquarter management is more capable or has better information then the average profit centre manager the quality of some of the decision may be reduced. Divisionalization may cause additional cost because it may require additional management staff personnel and recordkeeping and may lead to redundant at each profit centre.

Business units as profit centres:

Business units are usually set up at profit centres. Business unit managers tend to control product development, manufacturing, and marketing resources. They are in a position to influence revenue and cost and as such can be held accountable for the bottom line. However as pointed out in the next section a business unit manager authority may be constrained such constrained should be incorporated in designing and operating profit centre.

Constraint on business unit authority To realize fully the advantage of the profit centre concept the business unit manger would have to be as autonomous as the president of the independent company. As a practical matter however such autonomy is not feasible. If a company were divided into completely independent units the organization would be giving up the advantage of size and synergism. Also senior management authority that a board of director gives to the chief executive. Consequently business unit structure represents trade off between

business unit autonomy and corporate constraint. The effectiveness of a business units organization is largely dependent on how well these trade off are made. The performance of a profit centre is appraised by comparing actual results for one or more orf these measures with budgeting amounts. In addition, data on competitors and the industry provide a good cross check on the appropriate of the budget. Data for individual companies are available from the securities and exchange commission for about key business ratios; standard & poor computer services, Inc; Robert Morris associates annual statement studies; and annual survey published in fortune, business week, and Forbes. Trade associations publish data for the companies in their industries. Revenues: choosing the appropriate revenue recognition method is important. Should revenue be recognized at the time as order is received, at the time an order is shipped, or at the time cash is received? In addition to that decision, issues related to common revenues may need to be considered. There are some situations in which two or more profit centres participate in the sales effort that results in a sale; ideally, each should be given appropriate credit for its part in this transaction. Many companies have not given much attention to the solution of these common revenue problems. They take the position that the identification of price responsibility for revenue generation is too complicated to be practical and that sale personnel must recognize they are working not only for their own profit centre but also for the overall good of the company. They for example, may credit the business unit that takes an order for a product handled by the another unit with the equivalent of a brokerage commission or a finder fee. In the case of a bank the branch performing a service may be given explicit credit for that service even though the customer account is maintained in another branch. Role of controller budget. year. It should coordinate the work of budget departments in lower echelons It should administer the process of making budget revision during the It should publish procedure and forms for the preparation of the budget. It should provide assistance to budgetees in the preparation of their

It should analyze reported performance against budget, interprets the

result, and prepares summary report for senior management.

Q 7-11 Bertildas Questions not received


Q.7 Explain briefly various stages of management control process citing salient features of each. Management control process involves communication of information to the managers at various levels of hierarchy and their interactions arising out of them. These communications aim towards attaining the organization's goals. But individual managers have their personal goals also. For example, a young manager with good education, experience, personality and social background joins a company like Britannia Industries or Reliance. The company finds him fit for the position as per job specifications, appoints him and makes him aware of what the company expects of him. The young manager sets his goals of gaining rich experience for his career progress besides adequate compensation packages. Naturally, his actions will be directed towards achieving his own objectives and goals while serving the company. Thus, his self-interest and the best interest of the organization are apparently in conflict. But the best results can be achieved by perfectly matching the two interests and this is called 'goal congruence'. It is quite apparent that perfect congruence between the goals of the individual and the organization individual's goals and the organization's goals can never happen. Yet, the main purpose of a management control system is to assure goal congruence between the interest of the individual and the organization as far as practicable. Management control systems Formal and Informal Communication As mentioned earlier, all the communication of information may be either formal or informal. The formal communication system involves strategic plan, budgets, standards and reports whereas the informal communication is made through letters and memos, verbally or even by facial expression. Formal communications are all documented and addressed to the responsible managers for their information and actions, if necessary. However, the actions depend on the perception of the individual managers. Informal communication, on the other hand, relates to some external factors-work ethics, management style and culture. Added to these factors is the existence of an informal organization within the structured formal organization. Informality refers to the relaxation of sharp differentiation and explicit description of behavior as indicated in the hierarchy and thereby, moving away from superior/subordinate relationship. However, such relations depend on the personal capabilities of the manager such as education, experience, expertise, trust and cooperation. For example, Accounts Manager of Nasik Plant (see the organization chart in the diagram 3.2) reports to the General Manager of the Plant. While visiting the Corporate Office for attending a Training Course, he meets other colleagues, parallel officers and even the Finance Director. The latter communicates some important matter to him verbally and wants action thereon. Accounts Manager carried out the instructions so given. As per the organization chart, he should inform his General Manager, but it depends on his own perception of the situation, and he mayor may not report to the General Manager. Work Ethics, Management Style and Culture External factors like work ethics vary from place to place. Therefore, organization work culture depends on the general behavior of the people in the society where the organization situates. Work culture generally differs because of the life style and the attitude towards the work. For example, people of Mumbai lead very fast life.

Time has more value at Mumbai as compared to Kolkata, where people take things easily and leisurely. Japanese and Korean people have reputation for their excellent work culture. However, the most important internal factor is the organization's culture and climate. The culture refers to the set of common beliefs, attitudes, norms, relationships and assumptions that are explicitly or implicitly accepted and evidenced throughout the organization. The writer joined Union Carbide as an Assistant just three days before Christmas Eve. On the very second day, when he attended Christmas lunch, his table was shared by none other than the General Sales Manager Dr. W.R. Correa. He kept us amused with various stories of his recent tour abroad and recited Urdu 'shairies', even sharing jokes. Such a situation was unthinkable in Jessop & Co., where sharp differences were maintained at every level of hierarchy. Management control systems Climate is used to designate the quality of the internal environment that conditions the quality of cooperation, the development of individuals, the extent of members' dedication or commitment to organizational purpose and the efficiency with which that purpose is translated into results. Climate is the atmosphere in which individuals work help, judge, and reward, constrain and find out about each other. It influences moral-the attitude of the individual towards his/her work and environment. Culture differs between the organizations, but cultural norms are extremely important. They are not written like formal communication. But the existence of a good culture can be felt from the behavior of the members of the organization. Once the writer landed up with his family at Hyderabad in the early morning to discover that nobody had come to receive them at the station. His visit was arranged through non other than the Director of the company himself. His unit being new, telephone directory did not include any number of his unit, but the parent organization's telephone number was located. When an executive of the parent company was contacted, he immediately sent an officer of the company with a car to pick us up to their Guest House, entertain with coffee and then put up in a Hotel. What subsequently happened is a different matter, but the attitude and treatment of that member of organization speak volumes about their excellent culture. In any organization, the culture remains unchanged as long as the Chief Executive remains in position. When a new executive replaces him, there is likelihood of some change in the culture, unless the new Chief follows the footsteps of his predecessor and maintains it. Generally, if higher positions are filled in through promotion of internal executives, the culture remains unchanged and the traditions are maintained. The other important internal factor which influences management control system is management style-that is the attitude of the superior to his subordinates and the latter's reaction through their perception of the attitude of their superiors. Again, the attitude ultimately stems from the temperament of the Chief Executive, who controls the entire organization. That is why R. W. Emerson said "an institute is the lengthened shadow of a man". Importance of Informal Communication An organization indulges in informal control process when encountering non-routine decision-making or when seeking new information to increase understanding of some problem areas. During a very critical period in an organization, the writer found that the Chief Executive used to call managers informally at his residence or club to extract information in a relaxed manner rather than in a tense situation prevailing in the factory. Formal Control Process Formal communication system is structured as per the 'hierarchy outlined in the organization chart. The system has the following four components: (a) Strategic plan and programme (b) Budgeting

(c) Operations and measurement in responsibility centers (d) Reporting (a) Strategic Plan and Programme The foundation of management control process lies in the organization's goals and its strategies for attaining these goals. A strategic plan is prepared in order to implement the strategies, after carefully considering opportunities and threats in the external environment as well as the strengths and weaknesses in the internal environment. Thus, a strategic plan and programme is prepared as a guideline to budgeting. (b) Budgeting The strategic plan is converted to an annual budget incorporating planned expenditure and revenues for individual responsibility centers. Expenses and revenues are marked for each responsibility centre period wise, say monthly, quarterly, half yearly, and annually. (c) Operations and Measurement Responsibility centers operate within the framework of the budget, established standards, standing instructions, practices and operating procedures embodied in 'rules', and 'manuals'. Thus, besides budget, the responsibility centers are also guided by a large number of rules. They record the resources actually used and revenue earned. They also classify the data by programmes as well as by responsibility centers for performance measurement. (d) Reporting Actual performance is analyzed, measured and reported against plan, indicating variances and highlighting areas of weaknesses. If the performance is satisfactory, feedback information is sent to the responsibility centre concerned for praise or reward. If the same is unsatisfactory feedback communication is sent to the responsibility centre concerned for corrective action. If such action requires to be included in the budget, then the latter is revised to give effect to the changed position. If required, then the plan itself can be revised and a new basis of control may be established. Q 8For an engineered (standard) expense center, zero based budgets is not necessary- Comment in detail?(2003) Engineered expenses centre is place or centre where value of input measured in monitory term. In engineered expenses centre we measure the performance of each input in relation with there output, in zero budget review also we measure the performance of input through the review of each expense centre. In engineered expense centre output multiplied by the standard cost of each unit produced measure what the finished product should have costs. The difference between the theoretical cost and actual cost represents the efficiency of the expenses centre. In zero budget review also the further review of performance analyses In Engineered Expense centre the manager of centre is responsible for maintaining the based quality of product .In zero based review the further the quality of the product analyzed .It does not require any further analysis if the manager in expense centre maintain or do there job properly there is no requirement of any further review In expense centre the manager is

responsible for timely delivery of product ,training and development of his employee. In zero based review the review is made to check should the function be performed in right way or not. In zero based review the review attempt to ascertain move that is from scratch the resources actually required to carry out each activity. This analysts established anew base the annual budget simply tries to keep the cash in line with this new base .engineered expenses centre are usually found in manufacturing operations .warehousing /distribution ,trucking and similar units within the marketing organization may also be engineered .Expense centre AS may certain responsibility centers within administrative and support departments for instance account receivable accounts payable and payroll sections in the controller department personnel records and cafeteria in the human resource department shareholders records in the corporate secretary department and the company motor pool. Such units perform repetitive tasks for which standard costs can be developed these expense centre are usually located within departments they are discretionary expense centre

Q.9 a. Briefly explain the nature of the information needed by managers to carryout control activities. Ans. Introduction Management control is a must I any organization that practices decentralization. One view argues that management control system must fit the firms strategy. This implies the strategy is first developed through a formal and rational process, and this strategy thendictates the design of the firms management systems. An alternative perspective is that strategies emerge through experimentation, which are influenced by the firms management systems. In this view, management control system can effect the development of strategies. We will consider both points of view, as well as their implications in terms of the design and operation of management control systems. When firms operate in industry contexts where environmental changes are predictable, they can use a formal and rational process to develop the strategy first and then design management control systems to execute that strategy. Defination Management control is the process by which managers influence other members of the organization to implement the organizations strategies. Control Press the accelerator and your car go faster rotate the steering wheel, and it changes direction. Press the break pedal, and the car slows or stops. With these devices, you control speed and direction; if any of the inoperative the car does not do what you want it to. In other words it is out of control. An organization must also be controlled; that is, devices must be in place to ensure that its strategic intentions are achieved. But controlling an organization is much more complicated than controlling a car. We will begin by describing the control process in simpler systems. Information needed by managers to carryout control activities To carryout control activities the manager needed some information which is based on

the following elements of a control system. Every Control system has at least four elements; A detector or sensor a device that measures what is actually happening in the process being controlled. An assessor a device that determines the significance of what is actually happening by comparing it with some standard or expectation of what should happen. An effector a device (often called feedback) that alters behavior if the assessor indicates the need to d so. A communications network devices that transmit information between he detector and the assessor and between the assessor and effector. These four basic elements of any control system are described their functioning by giving following examples Thermostat The components of the thermostat are (1) a thermometer (the detector), which measures the current temperature of a room; which compares the current temperature of a room; (2) an assessor, which compares the current temperature with the accepted standard for what the temperature should be; (3) an effector, which prompts a furnace to emit heat ( if the actual temperature is lower than the standard ) or activates an air conditioner (if the actual temperature is higher than the standard) and which also shuts off these appliances when the temperature reaches the standard level; and (4) a communications network, which transmits information from the thermometer to the assessor and from the assessor to the heating or cooling elements.

Q 9 b. Briefly explain with suitable diagrammatic representation, key differences between a functionally structured and profit center decentralized company. Explain two major advantages and disadvantages of each. Ans. Functionally Structured Company: Functionally structured company is co. in which each manager is responsible for a specific function such as production or marketing. The rational for the functional form of organization involves the notion of a manager who brings specialized knowledge to bear on decisions related to a specific function as contracted with general purpose manager who lacks specialized knowledge. A skilled marketing manager and a skilled production manager are likely to make better decisions in their respective fields than would a manager responsible for both functions. Moreover the skilled specialist should be able to supervise workers in same function better than the generalist would; just as skilled higher level managers should be able to provide better supervision of lower level managers in same or similar function. Thus a important advantage of a functional structure is efficiency. Profit Center Decentralized Company: When a responsibility centers financial performance is measured in terms of profit (i.e., by the difference between the revenues and expenses), the center is called a Profit center. Profit is a particularly useful performance measure since it allows senior management to use one comprehensive indicator rather than several (some of which may be pointing in different directions). Functional Units Multibusiness companies are typically divided into business units, each of which is treated as an independent profit-generating unit. The subunit within these business units, however, may be functionally organized. It is sometimes desirable to constitute one or more of the functional units e.g., marketing, manufacturing, and service operations as profit centers. There is no guiding principle declaring that certain types of units are inherently profit centers and others are not. Managements decision as to whether a given unit should be a profit center is based on the amount of influence (even if not total control) the units manager exercise over the activities that affect the bottom line. Marketing A marketing activity can be turned into a profit center by charging it with the cost of the products sold. This transfer price provides the marketing manager with the relevant information to make the optimum revenue/cost trade-offs, and the standard practice of measuring a profit centers manager by the centers profitability provides a check on how

well these trade offs have been made. The transfer price charged to the profit center should be based on the standard cost, rather than the actual cost, of the products being sold. Using a standard cost base separates the marketing cost performance from that of the manufacturing cost performance, which is affected by changes in the level of efficiency that are beyond the control of the marketing manager. Manufacturing The manufacturing activity is usually an expense center, with the management being judged on performance versus standard costs and overhead budgets. This measure can cause problems, however, since it does not necessarily indicate how well the manager is performing all aspects of his job. For example, A manager may skip on quality control, shipping products of inferior quality in order to obtain standard cost credit. A manager may be reluctant to interrupt production schedules in order to produce a rush order to accommodate a customer. A manager who is measured against standard may lack the incentive to manufacture products that are difficult to produce or to improve the standard themselves. Service and support units Units for maintenance, information technology, transportation, engineering, consulting, customer service, and similar support activities can all be made into profit centers. These may operate out of headquarters and service corporate divisions, or they may fulfill similar functions within business units. They charge customer for service rendered, with the financial objective of generating enough business so that their revenues equal their expenses. Usually, the units receiving these services have the option of procuring them from on outside vendor instead, provided the vendor can offer services of equal quality at a lower price. Advantages & Disadvantages of Functionally Structured Company Advantages Smooth Functioning : The rational for the functional form of organization involves the notion of a manager who brings specialized knowledge to bear on decisions related to a specific function, as contrasted with the general purpose manager who lacks that specialized knowledge. A skilled marketing manager and skilled production manager are likely to make better decisions in their respective fields than would a manager responsible for both functions. A skilled manager helps for the smooth functioning of the business activities. Efficiency : The skilled specialist should be able to supervise workers in the same function better than the generalist would, just as skilled higher level managers should be able to provide better supervision of lower level managers in the same or similar function. Which helps to increase the efficiency of the labour. Disadvantages No unambiguous way of determining the effectiveness: In a functional organization there is no unambiguous way of determining the effectiveness of the separate functional managers (e.g., the managers of marketing and of production) because each function contributes jointly to the organizations final output. Therefore there is no way of measuring what fraction of profit was contributed by each. Similarly at lower levels in the organization there is no way of determining how much of the profit was earned respectively by the several production departments, the product engineering department, and the sales office. If the organization consist of managers in one function who report to higher level managers in the same function, who, in turn, report to still higher level managers in that function, then a dispute between managers of different functions can be resolved only at the top, even though it may have originated at a much lower organizational level. Advantages & Disadvantages of Profit center Company Advantages The quality of decisions may improve because they are being made by managers closest to the point of decision. The speed of operating decisions may be increased since they do not have to be referred to corporate headquarters. Headquarter management, relieved of day-to-day decision making, can concentrate on broader issues. Manager, subject to fewer corporate restraints, are freer to use their imagination and initiative. Disadvantages Decentralized decision making will force top management to rely more on management

control reports than on personal knowledge of an operation, entailing some loss of control. If headquarter management is, more capable or better informed than the average profit center manager, the quality of decisions made at the unit level may be reduced. All the above are the advantages and disadvantages of the Functionally structured organization and profit center decentralized organization. Q 10) Part of a multinational group, Sonlya Shoe Company(SSC), established its own facilities in India over 75 years ago and enjoyed an excellent record-high market share for its diverse range of shoes, growth and profits. SC markets its products through company owned shops and its own personnel. Organization structure is functional. Since 2001, profitability, market share are slipping. Pressure from cheap Chinese shoes and also premium shoes like Nike has made the company think< of organizational restructuring and introducing Comensurate Control System to regain its position. Although SSC outsources, 30% of products, it is seen as a production oriented company. SSC wants to adopt measures to reduce costs, strengthen marketing and be in a position to produce and meet unexpected and unusual customer demands. How should the company reorganize to achieve Goal Congruence. Define Performance Metric? In a goal congruent process, the actions people are led to take in accordance with their perceived self-interest are also in the best interest of the organization. A firms strategy has a major influence on its structure. The type of structure in turn influences the design of the organizations management control systems. SONLYA Shoe Companys (SSC) organization structure is functional which involves the notion of a manager who brings specialized knowledge to bear on decisions related to a specific function, vis--vis a general purpose manager who lacks the specialized knowledge. A skilled marketing and production manager would be able to make better decisions in their respective fields. He would also be able to supervise workers in the same function better than the generalist would. Thus an important advantage of the functional structure is efficiency. A major disadvantage of this structure is that there is no unambiguous way of determining the effectiveness of the separate functional managers because each function contributes jointly to the organizations final output. Therefore, there is no way of determining how much of the profit was earned respectively by the several production departments. SONLYA Shoe Company which was a market leader for a period of over 75 years has been losing market share, which has impacted its profitability. Also it needs to be seen that the company outsources about 30% of its products. The company aims to strengthen marketing, reduce costs and wants to be in a position to customize products as per the demands of the customer. Thus, Sundaram needs to re-organize its organization structure which is functional to a Business Unit form of organization. The benefits of the re-organization would be that the business unit or the division would be responsible for all the functions involved in producing and marketing a specified product line. The business managers act almost as if their units are separate companies. They are responsible for planning and co-coordinating the work of the separate functions. Their performance is measured by the profitability of the business unit. This is a valid criterion because profit reflects the activities of both marketing and production. Though business unit managers exercise broad authority over their units, headquarters reserves certain key prerogatives. Headquarters are responsible for obtaining funds for the company as a whole and allocating it to the business unit, as well as approving budgets and judging the performance of business unit managers, setting their compensation. A major advantage of the Business unit structure of organization is that because it is close to the market for its products than the headquarters, its manager may make sounder production and marketing decisions than headquarters might and the unit as a whole reacts to new threats or opportunities quickly. This re-organization would help in achieving goal congruence in the organization.

Performance Metrics are high-level measures what you are doing; that is, they assess your overall performance in the areas you are measuring. They are external in nature and are most closely tied to outputs, customer requirements, and business needs for the process. The performance measurement system should cover the following areas at a minimum: CUSTOMERS 1. 2. Performance against customer requirements Customer Satisfaction

PERFORMANCE OF INTERNAL WORK PROCESSES 1. 2. 3. Cycle times Product and service quality Cost performance (could be productivity measures, inventory, etc.)

SUPPLIERS 1. Performance of suppliers against your requirements

FINANCIAL 1. 2. Profitability (could be at the company, product line, or individual level) Market share growth and other standard financial measures

EMPLOYEE 1. Associate satisfaction

Q11

Q 12
Standard Cost Per Unit: Raw Materials Direct Labour Variable Overhead Total Standard Cost / Variable Cost Fixed Overhead Profit * Product A Product B Product C (Rs./ ut) (Rs./ut) (Rs./ut) 20 30 10 10 10 20 10 10 20 40 30 1 50 40 0.6 161.6 71 (Transfer Price of Transfer Price ** (Transfer price of 50 10 0.46

Product A) Product B) 71 161.6 222.06

* Profit = 10% return on fixed assets and inventory (Given) For Product A, Profit = 10% (7 lacs + 3 lacs) = 10% of 10 lacs = 1 lac. Hence, profit per unit = Rs. 1 lac / 1 lac units = unit (Solve similarly for Products B and C)

** Transfer Price = Variable Cost + Fixed Cost + Profit

MAY 2002 1. Short Notes: 1A) Impact of Management style on controls i. The management control system in an organization is influenced by the style of management of the organization. Various management styles adopted by the CEO, the business unit manager and departmental managers affect the management control processes in the organization, business unit and functional areas respectively. ii. The operations of the control systems of an organization are significantly influenced by various dimensions of management style. For example, two CEOs with two differing styles of management may interpret and use the same report consisting of same set of data very differently to manage the business units. Hence, if a CEO changes, the management control systems tend to change correspondingly. iii. Management style also affects the personal and impersonal controls in an organization. Some managers are task oriented and prefer to a large amount of quantitative information to deliver tentative conclusions. iv. On the other hand, some managers are more people oriented and develop a on the field approach to learn about the functioning of the processes of the organization through interaction with supervisors and staff. v. vi. Management style affects the degree of tight versus loose control in an organization. As one moves up the organization hierarchy, controls are loosened. Greater attention is paid to achievement of overall results rather than the details involved. vii. Inversely, as one moves down the organizational hierarchy, the controls tend to get more tightened as a greater emphasis is paid on the details of the processes. viii. It is the duty of the Designer of Management control systems to identify the management style in an organization and to apply suitable control systems. 1B) Free Cash Flow (FCF) i. Free cash flow is a measure of financial performance calculated as Operating cash flow minus capital expenditures.

ii.

FCF represents the cash that a company is able to generate after laying out the money required to maintain or expand its current asset base.

iii.

FCF helps in enhancing the shareholder value. It enables a company to develop new products, make acquisitions, pay dividends, reduce debts etc.

iv.

FCF is also at times referred as Owners Earnings because it is the amount of money that a shareholder can withdraw from the treasury of the company without harming the business.

v.

A positive FCF gives financial flexibility because the firm isn't relying on the capital markets to fund its expansion. Firms that have negative FCF have to take out loans or sell additional shares to keep things going, and can thus become a risky proposition if the market becomes unsettled at a critical time for the company.

vi.

FCF is calculated as: Net Income + Amortization / Depreciation Changes in Working Capital Capital Expenditures = Free Cash Flow [Where, Net Income + Depreciation Changes in WC = Cash flow from Operations]

1C) Management Control Process in Organizations i. Management control is the process by which managers influence other members of the organization, in order to implement the organizations strategies effectively and efficiently. ii. The following activities are involved in the management control: Planning the future activities Coordinating the activities of several parts of the organization Communication of the information Evaluation of the information Deciding the action to be taken Influencing people to change their behavior

iii.

Management control does not necessarily require that all action correspond to a predetermined plan. Plans are based on circumstances believed to exist at the time they are formulated. Hence, management control in an organization needs to be flexible.

iv.

A management control system is used by the management to help an organization in achieving its strategic objectives. An effective management control system should possess the following characteristics: Strategic Plan: There should be a strategic plan of an organization and the strategies should be communicated to the subordinates who are responsible for their execution. Profit Plan: To ensure a proper direction to the organization. It sets forth standards of measurement and shows the desired profit-cost relationships. Motivation: Subordinates should be constantly motivated to put in their best performance in order to achieve the organizational goals and to perform efficiently and effectively. Effective management information system: MIS is essential to control the activities of an organization and to measure the performance of the subordinates.

1D) Implication of differentiated strategies on control: As firms become more diversified, corporate-level managers may not have significant knowledge of, or experience in, the activities of the company's various business units. If so, corporate-level managers for highly diversified firms cannot expect to control the different businesses on the basis of intimate knowledge of their activities, and performance evaluation tends to be carried out at arm's length. Single-industry and related diversified firms possess corporate wide core competencies (on which the strategies of most of the business units are based. Communication channels and transfer of competencies across business units, therefore, are critical in such firms. In contrast, there are low levels of interdependence among the business units of unrelated diversified firms. This implies that as firms become more diversified, it may be desirable to change the balance in control systems from an emphasis on fostering cooperation to an emphasis on encouraging entrepreneurial spirit.

a) Strategic planning: i. Given the low level of interdependencies, conglomerates tend to use vertical strategic planning systems-that is, business units prepare strategic plans & submit to senior management to review & approve. ii. The horizontal dimension might be incorporated into the strategic planning process in a number of different ways. First, a group executive might be given the responsibility to develop a strategic plan for the group as a whole that explicitly identifies synergies across individual business units within the group. Second, strategic plans of individual business units could have an interdependence section, in which the general manager of the business unit identifies the focal linkages with other business units and how those linkages will be exploited. Third, the corporate office could require joint strategic plans for interdependent business units.
iii.

Finally, strategic plans of individual business units could be circulated to managers of similar business units to critique and review. These methods are not mutually exclusive. In fact, several of them could be pursued fruit. Fully at the same time.

b) Budgeting: i. The chief executives of single-industry firms may be able to control the operations of subordinates through informal and personally oriented mechanisms, such as frequent personal interactions. This lessens the need to rely as heavily on the budgeting system as the tool of control.
ii.

On the other hand, in a conglomerate it is nearly impossible for the chief executive to rely on informal interpersonal interactions as a control tool; much of the communication and control has to be achieved through the formal budgeting stem.

c) Transfer Pricing: i. Transfers of goods and services between business units are more frequent in singleindustry and related diversified firms than in conglomerates. ii. The usual transfer pricing policy in a conglomerate is to give sourcing flexibility to business units and use arm's-length market prices. However, in a single-industry or a related diversified firm, synergies may be important, and business units may not be given the freedom to make sourcing decisions. d) Incentive Compensation

i.

Conglomerates, in general, are more likely to use formulas to determine business unit managers' bonuses; that is, they may base a larger portion of the bonus on quantitative, financial measures, such as economic value added (EVA).

ii.

Senior managers of single-industry and related diversified firms tend to base a larger fraction of the business unit managers bonus on subjective factors

iii.

In the case of unrelated diversified firms, the incentive bonus of the 'business unit managers tend to be determined primarily by the profitabi1ity of that unit, rather than the profitability of the firm.

iv.

In contrast, single-industry and related diversified firms tend to base the incentive bonus of a business unit manager on both the performance of that unit and the performance of a larger organizational unit.

2. Under which conditions, Management is better advised not to create profit centres? Explain the advantages of creation of Profit Centres. Ans: A profit centre is an organizational unit in which both revenues and expenses are measured in monetary terms. In setting up a profit centres, a company devolves decision making power to those lower levels that possesses relevant information for making expense or revenue trade-offs. a) Following are the conditions under which creation of Profit Centres are not advisable:
i. Decentralized decision making will force top management to rely more on management control reports than on personal knowledge of an operation, entailing some loss of control. If headquarters of an organization is more capable or better informed than the average profit center manager, the quality of decisions made at the unit level way be reduced. Friction may increase because of arguments over the appropriate transfer price, the assignment of common costs, and the credit for revenues that were formerly generated jointly by two or more business units working together. ii. Organization units that once cooperated as functional units may now be in competition with one another. An increase in profits for one manager may mean a decrease for another. In such situation a manager may fail to refer sales leads to another business unit better qualified to pursue them; may hoard personnel or equipment that, from the overall companys, standpoint, would be better off used in another unit; or may make production decisions that have undesirable cost consequences for other units.

iii.

Divisionalization may impose additional costs because of the additional management, staff personnel, and record keeping required, and may lead to task redundancies at each profit center. Competent general managers may not exist in a functional organization because there may not have been sufficient opportunities for them to develop general management competence.

iv.

There may be too much emphasis on short-run profitability at the expense of long-run profitability. In the desire to report high current profits, the profit center manager may skimp on R&D, training programs, or maintenance. This tendency is especially prevalent when the turnover of profit center managers is relatively high. In these circumstances, managers may have good reason to believe that their actions may not affect profitability until after they have moved to other jobs.

v.

There is no completely satisfactory system for ensuring that optimizing the profits of each individual profit center will optimize the profits of the company as a whole.

b) Advantages of Creation of Profit Centers: i. ii. iii. iv. v. vi. vii. Delegation of decision making authority by the top management to those lower in the hierarchy. Provision of up-to-date information to the top management on the profitability of the companys individual components. Better quality of decisions from those closest to the point of decision making. Increase in speed of operating decisions Profit consciousness is enhanced among the managers because the managers are responsible for profits. Profit centers provide excellent training grounds for general management to gain the required experience in managing all functional areas. The contribution of each manager to the goal of the entire organization is easier to measure.

3. A) Describe the salient features of cost based and market based transfer pricing methods? The transfer price is the price charged by one profit centre of an organization for a product supplied to another business unit of the same organization. Hence, the transfer price is the price that the selling division charges the buying division for the product.

a) Cost based transfer pricing: A cost-based transfer price requires that the following criteria be specified: Actual cost or budgeted (standard) cost. Full cost or variable cost.

i.

The amount of markup, if any, to allow the upstream division to earn a profit on the transferred product.

According to the cost of production method, transfer price is equal to the actual cost of production. This method is used where the responsibility for profit performance in centralized. For the selling division, profit figures would be an underestimate as it will not earn any profit on goods and services sold to other divisions and for the purchasing division, would be highly inflated.

ii.

In the marginal cost method, transfer price is equal to the variable cost. This method is used when profitability of the company is the main objective. It ensures full utilization of capacity of the selling division.

iii.

In the cost of sales method, along with the actual cost of production, the transfer prices also consists of expenses on selling and distribution which are allowed to be recovered from the user division.

b) Market price based transfer pricing method: i. ii. iii. iv. v. Market prices are determined by the forces of demand and supply in the long run. The use of open market prices ensures that the maximization of divisional profits will be reflected in the maximization of the company profits. In case of usage of external market price, a division can earn profit on sales by selling internally as well as externally. When the selling division is not trading externally, market price is determined on the basis of market reports. A disadvantage of a market-based transfer price is that the prices for some commodities can fluctuate widely and quickly. Companies sometimes attempt to protect divisional managers from these large unpredictable price changes. 3) B) Explain the problems faced in pricing corporate services furnished by corporate services staff to business units in the company. Assume profit centre decentralization? Ans.
i.

Business Units as Profit Centres

Most business units are created as profit centres since managers in charge of such units typically control product development, manufacturing, and marketing resources. These managers are in a position to influence revenues and costs and as such can be held accountable

for the "bottom line." However, as pointed out in the next section, a business unit manager's authority may be constrained in various ways, which ought to be reflected in a profit center's design and operation. ii. Constraints on Business Unit Authority

To realize fully the benefits of the profit centre concept, the business unit manager would have to be as autonomous as the president of an independent company. As a practical matter, however, such autonomy is not feasible. If a company were divided into completely independent units, the organization would lose the advantages of size and synergy. Furthermore in delegating to business unit management all the authority that the board of directors has given to the CEO, senior management would be abdicating its own responsibility. Consequently, business unit structures represent trade-offs between business unit autonomy and corporate constraints. The effectiveness of a business unit organization is largely dependent on how well these trade-offs are made. iii. Constraints from Other Business Units

One of the main problems occurs when business units must deal with one another. It is useful to think of managing a profit centre in terms of control over three types of decisions: The product decision (what goods or services to make and sell), The marketing decision (how, where, and for how much are these goods or services to be sold?), and

The procurement or sourcing decision (how to obtain or manufacture the goods or services). If a business unit manager controls all three activities, there is usually no difficulty in assigning profit responsibility and measuring performance. In general, the greater the degree of integration within a company, the more difficult it becomes to assign responsibility to a single profit centre for all three activities in a given product line; that is, if the production, procurement, and marketing decisions for a single product line are split among two or more business units, separating the contribution of each business unit to the overall success of the product line may be difficult.

iv.

Constraints from Corporate Management

The constraints imposed by corporate management can be grouped into three types: Those resulting from strategic considerations: Most companies retain certain decisions, especially financial decisions, at the corporate level, at least for domestic activities. Consequently, one of the major constraints on business units results from corporate control over new investments. Business units must compete with one another for a share of the available funds. Thus, a business unit could find its expansion plans thwarted because another unit has convinced senior management that it has a more attractive program. Those resulting because uniformity is required: Companies impose some

constraints on business units because of the necessity for Uniformity. Oneconstraint is that business Units must conform to corporate accounting and MCS This constraint is especially troublesome for units that have been acquired from another company and that have been accustomed to using different systems. Those resulting from the economies of centralization.

4) What is a responsibility centre? List and explain different types of Responsibility Centres in the Organization? Ans: A responsibility centre is an organization unit that is headed by a manager who is responsible for its activities. In a sense, a company is a collection of responsibility centres. Each of which is represented by box on the on the organization are responsibility centres for section work shifts or other small organization units. At a higher level are departments or business units that consist of several of these smaller units plus staff and management people these larger units are also responsibility centre. And from the stand point of senior management and the board of directors, the whole company is responsibility centre although the term is usually used to refer to unit within the company. Types of Responsibility Centres: Revenue Centre

i.

Revenue centres are divisions that add to the cost of the organization, but only indirectly add to the profit of the company.

ii.

Revenue centres create incentives for managers to under fund their units in order to benefit themselves, and this under funding may result in adverse consequences for the company as a whole (reduced sales because of bad customer service experiences, for example). Because the cost centre has a negative impact on profit (at least on the surface) it is a likely target for rollbacks and layoffs when budgets are cut.

iii.

Financial investments in new equipment, technology and staff are often difficult to justify to management because indirect profitability is hard to translate to bottom-line figures.

iv.

Business metrics are sometimes employed to quantify the benefits of a revenue centre and relate costs and benefits to those of the organization as a whole.

Profit Centre i. A responsibility centre is called a profit centre when the manager is held responsible for both costs (inputs) and revenues (outputs) and thus for profit. ii. A profit centre is a big segment of activity for which both revenues and costs are accumulated: A centre, whose performance is measured in terms of both - the expense it incurs and revenue it earns, is termed as a profit centre. iii. The output of a responsibility centre may either be meant for internal consumption or for outside customers. In the latter case, the revenue is realized when the sales are made. That is, when the output is meant for outsiders, then the revenue will be measured from the price charged from customers. If the output is meant for other responsibility centre, then management takes a decision whether to treat the centre as profit centre or not. iv. In fact, any responsibility centre can be turned into a profit centre by determining a selling price for its outputs. For instance, in case of a process industry, the output of one process may be transferred to another process at a profit by taking into account the market price. Such transfers will give some profit to that responsibility centre. Although such transfers do not increase the Companys assets, they help in management control process.

Investment Centre i. An investment centre goes a step further than a profit centre does. Its success is measured not only by its income but also by relating that income to its invested capital, as in a ratio of income to the value of the capital employed.
ii.

In

practice,

the

term investment centre is

not

widely

used.

Instead,

the

term profit centre is used indiscriminately to describe centres that are always assigned responsibility for revenues and expenses, but may or may not be assigned responsibility for the capital investment. It is defined as a responsibility centre in which inputs are measured in terms of cost / expenses and outputs are measured in terms of revenues and in which assets employed are also measured. iii. A responsibility centre is called an investment centre, when its manager is responsible for costs and revenues as well as for the investment in assets used by his centre. He is responsible for maintaining a satisfactory return on investment i.e. asset employed in his responsibility centre.
iv.

The investment centre manager has control over revenues, expenses and the amounts invested in the centres assets. The manager of an investment centre is required to earn a satisfactory return. Thus, return on investment (ROI) is used as the performance evaluation criterion in an investment centre.

v.

In the Investment Centre, the manager in charge is held responsible for the proper utilization of assets. He is expected to earn a satisfactory return on the assets employed in his responsibility centre.

vi.

Investment centres are generally used only for relatively large units, which have independent divisions, both manufacturing and marketing, for their individual products.

Expense Centre:
i.

Expense Centres are responsibility centres whose inputs are measured in monetary terms but outputs are not measured in monetary terms. Expense centres are of two types: Engineered and Discretionary.

ii.

Engineered costs are the costs for which the proper amount can be estimated with reasonable reliability e.g. Cost of material, labour, supplies etc. These costs vary in relation to the output.

iii. iv.

Engineered expense centres are generally found in manufacturing operations. In engineered expense centres, control can be exercised by flexible budget. Discretionary costs are those costs for which no engineered estimates are possible. These costs depend on the judgment of the management.

v.

vi.

Discretionary expense centres include administrative and support units such as R&D and marketing activities. However, in these centres an optimal relationship cannot be established between the input and output.

vii.

In discretionary expense centre, control can be exercised by fixed or incremental budget.

Q 59 Nikhil not sent work as yet


Quest 10-12 Aditi to send Numericals

May 2001
Q1) How is RI(EVA) analysis carried out? Explain advantages and disadvantages. Ans) Economic Value added is defined as Excess profit of a firm after charging cost of capital. It is just a way of measuring an operations real profitability. Can be measured as: EVA = NOPAT (TCE*WACC) = NOPAT Cost Of Capital NOPAT Net Operating Profit after Tax TCE Total Capital Employed WACC Weighted Average Cost of Capital Advantages of EVA:

EVA eliminates economic distortions of GAAP to focus decisions on real economic results EVA provides for better assessment of decisions that affect balance sheet and income statement or tradeoffs between each through the use of the capital charge against NOPAT

EVA decouples bonus plans from budgetary targets EVA covers all aspects of the business cycle EVA aligns and speeds decision making, and enhances communication and teamwork

Disadvantages of EVA:

EVA does not control for size differences across plants or divisions EVA is based on financial accounting methods that can be manipulated by managers EVA may focus on immediate results which diminishes innovation EVA provides information that is obvious but offers no solutions in much the same way as historical financial statement do

Q 2) Discuss the significance of human behavioural patterns in management control. Ans2) Management control systems influence human behaviour. Good management control systems influence behaviour in a goal congruent manner; that is, they ensure that individual actions taken to achieve personal goals also help to achieve the organization's goals. The concept of goal congruence, describing how it is affected both by informal actions and by formal systems. Senior management wants the organization to attain the organization's goals. But the individual members of the organization have their own personal goals, and they are not necessarily consistent with those of the organization. The central purpose of a management control system, then, is to ensure a high level of what is called "goal congruence." In a goal congruent process, the actions people are led to take in accordance with their perceived self interest are also in the best interest of the organization. The significance of human behaviour patterns in management control system can be explained with the help of Informal Factors that influence Goal Congruence. In the informal forces both internal and external factors play a key role. External Factors External factors are norms of desirable behaviour that exist in the society of which the organization is a part. These norms include a set of attitudes, often collectively referred to as the work ethic,which is manifested in employees' loyalty to the organization, their diligence, their spirit, and their pride in doing a good job (rather than just putting in time). Some of these attitudes are local that is, specific to the city or region in which the organization does its work. In encouraging companies to locate in their city or state, chambers of commerce and other promotional organizations often claim that their locality has a loyal, diligent workforce. Other attitudes and norms are industry-specific. Still others are national; some countries, such as Japan and Singapore, have a reputation for excellent work ethics. Internal Factors Culture The most important internal factor is the organizations own culture-the common beliefs, shared values, norms of behaviour and assumptions that are implicitly and explicitly manifested

throughout the organization. Cultural norms are extremely important since they explain why two organizations with identical formal management control systems, may vary in terms of actual control. A company's culture usually exists unchanged for many years. Certain practices become rituals, carried on almost automatically because "this is the way things are done here." Others are taboo ("we just don't do that here"), although no one may remember why. Organizational culture is also influenced strongly by the personality and policies of the CEO, and by those of lower-level managers with respect to the areas they control. If the organization is unionized, the rules and norms accepted by the union also have a major influence on the organization's culture. Attempts to change practices almost always meet with resistance, and the larger and more mature the organization, the greater the resistance is. Management Style The internal factor that probably has the strongest impact on management control is management style. Usually, subordinates' attitudes reflect what they perceive their superiors' attitudes to be, and their superiors' attitudes ultimately stem from the CEO. Managers come in all shapes and sizes. Some are charismatic and outgoing; others are less ebullient. Some spend much time looking and talking to people (management by walking around); others rely more heavily on written reports. The Informal Organization The lines on an organization chart depict the formal relationships-that is, the official authority and responsibilities-of each manager. The chart may show, for example, that the production manager of Division A reports to the general manager of Division A. But in the course of fulfilling his or her responsibilities, the production manager of Division A actually communicates with many other people in the organization, as well as with other managers, support units, the headquarters staff, and people who are simply friends and acquaintances. In extreme situations, the production manager, with all these other communication sources available, may not pay adequate attention to messages received from the general manager; this is especially likely to occur when the production manager is evaluated on production efficiency rather than on overall performance. The realities of the management control process cannot be understood without recognizing the importance of the relationships that constitute the informal organization. Perception and Communication

In working toward the goals of the organization, operating managers must know what these goals are and what actions they are supposed to take in order to achieve them. They receive this information through various channels, both formal (e.g., budgets and other official documents) and informal (e.g., conversations). Despite this range of channels, it is not always clear what senior management wants done. An organization is a complicated entity, and the actions that should be taken by anyone part to further the common goals cannot be stated with absolute clarity even in the best of circumstances. Moreover, the messages received from different sources may conflict with one another, or be subject to differing interpretations. For example, the budget mechanism may convey the impression that managers are supposed to aim for the highest profits possible in a given year, whereas senior management does not actually want them to skimp on maintenance or employee training since such actions, although increasing current profits, might reduce future profitability. The informal factors discussed above have a major influence on the effectiveness of an organizations management control. The other major influence is the formal systems. These systems can be classified into two types: (1) the management control system itself and (2) rules, which are described in this section. The Formal Control System Rules We use the word rules as shorthand for all types of formal instructions and controls, including: standing instructions, job descriptions, standard operating procedures, manuals, and ethical guidelines. Rules range from the most trivial (e.g., paper clips will be issued only on the basis of a signed requisition) to the most important):e.g., capital expenditures of over $5 million must be approved by the board' of directors). Some rules are guides; that is, organization members are permitted, and indeed expected, to depart from them, either under specified circumstances or when their own best judgment indicates that a departure would be in the best interests of the organization. Some rules are positive requirements that certain actions be taken (e.g., fire drills at prescribed intervals). Others are prohibitions against unethical, illegal, or other undesirable actions. Finally, there are rules that should never be broken under any circumstances: a rule prohibiting

the payment of bribes, for example, or a rule that airline pilots must never take off without permission from the air traffic controller. Some specific types of rules are listed below: Physical Controls Security guards, locked storerooms, vaults, computer passwords, television surveillance, and other physical controls may be part of the control structure. Manuals Much judgment is involved in deciding which rules should be written into a manual, which should be considered to be guidelines rather than fiats, how much discretion should be allowed, and a host of other considerations. Manuals in bureaucratic organizations are more detailed than are those in other organizations; large organizations have more manuals and rules than small ones; centralized organizations have more than decentralized ones; and organizations with geographically dispersed units performing similar functions (such as fastfood restaurant chains) have more than do single-site organizations System Safeguards Various safeguards are built into the information processing system to ensure that the information flowing through the system is accurate, and to prevent (or at least minimize) fraud of every sort. These include: cross-checking totals with details, requiring signatures and other evidence that a transaction has been authorized, separating duties, counting cash and other portable assets frequently, and a number of other procedures described in texts on auditing. Task Control Systems Task control is the process of assuring that specific tasks are carried out efficiently and effectively. Many of these tasks are controlled by rules. If a task is automated, the automated system itself provides the control.
Ans: 3. (a) Explain how different types of expense centres operate with help of sketches. Expense centres are responsibility centres whose inputs are measured in monetary terms but outputs are not measured in monetary terms. There are two types of expense centres, engineered and discretionary.

1. Engineered expense centres:

Optimal relationship can be

INPUTS S PROCESS

OUTPUTS

Rupees

Physical

These are generally found in manufacturing operations. In an engineered expense centre an optimal relationship can be established between the inputs and he outputs. Engineered costs are costs for which the right or proper amount can be estimated with reasonable reliability. For e.g. cost of material, labour, supplies. The output of engineered expense centre gives the standard cost of finished product if it is multiplied by the standard cost of each unit manufactured. The difference between the engineered cost and actual costs represents the efficiency of the organizational unit.

2. Discretionary expense centres: Discretionary expense centres include administrative and support units, R & D and marketing activities. For e.g. accounting legal, industrial relations, public relations and human resource In an engineered expense centre an optimal relationship cannot be established between the inputs and he outputs. The term discretionary does not mean that managements judgement as to optimum cost is capricious or haphazard. The difference between budgeted input and actual input measures the efficiency of this expense centre.

Optimal relationship cannot be established


INPUTS OUTPUTS

PROCESS
Rupees Physical

R & D Function

Ans: 3. (b) Explain the significant features and the budget process in these centres.

1. Engineered expense centres: Inputs can be measured in monetary terms. Outputs are not measured in monetary terms; they are measured in physical terms. Optimum amount of inputs which are required to produce one unit of output can be established. The output of engineered expense centre gives the standard cost of finished product if it is multiplied by the standard cost of each unit manufactured. The difference between the engineered cost and actual costs represents the efficiency of the organizational unit.

2. Discretionary expense centres: Inputs can be measured in monetary terms. Outputs are measured in physical terms. An optimal relationship cannot be established between the inputs and he outputs. The difference between budgeted input and actual input measures the efficiency of this expense centre.

Ans: 4. What is a profit centre? Explain the conditions under which profit centre decentralization will be the most beneficial to an organization. A responsibility centre is called a profit centre when the manager is held responsible for both costs (inputs) and revenues (outputs) and thus for profit. Despite the name, a profit centre can exist in nonprofits organizations (though it might not be referred to as such) when a responsibility centre receives revenues for its services. A profit centre is a big segment of activity for which both revenues and costs are accumulated: A centre, whose performance is measured in terms of both - the expense it incurs and revenue it earns, is termed as a profit centre. The output of a responsibility centre may either be meant for internal consumption or for outside customers. In the latter case, the revenue is realized when the sales are made. That is, when the output is meant for outsiders, then the revenue will be measured from the price charged from customers. If the output is meant for other responsibility centre, then management takes a decision whether to treat the centre as profit centre or not. In fact, any responsibility centre can be turned into a profit centre by determining a selling price for its outputs. For instance, in case of a process industry, the output of one process may be transferred to another process at a profit by taking into account the market price. Such transfers will give some profit to that responsibility centre. Although such transfers do not increase the Companys assets, they help in management control process. Profit centre decentralization:

The idea of profit centres and decentralization often gets in the way of good management if the idea is taken very seriously. Such ideas are often not what they seem.

Many companies profess decentralization that does not really have it. Profit centres are not necessarily so if overall corporate profit performance is being optimized. Independent profit centres are by definition neither independent nor profit centres if, in fact, there is any significant mutual interaction or synergy between cost centres. There are several ambiguities involved. They grow out of the underlying assumptions and implications in the words profit centre and decentralization. Multi-business companies are generally divided into business units each of which is treated as an independent profit centre.

The sub-units within these business units can be functionally organized. It may be sometimes desirable to constitute one or more of the functional units as profit centre. It depends upon the management whether a unit should be a profit centre. It depends upon the amount of influence a unit manager exercises over the activities that affect the profit.

Ans: 5. (a) Transfer price is not an accounting tool Comment. If a group has subsidiaries that operate in different countries with different tax rates, manipulating the transfer prices between the subsidiaries can scale down the overall tax bill of the group. For example the tax rate in Country A is 20% and is 50% in Country B. In the larger interest of the group, it would be advisable to show lower profits in Country B and higher profits in Country A. For this, the group can adjust the transfer price in such a way that the profits in Country An increase and that in Country B get reduced. For this the group should fix a very high transfer price if the Division in Country A provides goods to the Division in Country B. This will maximize the profits in Country A and minimize the profits in Country B. The reverse will be true if the Division in Country A acquires goods from the Division in Country B. There is also a temptation to set up marketing subsidiaries in countries with low tax rates and transfer products to them at a relatively low transfer price. Transfer price is viewed as a major international tax issue. While companies indulge in all types of activities to lower their tax liability, the tax authorities monitor transfer prices closely in an attempt to collect the full amount of tax due. For this they enter into agreements whereby tax is paid on specific transactions in one country only. But if companies set unrealistic transfer price to minimize their tax liabilities and the same is spotted by the tax authority, then the company is forced to pay tax in both countries leading to double taxation. With a view to avoid such cases from recurring, Organisation for Economic Cooperation and Development issued some guidelines in 1995. These guidelines aim at encouraging world trade. They evolved what came to be known as the arm's length price. The principle states that the transfer price would be arrived at on the basis as if the two companies are independent and unrelated. The price is determined through: Comparable Price Method where the price is fixed on the basis of prices of similar products or an approximation to one. Gross Margin Method where a gross margin is established and applied to the seller's manufacturing cost.

In spite of all these efforts, it has to be admitted that setting a fair transfer price is not easy. So the onus of proving the price has been put on the taxpayer who is required to produce supporting documents. If the taxpayer fails to do this he is required to pay heavy penalty. For example, in USA, failure to provide documentary evidence results in a 40% penalty on the arm's length price. In UK the penalty is to the tune of 100% of any tax adjustment. Other countries are also in the process of evolving tight norms for the same. Countries across the globe also allow the taxpayer to enter into an Advance Pricing Agreement whereby dispute can be avoided and so also the costly penalty of double taxation and penalty. Ans: 5. (b) State the conditions under which transfer price mechanism is likely to induce Goal Congruence.

To provide adequate measures of profit for the division to ensure motivation for the divisional head and his team. Divisional interests are sacrificed for overall company interests without ignoring the interest of the concerned division. To provide adequate information to senior management for sue in the companys decision making and evaluation of divisional performance. To provide fair premise in treatment and due share both to transfer and transferee division.

Ans: 6. Explain with illustrations the different ways in which the profit objective of a profit centre can be stated and controlled. What role do corporate overhead allocations play in this process? In order to evaluate the economic performance of a profit centre, one must use net income after allocating all costs. However, in evaluating the performance of manager, any of five different measures of profitability can be used. 1) Contribution Margin: The logic behind using contribution margin as a measure is that fixed expenses are not controllable by the manager, and therefore he should focus on maximizing the spread between revenue and expenses. But the problem with this is that some fixed costs are controllable and all fixed costs are partially controllable. A focus on the contribution margin tends to direct attention away from this responsibility. 2) Direct Profit: This measure shows the amount that the profit centre contributes to the general overhead and profit of the corporation. It incorporates all expenses incurred in or directly traced to the profit centre, regardless of whether these items are entirely controllable by the

profit centre manager. A weakness of this measure is that it does not recognize the motivational benefit of charging headquarters costs. 3) Controllable Profit: Headquarters expenses are divided into two categories: controllable and non-controllable. The controllable expenses are controlled by business unit manager. Consequently, if these costs are included in the management system, the profit will be after the deduction of all expenses that are influenced by profit centre manager. 4) Income before Taxes: In this measure, all corporate overhead is allocated to profit centres. The basis of allocation reflects the relative amount of expense that is incurred for each profit centre. If corporate overheads are allocated to profit centres, budgeted costs, not actual costs, should be allocated. Then the performance report will show an identical amount in the budget and actual columns for such overheads. 5) Net Income: Here, companies measure performance of domestic profit centres at the bottom line, the amount of net income after income tax. There are two arguments 1) Income after tax is constant percentage of the pretax income, so there is no advantage in incorporating income taxes 2) many decisions that have impact on income taxes are made at headquarters, and it is believed that profit centre manager should not be judged by the consequences of these decisions. Role of Corporate overhead allocation: In income before taxes, all corporate overhead is allocated to profit centres based on the relative amount of expense each profit centre incurs. There are two arguments against such allocations. First, since the costs incurred by corporate staff departments such as finance, accounting, and human resource management are not controllable by profit centre managers, these managers should not be held accountable for them. Second, it may be difficult to allocate corporate staff services in a manner that would properly reflect the amount of costs incurred by each profit centre. There are, however, three arguments in favour of incorporating a portion of corporate overhead into the profit centres' performance reports. First, corporate service units have a tendency to increase their power base and to enhance their own excellence without regard to their effect on the company as a whole. Allocating corporate overhead costs to profit centres increases the likelihood that profit centre manager will question these costs, thus serving to keep head office spending in check. (Some companies have actually been known to sell their corporate jets because of complaints from profit centre managers about the cost of these expensive items.) Second, the performance of each profit centre will become more realistic and more readily comparable to the performance of competitors who pay for similar services. Finally, when managers know that their respective centres will not

show a profit unless all-costs, including the allocated share of corporate overhead, are recovered, they are motivated to make optimum long-term marketing decisions as to pricing, product mix, and so forth, that will ultimately benefit (and even ensure the viability of) the company as a whole. If profit centres are to be charged for a portion of corporate overhead, this item should be calculated on the basis of budgeted, rather than actual, costs, in which case the "budget" and "actual" columns in the profit center's performance report will show identical amounts for this particular item. This ensures that profit centre managers will not complain about either the arbitrariness of the allocation or their lack of control over these costs, since their performance reports will show no variance in the overhead allocation. Instead, such variances would appear in the reports of the responsibility centre that actually incurred these costs.

Ans: 7. (a) Explain the concept of Balanced Score Card.

The Balanced Scorecard (BSC) is a performance management tool which began as a concept for measuring whether the smaller-scale operational activities of a company are aligned with its larger-scale objectives in terms of vision and strategy.

By focusing not only on financial outcomes but also on the operational, marketing and developmental inputs to these, the Balanced Scorecard helps provide a more comprehensive view of a business, which in turn helps organizations act in their best long-term interests.

Organizations were encouraged to measurein addition to financial outputswhat influenced such financial outputs. For example, process performance, market share / penetration, long term learning and skills development, and so on.

The underlying rationale is that organizations cannot directly influence financial outcomes, as these are "lag" measures, and that the use of financial measures alone to inform the strategic control of the firm is unwise. Organizations should instead also measure those areas where direct management intervention is possible.

In so doing, the early versions of the Balanced Scorecard helped organizations achieve a degree of "balance" in selection of performance measures. In practice, early Scorecards achieved this balance by encouraging managers to select measures from three additional categories or perspectives: "Customer," "Internal Business Processes" and "Learning and Growth."

Ans: 7. (b) How can Balanced Score Card be implemented in an organization?

Implementing a Balanced Scorecard We can summarize the implantation of a balanced scorecard in four general steps; 1. Define strategy. 2. Define measure of strategy. 3. Integrate measures into the management system. 4. Review measures and result frequently.

Q 8 A) Internal Auditing means nothing but policing.

A policy is typically described as a principle or rule to guide decisions and achieve rational outcome(s). Policy differs from rules or law. While law can compel or prohibit behaviours (e.g. a law requiring the payment of taxes on income), policy merely guides actions toward those that are most likely to achieve a desired outcome. Policy or policy study may also refer to the process of making important organizational decisions, including the identification of different alternatives such as programs or spending priorities, and choosing among them on the basis of the impact they will have.

Internal auditing is an independent, objective assurance and consulting activity designed to add value and improve an organization's operations. It helps an organization accomplish its objectives by bringing a systematic, disciplined approach to evaluate and improve the effectiveness of risk management, control, and governance processes.

Internal auditing frequently involves measuring compliance with the entity's policies and procedures. Professionals called internal auditors are employed by organizations to perform the internal auditing activity. However, internal auditors are not responsible for the execution of company activities; they advise management and the Board of Directors (or similar oversight body) regarding how to better execute their responsibilities. Managers establish policies and processes to help the organization achieve specific objectives in each of these categories. Internal auditors perform audits to evaluate whether the policies and processes are designed and operating effectively and provide recommendations for improvement.

Hence Internal Auditing means nothing but policing.

Q 8 B) Identify some Internal Controls:

Internal control is a process designed to provide reasonable assurance regarding the achievement of objectives in the following categories: Effectiveness and efficiency of operations Reliability of financial reporting Compliance with applicable laws and regulations

1. People at every level of an organization affect internal control. Internal control is, to
some degree, everyone's responsibility. Within the University of California, administrative employees at the department-level are primarily responsible for internal control in their departments.

2. Effective internal control helps an organization achieve its operations, financial


reporting, and compliance objectives. Effective internal control is a built-in part of the management process (i.e., plan, organize, direct, and control). Internal control keeps an organization on course toward its objectives and the achievement of its mission, and minimizes surprises along the way. Internal control promotes effectiveness and efficiency of operations, reduces the risk of asset loss, and helps to ensure compliance with laws and regulations. Internal control also ensures the reliability of financial reporting (i.e., all transactions are recorded and that all recorded transactions are real, properly valued, recorded on a timely basis, properly classified, and correctly summarized and posted).

3. Internal control can provide only reasonable assurance - not absolute assurance
- regarding the achievement of an organization's objectives. Effective internal control helps an organization achieve its objectives; it does not ensure success. There are several reasons why internal control cannot provide absolute assurance that objectives will be achieved: cost/benefit realities, collusion among employees, and external events beyond an organization's control.

Internal control consists of five interrelated components as follows: Control (or Operating) environment Risk assessment Control activities Information and communication Monitoring Q 9 A) Factors which impact service organization: Absence of Inventory Buffer: Goods can be held as inventory, which is a buffer that dampens the impact on production activity of fluctuations in sales volume. Services cannot be stored. The airplane seat, hotel room, hospital operating room, or the hours of lawyers, physicians, scientists, and other professionals that are not used today are gone forever. Thus, although a manufacturing company can earn revenue in the future from products that are on hand today, a service company cannot do so. It must try to minimize its unused capacity. Moreover, the costs of many service organizations are essentially fixed in the short run. In the short run, a hotel cannot reduce its costs substantially by closing off some of its rooms. Accounting firms, law firms, and other professional organizations are reluctant to layoff professional personnel in times of low sales volume because of the effect on morale and the costs of rehiring and training. Difficulty in Controlling Quality: A manufacturing company can inspect its products before they are shipped to the consumer, and their quality can be measured visually or with instruments (tolerances, purity, weight, colour, and so on). A service company cannot judge product quality until the moment the service is rendered, and then the judgments are often subjective. Restaurant management can examine the food in the kitchen, but customer satisfaction depends to a considerable extent on the way it is served. The quality of education is so difficult to measure that few educational organizations have a formal quality control system. Labour Intensive: Manufacturing companies add equipment and automate production lines, thereby replacing labour and reducing costs. Most service companies are labour intensive and cannot do this. Hospitals do add expensive equipment, but mostly to provide better treatment, and this increases costs. A law firm expands by adding partners and new support personnel. Multi-Unit Organizations:

Some service organizations operate many units in various locations; each unit relatively small. These organizations are fast-food restaurant chains, auto rental companies, gasoline service stations, and many others. Some of the units are owned; others operate under a franchise. The similarity of the separate units provides a common basis for analyzing budgets and evaluating performance not available to the manufacturing company. The information for each unit can be compared with system wide or regional averages, and high performers and low performers can be identified. However because units differ in the mix of services they provide, in the resources that they use, and in other ways, care must be taken in making such comparisons. Q 9 B) Explain special characteristics of professional organization which would have a bearing on their control system. Ans: Special Characteristics of Professional Organization: Goals: A dominant goal of a manufacturing company is to earn a satisfactory profit, specifically a satisfactory return on assets employed. A professional organization has relatively few tangible assets; its principal asset is the skill of its professional staff, which doesn't appear on its balance sheet. Return on assets employed, therefore, is essentially meaningless in such organizations. Their financial goal is to provide adequate compensation to the professionals. In many organizations, a related goal is to increase their size. In part, this reflects the natural tendency to associate success with large size. In part, it reflects economies of scale in using the efforts of a central personnel staff and units responsible for keeping the organization up to- date. Large public accounting firms need to have enough local offices to enable them to audit clients who have facilities located throughout the world. Professionals: Professional organizations are labour intensive, and the labour is of a special type. Many professionals prefer to work independently, rather than as part of a team. Professionals who are also managers tend to work only part time on management activities; senior partners in an accounting firm participate actively in audit engagements; senior partners in law firms have clients. Education for most professions does not include education in management, but quite naturally stresses the skills of the profession, rather than management; for this and other reasons, professionals tend to look down on managers. Professionals tend to give inadequate weight to the financial implications of their decisions; they want to do the best job they can, re- I regardless of its cost. This attitude affects the attitude of support staffs and non-professionals in the organization; it leads to inadequate cost control. Output and Input Measurement: The output of a professional organization cannot be measured in physical terms, such as units, tons, or gallons. We can measure the number of hours a lawyer spends on a case, but this is a measure of input, not output. Output is the effectiveness of the lawyer's work, and this is not measured by

the number of pages in a brief or the number of hours in the courtroom. We can measure the number of patients a physician treats in a day, and even classify these visits by type of complaint; but this is by no means equivalent to measuring the amount or quality of service the physician has provided. At most, what is measured is the physician's efficiency in treating patients, which is of some use in identifying slackers and hard workers. Revenues earned is one measure of output in some professional organizations, but these monetary amounts, at most, relate to the quantity of services rendered, not to their quality (although poor quality is reflected in reduced revenues in the long run). Furthermore, the work done by many professionals is non repetitive. No two consulting jobs or research and development projects are quite the same. This makes it difficult to plan the time required for a task, to set reasonable standards for task performance, and to judge how satisfactory the performance was. Some tasks are essentially repetitive: the drafting of simple wills, deeds, sales contracts, and similar documents; the taking of a physical inventory by an auditor; and certain medical and surgical procedures. The development of standards for such tasks may be worthwhile, although in using these standards, unusual circumstances that affect a specific job must be taken into account. Small Size: With a few exceptions, such as some law firms and accounting firms, professional organizations are relatively small and operate at a single location. Senior management in such organizations can personally observe what is going on and personally motivate employees. Thus, there is less need for a sophisticated management control system, with profit centres and formal performance reports. Nevertheless, even a small organization needs a budget, a regular comparison of performance against budget, and a way of relating compensation to performance. Marketing: In a manufacturing company there is a clear dividing line between marketing activities and production activities; only senior management is concerned with both. Such a clean separation does not exist in most professional organizations. In some, such as law, medicine, and accounting, the profession's ethical code limits the amount and character of overt marketing efforts by professionals (although these restrictions have been relaxed in recent years). Marketing is an essential activity in almost all organizations, however. If it can't be conducted openly, it takes the form of personal contacts, speeches, articles, conversations on the golf course, and so on. These marketing activities are conducted by professionals, usually by professionals who spend much of their time in production work-that is, working for clients. In this situation, it is difficult to assign appropriate credit to the person responsible for "selling" a new customer. In a consulting firm, for example, a new engagement may result from a conversation between a member of the firm and an acquaintance in a company, or from the reputation of one of the firm's professionals as an outgrowth of speeches or articles. Moreover, the professional who is responsible for obtaining the engagement may not be personally involved in carrying it out. Until fairly recently, these marketing contributions were rewarded subjectively- that is, they were taken

into account in promotion and compensation decisions. Some organizations now give explicit credit, perhaps as a percentage of the project's revenue, if the person who "sold" the project can be identified. Q 10) ABC Division of Company X Budget Sales Less: Material & variable cost Contribution Less: Fixed Cost Less: Sales promotion Operating Profit 100 Net working Capital Fixed assets 40 103 37 3F 3A 200 120 80 30 10 40 Actual 185 109 76 30 7 39 Variance 15A 11A 4A 0 3A 1A

F=favorable A=Adverse Q 11) Company XYZ Particulars Budgeted (Rs. Per Unit) Direct and Variable Labour Cost Material Cost Fixed Overheads Total Cost Transfer Price Profit Investment ROI = Profit/Investment 2 6 2 10 12 2 2 20,000 60,000 20,000 1,00,000 1,20,000 20,000 1,00,000 20% Budgeted (Total in Rs.) Actual (Rs. Per Unit) Taken as 9600 2 5.7 19,200 54,720 20,000 93,920 1,15,065.6 22,000 1,10,000 20% Actual (Total in Rs.)

11.986

Despite of increase in investment by 10%, there is negligible difference in transfer price. Also the sales have decreased by 400 units. Therefore we can say that additional investment has not achieved any positive results Q 12) A, B, C, D & E Calculation of ROA Division Profit Fixed Assets Current Assets Total Assets ROA = (Profit/Total Assets)*100 31.25 11 6.25 9.167 18

A B C D E

150 110 50 55 90

400 200 300 200 100

80 800 500 400 400

480 1000 800 600 500

Calculation of EVA

Current Assets Require Divisi on A B C D E Profit 150 110 50 55 90 Amoun t 80 800 500 400 400 Rat e 5 5 5 5 5 d Earnings 4 40 25 20 20 t 400 200 300 200 100

Fixed assets Require Amoun Rat e 10 10 10 10 10 d Earnings 40 20 30 20 10 EVA 106 50 -5 15 60

EVA = Profit (Charges for Current & Fixed Asset)

Vous aimerez peut-être aussi